IASbaba's Flagship Course: Integrated Learning Programme (ILP) - 2024  Read Details

Posts

[DAY 04] 60 DAY RAPID REVISION (RaRe) SERIES for UPSC Prelims 2024 – HISTORY, CURRENT AFFAIRS & CSAT TEST SERIES!

Archives Hello Friends The 60 Days Rapid Revision (RaRe) Series is IASbaba’s Flagship Initiative recommended by Toppers and loved by the aspirants’ community every year. It is the most comprehensive program which will help you complete the syllabus, revise and practice tests on a daily basis. The Programme on a daily basis includes Daily Prelims MCQs from Static (Monday – Saturday) Daily Static Quiz will cover all the topics of static subjects – Polity, History, Geography, Economics, Environment and Science and technology. 20 questions will be posted daily and these questions are framed from the topics mentioned in the schedule. It will ensure timely and streamlined revision of your static subjects. Daily Current Affairs MCQs (Monday – Saturday) Daily 5 Current Affairs questions, based on sources like ‘The Hindu’, ‘Indian Express’ and ‘PIB’, would be published from Monday to Saturday according to the schedule. Daily CSAT Quiz (Monday – Friday) CSAT has been an Achilles heel for many aspirants. Daily 5 CSAT Questions will be published. Note – Daily Test of 20 static questions, 10 current affairs, and 5 CSAT questions. (35 Prelims Questions) in QUIZ FORMAT will be updated on a daily basis. To Know More about 60 Days Rapid Revision (RaRe) Series – CLICK HERE   60 Day Rapid Revision (RaRe) Series Schedule – CLICK HERE  Important Note Comment your Scores in the Comment Section. This will keep you accountable, responsible and sincere in days to come. It will help us come out with the Cut-Off on a Daily Basis. Let us know if you enjoyed today’s test 🙂  You can post your comments in the given format  (1) Your Score (2) Matrix Meter (3) New Learning from the Test Time limit: 0 Test-summary 0 of 35 questions completed Questions: 1 2 3 4 5 6 7 8 9 10 11 12 13 14 15 16 17 18 19 20 21 22 23 24 25 26 27 28 29 30 31 32 33 34 35 Information The following Test is based on the syllabus of 60 Days Plan-2023 for UPSC IAS Prelims 2022. To view Solutions, follow these instructions: Click on – ‘Start Test’ button Solve Questions Click on ‘Test Summary’ button Click on ‘Finish Test’ button Now click on ‘View Questions’ button – here you will see solutions and links. You have already completed the test before. Hence you can not start it again. Test is loading... You must sign in or sign up to start the test. You have to finish following test, to start this test: Results 0 of 35 questions answered correctly Your time: Time has elapsed You have scored 0 points out of 0 points, (0) Average score     Your score     Categories Not categorized 0% Your result has been entered into leaderboard Loading Name: E-Mail: Captcha: maximum of 70 points Pos. Name Entered on Points Result Table is loading No data available 1 ASHEESH KUMAR 2024/03/07 11:45 AM 14 31.82 % 1 2 3 4 5 6 7 8 9 10 11 12 13 14 15 16 17 18 19 20 21 22 23 24 25 26 27 28 29 30 31 32 33 34 35 Answered Review Question 1 of 35 1. Question Consider the following statements about Swami Dayanand Saraswati He opposed idolatry, rituals and priesthood. He considered the Puranas to be full of falsehood. He considered the Vedas to be fallible. How many of the above statements are not correct?  a) Only one b) Only two c) Only three d) All four Correct Solution (a) Statement 1 Statement 2 Statement 3 Correct Correct Incorrect He was against idolatry, rituals and priesthood, and attacked child marriages and the caste system based on birth; encouraged inter-caste marriages and widow marriages. He considered the Puranas to be full of falsehood. Swami Dayanand Saraswati considered the Vedas as eternal and infallible. Incorrect Solution (a) Statement 1 Statement 2 Statement 3 Correct Correct Incorrect He was against idolatry, rituals and priesthood, and attacked child marriages and the caste system based on birth; encouraged inter-caste marriages and widow marriages. He considered the Puranas to be full of falsehood. Swami Dayanand Saraswati considered the Vedas as eternal and infallible. Question 2 of 35 2. Question Consider the following statements about ‘Prarthana Samaj’ It gave approval to the existing caste system in the society. It denounced the traditional thought and practice prevailing against women. Mahadev Govind Ranade, R.G. Bhandakar and Debendranath Tagore were its prominent leaders. How many of the above statements are correct? a) Only one b) Only two c) All three d) None Correct Solution (a) Statement 1 Statement 2 Statement 3 Incorrect Correct Incorrect There was a four- point social agenda of Prarthana Samaj: (i) disapproval of caste system, (ii) women’s education, (iii) widow remarriage, and (iv) raising the age of marriage for both males and females. The members were all Hindus and remained so throughout. They wanted to reform the religion from within. They were only against the social evils prevalent then like child marriage, widow suppression, dowry, sati, untouchability, etc. and not against the religion per se. The Prarthana Samaj had its prominent leaders – Mahadeo Govind Ranade and R.G. Bhandarkar. Debendranath Tagore was involved with Brahma Samaj.   Notes: Prarthana Samaj – The Prarthana Samaj, a leading society for socio-religious reform was established on 31 March 1867 in Bombay by Atmaram Panduranga. Prarthana Samaj became very popular after M G Ranade joined the organisation. The Samaj was different from the Brahmo Samaj of Bengal in that it was not as radical and took a cautious approach to the reformist programmes. For this reason, it was better received by the public as well. They also propagated monotheism and denounced idol worship. They also accepted all religious teaching including Christian and Buddhist ideas. They were staunchly against the division of society into castes. Members of the Samaj had communal meals prepared by a cook from a ‘low caste’. They also ate bread that was baked by a Christian and drank water brought by a Muslim. The Samaj did not question the existence of God but fostered a firm belief in one god. They sang hymns during the society’s meetings. The Samaj also encouraged a firm love and reverence for God. It was also against certain tenets of Hinduism like god’s incarnations. Through the society was against idol worship, its members could continue to practice Hindu ceremonies at home. It was often said that the Prarthana Samaj paid homage to Hinduism but with a protest. The society had many programmes to propagate women’s education. It also opened homes for orphans and widows. It also supported widow remarriage. It established several schools as well that imparted western education. The society’s meetings were done in secret for fear of a backlash from the orthodoxy. In fact, the Prarthana Samaj never directly attacked the orthodox sections of society or Brahminical power. The Samaj was greatly influenced by the Brahmo Samaj and the Arya Samaj of Dayananda Saraswati, but it continued as an independent movement. Apart from Ranade, other important members of the Samaj included Sanskrit scholar Sir Ramakrishna Gopal Bhandarkar and political leader Sir Narayan Chandavarkar. Incorrect Solution (a) Statement 1 Statement 2 Statement 3 Incorrect Correct Incorrect There was a four- point social agenda of Prarthana Samaj: (i) disapproval of caste system, (ii) women’s education, (iii) widow remarriage, and (iv) raising the age of marriage for both males and females. The members were all Hindus and remained so throughout. They wanted to reform the religion from within. They were only against the social evils prevalent then like child marriage, widow suppression, dowry, sati, untouchability, etc. and not against the religion per se. The Prarthana Samaj had its prominent leaders – Mahadeo Govind Ranade and R.G. Bhandarkar. Debendranath Tagore was involved with Brahma Samaj.   Notes: Prarthana Samaj – The Prarthana Samaj, a leading society for socio-religious reform was established on 31 March 1867 in Bombay by Atmaram Panduranga. Prarthana Samaj became very popular after M G Ranade joined the organisation. The Samaj was different from the Brahmo Samaj of Bengal in that it was not as radical and took a cautious approach to the reformist programmes. For this reason, it was better received by the public as well. They also propagated monotheism and denounced idol worship. They also accepted all religious teaching including Christian and Buddhist ideas. They were staunchly against the division of society into castes. Members of the Samaj had communal meals prepared by a cook from a ‘low caste’. They also ate bread that was baked by a Christian and drank water brought by a Muslim. The Samaj did not question the existence of God but fostered a firm belief in one god. They sang hymns during the society’s meetings. The Samaj also encouraged a firm love and reverence for God. It was also against certain tenets of Hinduism like god’s incarnations. Through the society was against idol worship, its members could continue to practice Hindu ceremonies at home. It was often said that the Prarthana Samaj paid homage to Hinduism but with a protest. The society had many programmes to propagate women’s education. It also opened homes for orphans and widows. It also supported widow remarriage. It established several schools as well that imparted western education. The society’s meetings were done in secret for fear of a backlash from the orthodoxy. In fact, the Prarthana Samaj never directly attacked the orthodox sections of society or Brahminical power. The Samaj was greatly influenced by the Brahmo Samaj and the Arya Samaj of Dayananda Saraswati, but it continued as an independent movement. Apart from Ranade, other important members of the Samaj included Sanskrit scholar Sir Ramakrishna Gopal Bhandarkar and political leader Sir Narayan Chandavarkar. Question 3 of 35 3. Question Consider the following statements: He is known to represent a happy blend of Indian and western culture though he was a great Sanskrit scholar. He opposed the monopoly of Sanskrit to the upper castes only. He was the flag bearer of widow remarriage in India. He also served as the principal of Sanskrit College. Which of the following personalities is being described by the statements given above? a) Ishwar Chandra Vidyasagar b) David Hare c) Karsondas Mulji d) Radhakant Deb Correct Solution (a) a)       b)       c)       d)       Correct Incorrect Incorrect Incorrect Ishwar Chandra Vidyasagar: He was an Indian educator and social reformer. Vidyasagar championed the upliftment of the status of women in India, particularly in his native Bengal. Unlike some other reformers who sought to set up alternative societies or systems, he sought to transform society from within. He was the most prominent campaigner for Hindu widow remarriage and petitioned the Legislative council despite severe opposition and a counter-petition against the proposal with nearly four times more signatures by Radhakanta Deb and the Dharma Sabha. He was the principal of the Sanskrit College and opposed the monopoly of Sanskrit only to upper castes. David Hare: He was a Scottish watchmaker and philanthropist in Bengal, India. He founded many educational institutions in Calcutta such as the Hindu School, and Hare School, and helped in founding Presidency College. David Hare was instrumental in establishing the School Book Society on 6 May 1817. It took the initiative to print and publish textbooks in both English and Bengali. This society contributed substantially to the flowering of the Bengal Renaissance. Karsondas Mulji: He was a Gujarati language journalist, writer, and social reformer from India. He became a vernacular schoolmaster and started Satyaprakash, a weekly in Gujarati, in which he attacked what he perceived to be the immoralities of the Maharajas or hereditary high priests. Sir Raja Radhakanta Deb Bahadur: He was a scholar and a leader of the Calcutta conservative Hindu society. An accomplished scholar, Radhakanta was proficient in Sanskrit, Persian, and Arabic. He published Shabda Kalpadruma, a Sanskrit language dictionary. Radhakanta Deb always showed a marked interest in promoting education, particularly English education among the Hindus; he also advocated female education. Radhakanta Deb was actively involved in the establishment and activities of the Calcutta School Book Society in 1817 and the Calcutta School Society in 1818. Despite his contribution to the cause of education, he was a strong upholder of social conservatism. Although sati was not practiced in his own family, he came forward to defend the custom when the Government contemplated its abolition. Incorrect Solution (a) a)       b)       c)       d)       Correct Incorrect Incorrect Incorrect Ishwar Chandra Vidyasagar: He was an Indian educator and social reformer. Vidyasagar championed the upliftment of the status of women in India, particularly in his native Bengal. Unlike some other reformers who sought to set up alternative societies or systems, he sought to transform society from within. He was the most prominent campaigner for Hindu widow remarriage and petitioned the Legislative council despite severe opposition and a counter-petition against the proposal with nearly four times more signatures by Radhakanta Deb and the Dharma Sabha. He was the principal of the Sanskrit College and opposed the monopoly of Sanskrit only to upper castes. David Hare: He was a Scottish watchmaker and philanthropist in Bengal, India. He founded many educational institutions in Calcutta such as the Hindu School, and Hare School, and helped in founding Presidency College. David Hare was instrumental in establishing the School Book Society on 6 May 1817. It took the initiative to print and publish textbooks in both English and Bengali. This society contributed substantially to the flowering of the Bengal Renaissance. Karsondas Mulji: He was a Gujarati language journalist, writer, and social reformer from India. He became a vernacular schoolmaster and started Satyaprakash, a weekly in Gujarati, in which he attacked what he perceived to be the immoralities of the Maharajas or hereditary high priests. Sir Raja Radhakanta Deb Bahadur: He was a scholar and a leader of the Calcutta conservative Hindu society. An accomplished scholar, Radhakanta was proficient in Sanskrit, Persian, and Arabic. He published Shabda Kalpadruma, a Sanskrit language dictionary. Radhakanta Deb always showed a marked interest in promoting education, particularly English education among the Hindus; he also advocated female education. Radhakanta Deb was actively involved in the establishment and activities of the Calcutta School Book Society in 1817 and the Calcutta School Society in 1818. Despite his contribution to the cause of education, he was a strong upholder of social conservatism. Although sati was not practiced in his own family, he came forward to defend the custom when the Government contemplated its abolition. Question 4 of 35 4. Question Consider the following statements: The Aravipuram Movement in 1888 was started by Shri Narayan Guru and was for temple entry for the lower castes. Bahishkrit Hitakarini Sabha was founded by Dr. B. R Ambedkar in 1929 in Bombay to promote Social Equality and for Constitutional safeguards for the depressed classes. Harijan Sevak Sangh in 1932 was founded by Mahatma Gandhi at Pune for the removal of untouchability. How many of the above statements are correct? a) Only one b) Only two c) All three d) None Correct Solution (c) Statement 1 Statement 2 Statement 3 Correct Correct Correct The Aravipuram Movement in 1888 was started by Shri Narayan Guru and was for temple entry for the lower castes. Bahishkrit Hitakarini Sabha was founded by Dr. B. R Ambedkar in 1929 in Bombay to promote Social Equality and for Constitutional safeguards for the depressed classes. Harijan Sevak Sangh in 1932 was founded by Mahatma Gandhi at Pune for the removal of untouchability. Incorrect Solution (c) Statement 1 Statement 2 Statement 3 Correct Correct Correct The Aravipuram Movement in 1888 was started by Shri Narayan Guru and was for temple entry for the lower castes. Bahishkrit Hitakarini Sabha was founded by Dr. B. R Ambedkar in 1929 in Bombay to promote Social Equality and for Constitutional safeguards for the depressed classes. Harijan Sevak Sangh in 1932 was founded by Mahatma Gandhi at Pune for the removal of untouchability. Question 5 of 35 5. Question Which of the following features are associated with Arya Samaj? Started Shuddhi movement to reconvert the converts of Hinduism from Islam and Christianity. It condemned the system of polytheism and ritual like animal sacrifices. Advocated for a classless, casteless and united society. How many of the above statements are correct? a) Only one b) Only two c) All three d) None Correct Solution (c) Statement 1 Statement 2 Statement 3 Correct Correct Correct In the zeal of Protecting the hindu Society from the Onslaught of Christianity and Islam, Araya samaj started Shudhhi movement.   Dayanand launched a frontal attack on Hindu Orthodoxy, caste rigidities, untouchability, idolatry, polytheism, belief in magic, charms and animal sacrifices, taboo on sea voyages etc. Dayanand’s Vision of India included a casteless and classless society.   Incorrect Solution (c) Statement 1 Statement 2 Statement 3 Correct Correct Correct In the zeal of Protecting the hindu Society from the Onslaught of Christianity and Islam, Araya samaj started Shudhhi movement.   Dayanand launched a frontal attack on Hindu Orthodoxy, caste rigidities, untouchability, idolatry, polytheism, belief in magic, charms and animal sacrifices, taboo on sea voyages etc. Dayanand’s Vision of India included a casteless and classless society.   Question 6 of 35 6. Question Consider the following statements regarding Rahnumai Mazdayasnan Sabha: It was a Parsi reform movement founded by a group of English educated Parsis for the restoration of the Zoroastrian religion. The views of the Sabha were projected through the newspaper Hitavada. It was organized under the leadership of Pherozeshah Mehta. How many of the above statements are correct?  a) Only one b) Only two c) All three d) None Correct Solution (a) Statement 1 Statement 2 Statement 3 Correct Incorrect Incorrect The association had for its object ―the regeneration of the social condition of the Parsis and the restoration of the Zoroastrian religion to its pristine purity. Education as the driving force of social reform was its prime objective. The Sabha campaigned ardently for the education of Parsi priests and for the spread of Western education among the Parsis, including girls. Through its efforts, the age of marriage was increased and Parsi women achieved emancipation. The message of the reform was spread by the newspaper Rast Goftar (Truth teller). The Western-educated progressive Parsis like Dadabhai Naoroji, J.B. Wacha, S.S. Bangali and Naoroji Furdonji founded the Rahanumai Mazdayasanan Sabha (Religious Reform Association) in 1851.    Note: Hitavada began to be published in the year 1911 to project the views of the Servants of India Society, founded by Gopal Krishna Gokhale in 1905 with the help of MG Ranade. Incorrect Solution (a) Statement 1 Statement 2 Statement 3 Correct Incorrect Incorrect The association had for its object ―the regeneration of the social condition of the Parsis and the restoration of the Zoroastrian religion to its pristine purity. Education as the driving force of social reform was its prime objective. The Sabha campaigned ardently for the education of Parsi priests and for the spread of Western education among the Parsis, including girls. Through its efforts, the age of marriage was increased and Parsi women achieved emancipation. The message of the reform was spread by the newspaper Rast Goftar (Truth teller). The Western-educated progressive Parsis like Dadabhai Naoroji, J.B. Wacha, S.S. Bangali and Naoroji Furdonji founded the Rahanumai Mazdayasanan Sabha (Religious Reform Association) in 1851.    Note: Hitavada began to be published in the year 1911 to project the views of the Servants of India Society, founded by Gopal Krishna Gokhale in 1905 with the help of MG Ranade. Question 7 of 35 7. Question With regard to the Brahmo Samajists, consider the following statements: They have no faith in any scripture as an authority. They denounce polytheism and idol-worship. They are against caste restrictions. How many of the above statements are correct?  a) Only one b) Only two c) All three d) None Correct Solution (c) Statement 1 Statement 2 Statement 3 Correct Correct Correct Brahmo Samajists have no faith in any scripture as an authority. Brahmo Samajists denounce polytheism and idol-worship. Brahmo Samajists are against caste restrictions.   Brahmo Samaj: The Brahmo Samaj was a monotheistic sect of Hinduism. The movement began through meetings of Bengalis in Calcutta in 1828. One of the leading figures was Raja Ram Mohan Roy. This group was known as the Brahmo Sabha. The followers of Brahmo Samaj are known as Brahmo Samajists, they- Have no faith in any scripture as an authority. Have no faith in Avatars. Denounce polytheism and idol-worship. Are against caste restrictions. Make faith in the doctrines of Karma and Rebirth optional Incorrect Solution (c) Statement 1 Statement 2 Statement 3 Correct Correct Correct Brahmo Samajists have no faith in any scripture as an authority. Brahmo Samajists denounce polytheism and idol-worship. Brahmo Samajists are against caste restrictions.   Brahmo Samaj: The Brahmo Samaj was a monotheistic sect of Hinduism. The movement began through meetings of Bengalis in Calcutta in 1828. One of the leading figures was Raja Ram Mohan Roy. This group was known as the Brahmo Sabha. The followers of Brahmo Samaj are known as Brahmo Samajists, they- Have no faith in any scripture as an authority. Have no faith in Avatars. Denounce polytheism and idol-worship. Are against caste restrictions. Make faith in the doctrines of Karma and Rebirth optional Question 8 of 35 8. Question With regard to Ahmadiyya Movement, consider the following statements: It was founded by Mirza Ghulam Ahmad. It was based on the principles of universal religion of all humanity. Which of the statements given above is/are correct? a) 1 only b) 2 only c) Both 1 and 2 d) Neither 1 nor 2 Correct Solution (c) Statement 1 Statement 2 Correct Correct The Ahmadiyya Movement forms a sect of Islam which originated from India. It was founded by Mirza Ghulam Ahmad in Punjab in 1889. The Ahmadiyya Movement was based on liberal principles. It described itself as the standard-bearer of Mohammedan Renaissance, and based itself on the principles of universal religion of all humanity, opposing jihad (sacred war against non-Muslims). Ahmadiyya Movement: The Ahmadiyya Movement spread Western liberal education among Indian Muslims. The Ahmadiyya community is the only Islamic sect to believe that the Messiah had come in the person of Mirza Ghulam Ahmad to end religious wars and bloodshed and to reinstate morality, peace and justice. They believed in separating the mosque from the State as well as in human rights and tolerance. However, the Ahmadiyya Movement, like Baha’ism which flourished in the West Asian countries, suffered from mysticism. Incorrect Solution (c) Statement 1 Statement 2 Correct Correct The Ahmadiyya Movement forms a sect of Islam which originated from India. It was founded by Mirza Ghulam Ahmad in Punjab in 1889. The Ahmadiyya Movement was based on liberal principles. It described itself as the standard-bearer of Mohammedan Renaissance, and based itself on the principles of universal religion of all humanity, opposing jihad (sacred war against non-Muslims). Ahmadiyya Movement: The Ahmadiyya Movement spread Western liberal education among Indian Muslims. The Ahmadiyya community is the only Islamic sect to believe that the Messiah had come in the person of Mirza Ghulam Ahmad to end religious wars and bloodshed and to reinstate morality, peace and justice. They believed in separating the mosque from the State as well as in human rights and tolerance. However, the Ahmadiyya Movement, like Baha’ism which flourished in the West Asian countries, suffered from mysticism. Question 9 of 35 9. Question With regard to Faraizi Movement, consider the following statements: It developed under the leadership of Haji Shariatullah. It tried to establish a parallel government to the British. Which of the statements given above is/are correct? a) 1 only b) 2 only c) Both 1 and 2 d) Neither 1 nor 2 Correct Solution (c) Statement 1 Statement 2 Correct Correct Faraizi Movement developed among the peasants of Eastern Bengal, under the leadership of Haji Shariatullah. It was indigenous in origin and sought to purify Islam by purging all un-Islamic beliefs and practices. Haji Shariatullah through the movement tried to establish a parallel government to the British. He even organized a small army which attacked the Zamindars and European Indigo planters. Incorrect Solution (c) Statement 1 Statement 2 Correct Correct Faraizi Movement developed among the peasants of Eastern Bengal, under the leadership of Haji Shariatullah. It was indigenous in origin and sought to purify Islam by purging all un-Islamic beliefs and practices. Haji Shariatullah through the movement tried to establish a parallel government to the British. He even organized a small army which attacked the Zamindars and European Indigo planters. Question 10 of 35 10. Question Consider the following statements regarding Social and Religious reform movement: Lala Lajpat Rai opposed the appeal of Arya Samaj of embracing of Vedic values. Under Keshab Chandra Sen Brahmo Samaj campaigned for inter-caste marriages. Vinoba Bhave founded the Sarvodaya Samaj to work for refugees. How many of the above statements are correct?  a) Only one b) Only two c) All three d) None Correct Solution (b) Statement 1 Statement 2 Statement 3 Incorrect Correct Correct Lala Lajpat Rai was one of the members of Arya Samaj and he never opposed the appeal of Arya Samaj to the authority of Vedas in support of its social reform program. The Brahmo Samaj experienced another phase of energy and vigour when Keshab Chandra Sen was made the acharya by Debendranath Tagore soon after the former joined the Samaj in 1858. His views against the caste system, even open support to inter-caste marriages were echoed in Samaj’s meetings. Vinoba founded Sarvodaya Samaj after assassination of Gandhi in 1948 which actively worked for refugees. Incorrect Solution (b) Statement 1 Statement 2 Statement 3 Incorrect Correct Correct Lala Lajpat Rai was one of the members of Arya Samaj and he never opposed the appeal of Arya Samaj to the authority of Vedas in support of its social reform program. The Brahmo Samaj experienced another phase of energy and vigour when Keshab Chandra Sen was made the acharya by Debendranath Tagore soon after the former joined the Samaj in 1858. His views against the caste system, even open support to inter-caste marriages were echoed in Samaj’s meetings. Vinoba founded Sarvodaya Samaj after assassination of Gandhi in 1948 which actively worked for refugees. Question 11 of 35 11. Question With reference to the Theosophical society, which of the following statements is/are correct? It was established by Annie Besant in Madras. It accepted the Hindu beliefs like reincarnation and karma. Select the correct answer using the code given below: a) 1 only b) 2 only c) Both 1 and 2 d) Neither 1 nor 2 Correct Solution (b) Statement 1 Statement 2 Incorrect Correct The theosophical society was founded by Madame H.P. Blavatsky and Colonel M.S. Olcott in New York City, United States in 1875. Annie Besant was a member of the society. She was elected as its president after the death of Olcott in 1907. The society believed that a special relationship could be established between a person’s soul and God by contemplation, prayer, revelation, etc. It accepted the Hindu beliefs in reincarnation and karma, and drew inspiration from the philosophy of the Upanishads and Samkhya, Yoga and Vedanta schools of thought. Incorrect Solution (b) Statement 1 Statement 2 Incorrect Correct The theosophical society was founded by Madame H.P. Blavatsky and Colonel M.S. Olcott in New York City, United States in 1875. Annie Besant was a member of the society. She was elected as its president after the death of Olcott in 1907. The society believed that a special relationship could be established between a person’s soul and God by contemplation, prayer, revelation, etc. It accepted the Hindu beliefs in reincarnation and karma, and drew inspiration from the philosophy of the Upanishads and Samkhya, Yoga and Vedanta schools of thought. Question 12 of 35 12. Question Consider the following pairs: Personality                                               Movement Vishnu Shastri Pandit        Widow Remarriage Association Debendranath Tagore          Tatvabodhini Sabha Dadabhai Naoroji                 Parsi Law Association How many of the above pairs is/are correctly matched? a) Only one pair b) Only two pairs c) All three pairs d) None Correct Solution (c) Statement 1 Statement 2 Statement 3 Correct Correct Correct Vishnushastri Pandit was born in 1827 at Badhava at Satara District. He worked as a translator in the British government. He became very active in Widow Marriage [Vidhava Vivah] Movement. On 28 January 1866, Vishnushastri, inspired by Phule’s movement opened an institution to promote widow remarriage. The institution (a society) was known as Punar Vivahtojak Mandal or Widow Remarriage Association. The Tattwabodhinī Sabhā was a group started in Calcutta on 6 October 1839 as a splinter group of the Brahmo Samaj, reformers of Hinduism and Indian Society. The founding member was Debendranath Tagore, previously of the Brahmo Samaj, eldest son of influential entrepreneur Dwarkanath Tagore, and eventually father to renowned polymath Rabindranath Tagore. In 1859, the Tattwabodhinī Sabhā was dissolved back into the Brāhmo Samāj by Debendranath Tagore. Nowrozjee Furdoonjee and Dadabhai Naoroji founded the Rahnumai Mazdayasnan Sabha and did considerable efforts for the purification of the Zoroastrian religion which was being prejudiced by Hindu elements like child marriages, polygamy, separate dining of men and women, and the use of nirang for sterilizing customs. In 1855 the Parsi Law Association was organized at a public meeting attended by 3,000 Parsis with Maneckji Nusserwanji Petit, Furdoonjee, Sorobjee, and Dadabhai Nauroji as founding members. They were in charge later for the introduction of a uniform system of Parsi laws. All such important agenda was discussed in the Rahnumai Mazdayasnan Sabhas. Incorrect Solution (c) Statement 1 Statement 2 Statement 3 Correct Correct Correct Vishnushastri Pandit was born in 1827 at Badhava at Satara District. He worked as a translator in the British government. He became very active in Widow Marriage [Vidhava Vivah] Movement. On 28 January 1866, Vishnushastri, inspired by Phule’s movement opened an institution to promote widow remarriage. The institution (a society) was known as Punar Vivahtojak Mandal or Widow Remarriage Association. The Tattwabodhinī Sabhā was a group started in Calcutta on 6 October 1839 as a splinter group of the Brahmo Samaj, reformers of Hinduism and Indian Society. The founding member was Debendranath Tagore, previously of the Brahmo Samaj, eldest son of influential entrepreneur Dwarkanath Tagore, and eventually father to renowned polymath Rabindranath Tagore. In 1859, the Tattwabodhinī Sabhā was dissolved back into the Brāhmo Samāj by Debendranath Tagore. Nowrozjee Furdoonjee and Dadabhai Naoroji founded the Rahnumai Mazdayasnan Sabha and did considerable efforts for the purification of the Zoroastrian religion which was being prejudiced by Hindu elements like child marriages, polygamy, separate dining of men and women, and the use of nirang for sterilizing customs. In 1855 the Parsi Law Association was organized at a public meeting attended by 3,000 Parsis with Maneckji Nusserwanji Petit, Furdoonjee, Sorobjee, and Dadabhai Nauroji as founding members. They were in charge later for the introduction of a uniform system of Parsi laws. All such important agenda was discussed in the Rahnumai Mazdayasnan Sabhas. Question 13 of 35 13. Question Consider the following statements: After Raja Ram Mohan Roys death in 1833, Debendranath Tagore became the leader of the Brahmo Samaj. Debendranath tried to popularise the ideas of Ram mohan by founding the Tattvabodhini Sabha, which became a platform for the expression of progressive public opinions and religious views. Kandukuri Veerasalingam was influenced by Keshab Chandra Sen’s faction of Brahmo Samaj. How many of the above pairs is/are correctly matched? a) Only one pair b) Only two pairs c) All three pairs d) None Correct Solution (c) Statement 1 Statement 2 Statement 3 Correct Correct Correct Debendranath Tagore (1817-1905), father of Rabindranath Tagore and a product of the best in traditional Indian learning and Western thought, gave a new life to Brahmo Samaj and a definite form and shape to the theist movement, when he joined the Samaj in 1842. Tattvabodhini Sabha (founded in 1839) which, along with its organ Tattvabodhini Patrika in Bengali, was devoted to the systematic study of India’s past with a rational outlook and to the propagation of Rammohan’s ideas. Kandukuri Veeresalingam was one among them, deeply influenced by Keshab Chandra Sen. Incorrect Solution (c) Statement 1 Statement 2 Statement 3 Correct Correct Correct Debendranath Tagore (1817-1905), father of Rabindranath Tagore and a product of the best in traditional Indian learning and Western thought, gave a new life to Brahmo Samaj and a definite form and shape to the theist movement, when he joined the Samaj in 1842. Tattvabodhini Sabha (founded in 1839) which, along with its organ Tattvabodhini Patrika in Bengali, was devoted to the systematic study of India’s past with a rational outlook and to the propagation of Rammohan’s ideas. Kandukuri Veeresalingam was one among them, deeply influenced by Keshab Chandra Sen. Question 14 of 35 14. Question Which of the following social reforms were undertaken by the British prior to 1857? Abolition of Slavery and Child trafficking Suppression of Thugs Abolition of female infanticide and human sacrifice Abolition of caste and untouchability Legalizing widow remarriage Select the correct answer using the codes given below: a) 1, 2 and 5 only b) 1, 2, 3 and 5 only c) 1, 3, 4 and 5 only d) 2, 3, 4 and 5 only Correct Solution (b) Statement 1 Statement 2 Statement 3 Statement 4 Statement 5 Correct Correct Correct Incorrect Correct The British administration was urged to take steps to ameliorate the conditions of slaves and to ultimately abolish slavery. Slavery was abolished in 1843. Suppression of Thugs (1830). Social reforms including abolition of female infanticide and human sacrifice had undertaken by Lord Hardinge I in 1844-1848. In 1955, the government passed the Anti-Untouchability Law, making the practice of untouchability punishable and a cognizable offence. Widow remarriage had legalized in Widow Remarriage Act (1856). Incorrect Solution (b) Statement 1 Statement 2 Statement 3 Statement 4 Statement 5 Correct Correct Correct Incorrect Correct The British administration was urged to take steps to ameliorate the conditions of slaves and to ultimately abolish slavery. Slavery was abolished in 1843. Suppression of Thugs (1830). Social reforms including abolition of female infanticide and human sacrifice had undertaken by Lord Hardinge I in 1844-1848. In 1955, the government passed the Anti-Untouchability Law, making the practice of untouchability punishable and a cognizable offence. Widow remarriage had legalized in Widow Remarriage Act (1856). Question 15 of 35 15. Question With reference to the Wahabi movement, consider the following statements: It was a revivalist Islamic movement that aimed to return to the pure Islam and society of the Prophet’s times. It became a Quasi-nationalist movement and posed a potential danger to the colonial government in India. Which of the statements given above is/are correct? a) 1 only b) 2 only c) Both 1 and 2 d) Neither 1 nor 2 Correct Solution (c) Statement 1 Statement 2 Correct Correct The Wahabi movement by Shah Walliullah was essentially a revivalist movement that aimed to return to the pure Islamic society that went by the interpretations of the Quran and Hadis. The movement was initially directed against Sikhs in Punjab but after the annexation of Punjab by the British, it was targeted against the British. The movement fizzled out in front of the British military might. Wahabi Movement: Shah Walliullah (1702-62) inspired this essentially revivalist response to Western influences and the degeneration which had set in among Indian Muslims. He was the first Indian Muslim leader of the 18th century to organise Muslims around the two-fold ideals of this movement- Desirability of harmony among the four schools of Muslim jurisprudence which had divided the Indian Muslims (he sought to integrate the best elements of the four schools) Recognition of the role of individual conscience in a religion where conflicting interpretations were derived from the Quran and the Hades. Incorrect Solution (c) Statement 1 Statement 2 Correct Correct The Wahabi movement by Shah Walliullah was essentially a revivalist movement that aimed to return to the pure Islamic society that went by the interpretations of the Quran and Hadis. The movement was initially directed against Sikhs in Punjab but after the annexation of Punjab by the British, it was targeted against the British. The movement fizzled out in front of the British military might. Wahabi Movement: Shah Walliullah (1702-62) inspired this essentially revivalist response to Western influences and the degeneration which had set in among Indian Muslims. He was the first Indian Muslim leader of the 18th century to organise Muslims around the two-fold ideals of this movement- Desirability of harmony among the four schools of Muslim jurisprudence which had divided the Indian Muslims (he sought to integrate the best elements of the four schools) Recognition of the role of individual conscience in a religion where conflicting interpretations were derived from the Quran and the Hades. Question 16 of 35 16. Question Which of the following were the religious reasons that led to Revolt of 1857? Allowed an Indian who had converted to Christianity to inherit the property of his ancestors. The Company has made a policy promote the converted Indians in army and government services. The Company allowed Christian missionaries to function freely in its domain and even own land and property. How many of the above pairs are correctly matched?  a) Only one b) Only two c) All three d) None Correct Solution (b) Statement 1 Statement 2 Statement 3 Correct Incorrect Correct In 1850, a new law was passed to make the conversion to Christianity easier. This law allowed an Indian who had converted to Christianity to inherit the property of his ancestors. There was no policy to promote the converted Indians in army and government services. English-language education was actively promoted. After 1830, the Company allowed Christian missionaries to function freely in its domain and even own land and property.   Notes: The Revolt of 1857 was primarily an anti-imperialist and the sepoys and the civilians wanted to throw out the imperial rulers. Broadly viewed this rebellion on an extensive scale with an object of ending the country’s alien rule appeared as a “war of independence” for some people. To study about the factors responsible for the revolt, one can visualize that the help of political frustration, social inequality, religious beliefs and the imbalanced economy were exploded by the match stick of greased cartridges and the discontentment of the sepoys.Other religious reasons were- Laws were passed to stop practice of sati and to encourage the remarriage of widows. Many Indians began to feel that the British were destroying their religion, social customs, and traditional way of life. Incorrect Solution (b) Statement 1 Statement 2 Statement 3 Correct Incorrect Correct In 1850, a new law was passed to make the conversion to Christianity easier. This law allowed an Indian who had converted to Christianity to inherit the property of his ancestors. There was no policy to promote the converted Indians in army and government services. English-language education was actively promoted. After 1830, the Company allowed Christian missionaries to function freely in its domain and even own land and property.   Notes: The Revolt of 1857 was primarily an anti-imperialist and the sepoys and the civilians wanted to throw out the imperial rulers. Broadly viewed this rebellion on an extensive scale with an object of ending the country’s alien rule appeared as a “war of independence” for some people. To study about the factors responsible for the revolt, one can visualize that the help of political frustration, social inequality, religious beliefs and the imbalanced economy were exploded by the match stick of greased cartridges and the discontentment of the sepoys.Other religious reasons were- Laws were passed to stop practice of sati and to encourage the remarriage of widows. Many Indians began to feel that the British were destroying their religion, social customs, and traditional way of life. Question 17 of 35 17. Question Which of the following were the grievances which led to the Khurda uprising of 1817? British policy of resuming service tenures. Insistence on payment of revenue in a newly issued currency by the British. Unprecedented rise in the prices of food-stuff and salt. How many of the above statements are correct?  a) Only one b) Only two c) All three d) None Correct Solution (c) Statement 1 Statement 2 Statement 3 Correct Correct Correct Soon after taking over Khurda, British followed a policy of resuming service tenures. It bitterly affected the lives of the ex-militia of the state, the Paiks. The introduction of sicca rupee (silver currency) in the region and the insistence on payment of revenue in the new currency added to the grievances of the people. The severity of the measure was compounded on account of an unreasonable increase in the demand of revenue and also the oppressive ways of its collection. Consequently, there was large scale desertion of people from Khurda between 1805 and 1817. Yet, the British went for a series of short-term settlements, each time increasing the demands, not recognising either the productive capacity of the land or the paying capacity of the Ryots. No leniency was shown even in case of natural calamities, which Odisha was frequently prone to. Rather, lands of defaulters were sold off to scheming revenue officials or speculators from Bengal. An unprecedented rise in the prices of food-stuff and salt, which had become farfetched following the introduction of salt monopoly because of which the traditional salt makers of Odisha were deprived of making salt, and the auction of local estates in Calcutta, which brought in absentee landlords from Bengal to Odisha. Besides, the insensitive and corrupt police system also made the situation worse for the armed uprising to take a sinister shape. Khurda Uprising of 1817: Khurda was a small kingdom built up in the late 16th century in the south-eastern part of Odisha. During the early nineteenth century, its king Mukunda Dev II was deposed by British and he had to agree to an unfair settlement by the British. This unfair settlement commenced an era of oppressive foreign rule in Odisha, which paved the way for a serious armed uprising in 1817. Incorrect Solution (c) Statement 1 Statement 2 Statement 3 Correct Correct Correct Soon after taking over Khurda, British followed a policy of resuming service tenures. It bitterly affected the lives of the ex-militia of the state, the Paiks. The introduction of sicca rupee (silver currency) in the region and the insistence on payment of revenue in the new currency added to the grievances of the people. The severity of the measure was compounded on account of an unreasonable increase in the demand of revenue and also the oppressive ways of its collection. Consequently, there was large scale desertion of people from Khurda between 1805 and 1817. Yet, the British went for a series of short-term settlements, each time increasing the demands, not recognising either the productive capacity of the land or the paying capacity of the Ryots. No leniency was shown even in case of natural calamities, which Odisha was frequently prone to. Rather, lands of defaulters were sold off to scheming revenue officials or speculators from Bengal. An unprecedented rise in the prices of food-stuff and salt, which had become farfetched following the introduction of salt monopoly because of which the traditional salt makers of Odisha were deprived of making salt, and the auction of local estates in Calcutta, which brought in absentee landlords from Bengal to Odisha. Besides, the insensitive and corrupt police system also made the situation worse for the armed uprising to take a sinister shape. Khurda Uprising of 1817: Khurda was a small kingdom built up in the late 16th century in the south-eastern part of Odisha. During the early nineteenth century, its king Mukunda Dev II was deposed by British and he had to agree to an unfair settlement by the British. This unfair settlement commenced an era of oppressive foreign rule in Odisha, which paved the way for a serious armed uprising in 1817. Question 18 of 35 18. Question Which of the following statements is/are correct regarding Poligars revolt? Poligars or Palayakkars were the class of viceroys forming the second tier in the administrative system created by the Vijayanagar Empire. While the Northern Poligars fell under the kingdom of Mysore, those in southern Tamil Nadu had to deal with the revenue demands of the British East India Company. The poligars of North Arcot rose in rebellion against the company as they were deprived of their right to collect the ‘kawal’ fees. How many of the above statements are correct?  a) Only one b) Only two c) All three d) None Correct Solution (b) Statement 1 Statement 2 Statement 3 Incorrect Correct Correct The roots of the poligars or Palayakkars date back to 14th century to the beginning of Vijayanagar empire in south India who established a unique three tier administrative system thereby creating an administrative class called the ‘ Palayakkars’. This three-tier set up consisted of the emperor at Vijayanagar at the top of pyramid; the Nayakas or viceroys forming the second tier and the Palayakkars or polygars comprising the grassroot leaders in the hierarchy. This system allowed for an efficient revenue administration as well as created a mechanism to help raise an army, besides keeping generals, warriors, courtiers etc. By the 18th century While the Northern Polygars fell under the kingdom of Mysore, those in southern Tamil Nadu had to deal with the revenue demands of the British East India company. Between 1803 and 1805, the poligars of North Arcot rose in rebellion against the company as they were deprived of their right to collect the kawal fees (Kawal or ‘watch’ was an ancient institution of Tamil Nadu. It was a hereditary village police office with specified rights and responsibilities. Incorrect Solution (b) Statement 1 Statement 2 Statement 3 Incorrect Correct Correct The roots of the poligars or Palayakkars date back to 14th century to the beginning of Vijayanagar empire in south India who established a unique three tier administrative system thereby creating an administrative class called the ‘ Palayakkars’. This three-tier set up consisted of the emperor at Vijayanagar at the top of pyramid; the Nayakas or viceroys forming the second tier and the Palayakkars or polygars comprising the grassroot leaders in the hierarchy. This system allowed for an efficient revenue administration as well as created a mechanism to help raise an army, besides keeping generals, warriors, courtiers etc. By the 18th century While the Northern Polygars fell under the kingdom of Mysore, those in southern Tamil Nadu had to deal with the revenue demands of the British East India company. Between 1803 and 1805, the poligars of North Arcot rose in rebellion against the company as they were deprived of their right to collect the kawal fees (Kawal or ‘watch’ was an ancient institution of Tamil Nadu. It was a hereditary village police office with specified rights and responsibilities. Question 19 of 35 19. Question Consider the following pairs: Peasant Movement                Causes Moplah Uprisings      Rise in revenue demand and reduction of field size Deccan Peasants’       Uprising British failure to take up anti-famine measures Ramosi Uprising        Corrupt practices of Gujarati and Marwari moneylenders How many of the above pairs is/are correctly matched? a) Only one pair b) Only two pairs c) All three pairs d) None Correct Solution (a) Statement 1 Statement 2 Statement 3 Correct Incorrect Incorrect Moplah Uprisings (1836-1854; Malabar) – Against rise in revenue demand and reduction of field size, coupled with the oppression of officials. It resulted in widespread peasant unrest among the Moplahs of Malabar. Twenty-two rebellions took place between 1836 and 1854. None, however, proved successful. (The second Moplah uprising occurred after the Moplahs came to be organized by the Congress and the Khilafat supporters during the Non-cooperation Movement. But Hindu-Muslim differences distanced the Congress and the Moplahs from each other. By 1921, the Moplahs had been subdued.) Deccan Peasants’ Uprising by the Maratha peasants (1875; Kardeh village and Poona in Maharashtra) – The Deccan peasants uprising was directed mainly against the corrupt practices of the Marwari and Gujarati moneylenders. Social boycott of money lenders by the peasant was later transformed into armed peasant revolt in the Poona and Ahamadnagar districts of Maharashtra. The peasants attacked the money lenders’s houses, shops, and burnt them down. Phadke’s Ramosi Uprising by Ramosi peasants led by Vasudev Balwant Phadke (1877-1887; Maharashtra) – Against the British failure to take up anti-famine measures. It was protested in the violent manner not in the peaceful manner. Incorrect Solution (a) Statement 1 Statement 2 Statement 3 Correct Incorrect Incorrect Moplah Uprisings (1836-1854; Malabar) – Against rise in revenue demand and reduction of field size, coupled with the oppression of officials. It resulted in widespread peasant unrest among the Moplahs of Malabar. Twenty-two rebellions took place between 1836 and 1854. None, however, proved successful. (The second Moplah uprising occurred after the Moplahs came to be organized by the Congress and the Khilafat supporters during the Non-cooperation Movement. But Hindu-Muslim differences distanced the Congress and the Moplahs from each other. By 1921, the Moplahs had been subdued.) Deccan Peasants’ Uprising by the Maratha peasants (1875; Kardeh village and Poona in Maharashtra) – The Deccan peasants uprising was directed mainly against the corrupt practices of the Marwari and Gujarati moneylenders. Social boycott of money lenders by the peasant was later transformed into armed peasant revolt in the Poona and Ahamadnagar districts of Maharashtra. The peasants attacked the money lenders’s houses, shops, and burnt them down. Phadke’s Ramosi Uprising by Ramosi peasants led by Vasudev Balwant Phadke (1877-1887; Maharashtra) – Against the British failure to take up anti-famine measures. It was protested in the violent manner not in the peaceful manner. Question 20 of 35 20. Question Consider the following statements regarding leadership during the revolt of 1857: The revolt at Lucknow was led by the Begum of Awadh. At Kanpur the Revolt was led by Nana Sahib, the adopted son of Baji Rao II Kunwar Singh of Jagdishpur near Arrah, was the chief organizer of the Revolt in Bihar. How many of the above statements are correct?  a) Only one b) Only two c) All three d) None Correct Solution (c) Statement 1 Statement 2 Statement 3 Correct Correct Correct The revolt at Lucknow was led by the Begum of Avadh who had proclaimed her young son, Bitjis Kadr, as the Nawab of Avadh. Helped by the sepoys at Lucknow, and by the zamindars and peasants of Avadh, the Begum organised an all-out attack on the British. At Kanpur the Revolt was led by Nana Sahib, the adopted son of Baji Rao II, the last Peshwa. Nana Sahib expelled the English from Kanpur with the help of the sepoys and proclaimed himself the Peshwa. At the same time, he acknowledged Bahadur Shah as the Emperor of India and declared himself to be his Governor. The chief burden of fighting on behalf of Nana Sahib fell on the shoulders of Tantia Tope, one of his most loyal servants. Tantia Tope has won immortal fame by his patriotism, determined fighting, and skillful guerrilla operations. Azi- mullah was another loyal servant of Nana Sahib. He was an expert in political propaganda. Unfortunately, Nana Sahib tarnished his brave record by deceitfully killing the garrison at Kanpur after he had agreed to give them safe conduct. Kunwar Singh, a ruined and discontented Zamindar of Jagdishpur near Arrah, was the chief organiser of the Revolt in Bihar. Though nearly 80 years old, he was perhaps the most outstanding military leader and strategist of the Revolt, He fought the British in Bihar, and, later joining hands with Nana Sahib’s forces, he also campaigned in Avadh and Central India. Racing back home he defeated the British forces near Arrah. But this proved to be his last battle. He had sustained a fatal wound in the fighting. He died on 27 April 1858 in his ancestral house in the village of Jagdishpur. Incorrect Solution (c) Statement 1 Statement 2 Statement 3 Correct Correct Correct The revolt at Lucknow was led by the Begum of Avadh who had proclaimed her young son, Bitjis Kadr, as the Nawab of Avadh. Helped by the sepoys at Lucknow, and by the zamindars and peasants of Avadh, the Begum organised an all-out attack on the British. At Kanpur the Revolt was led by Nana Sahib, the adopted son of Baji Rao II, the last Peshwa. Nana Sahib expelled the English from Kanpur with the help of the sepoys and proclaimed himself the Peshwa. At the same time, he acknowledged Bahadur Shah as the Emperor of India and declared himself to be his Governor. The chief burden of fighting on behalf of Nana Sahib fell on the shoulders of Tantia Tope, one of his most loyal servants. Tantia Tope has won immortal fame by his patriotism, determined fighting, and skillful guerrilla operations. Azi- mullah was another loyal servant of Nana Sahib. He was an expert in political propaganda. Unfortunately, Nana Sahib tarnished his brave record by deceitfully killing the garrison at Kanpur after he had agreed to give them safe conduct. Kunwar Singh, a ruined and discontented Zamindar of Jagdishpur near Arrah, was the chief organiser of the Revolt in Bihar. Though nearly 80 years old, he was perhaps the most outstanding military leader and strategist of the Revolt, He fought the British in Bihar, and, later joining hands with Nana Sahib’s forces, he also campaigned in Avadh and Central India. Racing back home he defeated the British forces near Arrah. But this proved to be his last battle. He had sustained a fatal wound in the fighting. He died on 27 April 1858 in his ancestral house in the village of Jagdishpur. Question 21 of 35 21. Question Consider the following statements: Global Gibbon Network is established by the United Nations Environment Programme. Hoolock Gibbon is the only ape found in India. Both Western Hoolock Gibbon and Eastern Hoolock Gibbon are protected under Schedule I of the Wildlife Protection Act of 1972. How many of the above statements are correct? a) Only one b) Only two c) All three d) None Correct Solution (b) Global Gibbon Network is established by the Hainan Institute of National Parks and Eco Foundation Global. It aims to protect and preserve Asia’s unique heritage, specifically the singing Gibbon and their habitats, by promoting participatory conservation policies, legislation, and actions. Hence statement 1 is incorrect. Hoolock Gibbon is the only ape found in India and is the state animal of Arunachal Pradesh. They are found in tropical and subtropical forests in the southeastern part of Asia. Hence statement 2 is correct. Both Western Hoolock Gibbon and Eastern Hoolock Gibbon are protected under Schedule I of the Wildlife Protection Act of 1972. Hence statement 3 is correct. Western Hoolock Gibbons are found in all the states of the northeast, restricted between the south of the Brahmaputra River and east of the Dibang River. And outside India, it is found in eastern Bangladesh and north-west Myanmar. It is listed as endangered on the IUCN Red List. Eastern Hoolock Gibbons are found in specific pockets of Arunachal Pradesh and Assam in India and southern China and northeast Myanmar outside India. It is listed as vulnerable on the IUCN Red List. Note: International Gibbon Day is celebrated on October 24 every year. It originated in 2015 when the IUCN Primate Specialist Group established the day as a commemoration to draw public attention to gibbon protection. Incorrect Solution (b) Global Gibbon Network is established by the Hainan Institute of National Parks and Eco Foundation Global. It aims to protect and preserve Asia’s unique heritage, specifically the singing Gibbon and their habitats, by promoting participatory conservation policies, legislation, and actions. Hence statement 1 is incorrect. Hoolock Gibbon is the only ape found in India and is the state animal of Arunachal Pradesh. They are found in tropical and subtropical forests in the southeastern part of Asia. Hence statement 2 is correct. Both Western Hoolock Gibbon and Eastern Hoolock Gibbon are protected under Schedule I of the Wildlife Protection Act of 1972. Hence statement 3 is correct. Western Hoolock Gibbons are found in all the states of the northeast, restricted between the south of the Brahmaputra River and east of the Dibang River. And outside India, it is found in eastern Bangladesh and north-west Myanmar. It is listed as endangered on the IUCN Red List. Eastern Hoolock Gibbons are found in specific pockets of Arunachal Pradesh and Assam in India and southern China and northeast Myanmar outside India. It is listed as vulnerable on the IUCN Red List. Note: International Gibbon Day is celebrated on October 24 every year. It originated in 2015 when the IUCN Primate Specialist Group established the day as a commemoration to draw public attention to gibbon protection. Question 22 of 35 22. Question The strategic roadmap “Horizon 2045” is between which of the following countries? a) United States and Germany b) Australia and Japan c) India and France d) China and Sri Lanka Correct Solution (c) The strategic roadmap “Horizon 2045” is between India and France. It includes cooperation in defence, space, nuclear energy, climate change, and green transitions as well as education and people-to-people ties for the next 25 years. Hence option c is correct. Incorrect Solution (c) The strategic roadmap “Horizon 2045” is between India and France. It includes cooperation in defence, space, nuclear energy, climate change, and green transitions as well as education and people-to-people ties for the next 25 years. Hence option c is correct. Question 23 of 35 23. Question Consider the following statements about PM Mega Integrated Textile Regions and Apparel (PM MITRA) Parks: It will help in creating world-class industrial infrastructure by encouraging investment, innovation, and job creation in the textile sector. It will be implemented by a special purpose vehicle (SPV) completely owned by the centre through the Ministry of Textiles. Choose the correct codes: a) 1 only b) 2 only c) Both 1 and 2 d) Neither 1 nor 2 Correct Solution (a) PM Mega Integrated Textile Regions and Apparel (PM MITRA) Parks will help in creating world-class industrial infrastructure by encouraging investment, innovation, and job creation in the textile sector. Hence statement 1 is correct. They are a major step forward in realising the Government’s vision of making India a global hub for textile manufacturing and exports. They will enhance the competitiveness of the textiles industry by helping it achieve economies of scale as well as attract global players to manufacture in India. It will be implemented by a special purpose vehicle (SPV) owned by the centre and state through the Ministry of Textiles. Hence statement 2 is incorrect. The Ministry of Textiles will provide financial support in the form of Development Capital Support up to Rs. 500 crore per park to the Park SPV. A Competitive Incentive Support (CIS) of up to Rs 300 crore per park to the units in PM MITRA Park shall also be provided to incentivise speedy implementation. State governments will provide contiguous and encumbrance-free land, provision of all utilities, reliable power supply, water availability, wastewater disposal system, an effective single window clearance as well as a conducive and stable industrial/textile policy. Incorrect Solution (a) PM Mega Integrated Textile Regions and Apparel (PM MITRA) Parks will help in creating world-class industrial infrastructure by encouraging investment, innovation, and job creation in the textile sector. Hence statement 1 is correct. They are a major step forward in realising the Government’s vision of making India a global hub for textile manufacturing and exports. They will enhance the competitiveness of the textiles industry by helping it achieve economies of scale as well as attract global players to manufacture in India. It will be implemented by a special purpose vehicle (SPV) owned by the centre and state through the Ministry of Textiles. Hence statement 2 is incorrect. The Ministry of Textiles will provide financial support in the form of Development Capital Support up to Rs. 500 crore per park to the Park SPV. A Competitive Incentive Support (CIS) of up to Rs 300 crore per park to the units in PM MITRA Park shall also be provided to incentivise speedy implementation. State governments will provide contiguous and encumbrance-free land, provision of all utilities, reliable power supply, water availability, wastewater disposal system, an effective single window clearance as well as a conducive and stable industrial/textile policy. Question 24 of 35 24. Question Consider the following statements: Statement I: Urbanization and concrete structures in cities cause heat waves. Statement II: Because concrete and asphalt in cities absorb and retain heat leading to increased temperatures contributing to heat waves. Which one of the following is correct in respect of the above statements? a) Both Statement-I and Statement-II are correct and Statement-II is the correct explanation for Statement-I b) Both Statement-I and Statement-II are correct and Statement-II is not the correct explanation' for Statement-I c) Statement I is correct but Statement II is incorrect d) Statement I is incorrect Statement II is correct Correct Solution (a) A heat wave is defined as a period of abnormally high temperatures over a region. In plains, if the temperature reaches at least 40⁰ Celcius, and in hilly regions if the temperature reaches at least 30⁰ Celcius then it is considered a heat wave. Urbanization and concrete structures in cities cause heat waves. Hence statement 1 is correct. It is because concrete and asphalt in cities absorb and retain heat leading to increased temperatures contributing to heat waves. Hence statement 2 is correct and correct explanation for statement 1. An urban heat island is a temporary phenomenon where certain parts of the city experience a higher temperature than its surroundings. It is due to the heat trapped in the buildings of cities. Incorrect Solution (a) A heat wave is defined as a period of abnormally high temperatures over a region. In plains, if the temperature reaches at least 40⁰ Celcius, and in hilly regions if the temperature reaches at least 30⁰ Celcius then it is considered a heat wave. Urbanization and concrete structures in cities cause heat waves. Hence statement 1 is correct. It is because concrete and asphalt in cities absorb and retain heat leading to increased temperatures contributing to heat waves. Hence statement 2 is correct and correct explanation for statement 1. An urban heat island is a temporary phenomenon where certain parts of the city experience a higher temperature than its surroundings. It is due to the heat trapped in the buildings of cities. Question 25 of 35 25. Question Consider the following statements about the Comprehensive and Progressive Agreement for Trans-Pacific Partnership (CPTPP): It is a free trade agreement that succeeded the Trans-Pacific Partnership (TPP). All members of the Asia-Pacific Economic Cooperation (APEC) are members of the CPTPP. The United Kingdom is the first European country and the first new member of the bloc since its inception. How many of the above statements are correct? a) Only one b) Only two c) All three d) None Correct Solution (b) The UK has signed a deal to join a trade pact with several countries in Asia and the Pacific, including Japan and Australia. The name of this pact – CPTPP. The Comprehensive and Progressive Agreement for Trans-Pacific Partnership (CPTPP) is a free trade agreement that succeeded the Trans-Pacific Partnership (TPP). TPP was a trade agreement formed in 2005 that consists of 12 member countries. Later the United States withdrew in 2017 and the grouping came to be called CPTPP. Hence statement 1 is correct. All members of the CPTPP are members of the Asia-Pacific Economic Cooperation (APEC). Members of the CPTPP are Australia, Brunei, Canada, Chile, Japan, Malaysia, Mexico, New Zealand, Peru, Singapore, and Vietnam. Members of APEC are Australia, Brunei Darussalam, Canada, Chile, the People’s Republic of China, Hong Kong, Indonesia, Japan, the Republic of Korea, Malaysia, Mexico, New Zealand, Papua New Guinea, Peru, the Philippines, the Russian Federation, Singapore, Chinese Taipei, Thailand, the United States of America, and  Vietnam. Hence statement 2 is incorrect. The United Kingdom is the first European country and the first new member of the bloc since its inception. Hence statement 3 is correct. Incorrect Solution (b) The UK has signed a deal to join a trade pact with several countries in Asia and the Pacific, including Japan and Australia. The name of this pact – CPTPP. The Comprehensive and Progressive Agreement for Trans-Pacific Partnership (CPTPP) is a free trade agreement that succeeded the Trans-Pacific Partnership (TPP). TPP was a trade agreement formed in 2005 that consists of 12 member countries. Later the United States withdrew in 2017 and the grouping came to be called CPTPP. Hence statement 1 is correct. All members of the CPTPP are members of the Asia-Pacific Economic Cooperation (APEC). Members of the CPTPP are Australia, Brunei, Canada, Chile, Japan, Malaysia, Mexico, New Zealand, Peru, Singapore, and Vietnam. Members of APEC are Australia, Brunei Darussalam, Canada, Chile, the People’s Republic of China, Hong Kong, Indonesia, Japan, the Republic of Korea, Malaysia, Mexico, New Zealand, Papua New Guinea, Peru, the Philippines, the Russian Federation, Singapore, Chinese Taipei, Thailand, the United States of America, and  Vietnam. Hence statement 2 is incorrect. The United Kingdom is the first European country and the first new member of the bloc since its inception. Hence statement 3 is correct. Question 26 of 35 26. Question Consider the following statements regarding the National Multidimensional Poverty Index: A Progress Review 2023 by NITI Aayog: Bihar topped the list with the highest percentage of people who are living in multidimensional poverty. Uttar Pradesh registered the largest decline in number of poor people escaping multidimensional poverty. The rural areas witnessed the fastest decline in poverty than urban areas. How many of the above statements are correct? a) Only one b) Only two c) All three d) None Correct Solution (c) Bihar, Jharkhand, Meghalaya, Uttar Pradesh, and Madhya Pradesh topped the list with the highest percentage of people who are living in multidimensional poverty. Hence statement 1 is correct. Uttar Pradesh registered the largest decline in number of poor people escaping multidimensional poverty. Hence statement 2 is correct. The rural areas witnessed the fastest decline in multidimensional poverty than urban areas. In rural areas, it saw a decline from 32.59% to 19.28% whereas in urban areas, it saw a decline from 8.65% to 5.27%. Hence statement 3 is correct. Incorrect Solution (c) Bihar, Jharkhand, Meghalaya, Uttar Pradesh, and Madhya Pradesh topped the list with the highest percentage of people who are living in multidimensional poverty. Hence statement 1 is correct. Uttar Pradesh registered the largest decline in number of poor people escaping multidimensional poverty. Hence statement 2 is correct. The rural areas witnessed the fastest decline in multidimensional poverty than urban areas. In rural areas, it saw a decline from 32.59% to 19.28% whereas in urban areas, it saw a decline from 8.65% to 5.27%. Hence statement 3 is correct. Question 27 of 35 27. Question Consider the following statements regarding Pre-arrest Bail: It is granted under section 438 of the Code of Criminal Procedure of  1973. It is issued only by the Supreme Court and High Court. The person can seek pre-arrest bail only for non-bailable offences. How many of the above statements are correct? a) Only one b) Only two c) All three d) None Correct Solution (b) Accused person to apply for bail before being arrested. It is granted under section 438 of the Code of Criminal Procedure of 1973. Hence statement 1 is correct. It is issued only by the Sessions Court and High Court. Hence statement 2 is incorrect. The provision of pre-arrest bail is discretionary, and the court may grant bail after considering the nature and gravity of the offence, the antecedents of the accused, and other relevant factors. The person can seek pre-arrest bail only for non-bailable offences. Hence statement 3 is correct. The court may also impose a monetary bond, which the person seeking anticipatory bail will have to pay if they fail to appear before the court or violate the conditions imposed. Incorrect Solution (b) Accused person to apply for bail before being arrested. It is granted under section 438 of the Code of Criminal Procedure of 1973. Hence statement 1 is correct. It is issued only by the Sessions Court and High Court. Hence statement 2 is incorrect. The provision of pre-arrest bail is discretionary, and the court may grant bail after considering the nature and gravity of the offence, the antecedents of the accused, and other relevant factors. The person can seek pre-arrest bail only for non-bailable offences. Hence statement 3 is correct. The court may also impose a monetary bond, which the person seeking anticipatory bail will have to pay if they fail to appear before the court or violate the conditions imposed. Question 28 of 35 28. Question Consider the following statements about the Kuki tribe: They are present in all Northeast Indian states except Arunachal Pradesh. Lal is known as their village chief and has their customary laws. They resisted British hegemony before India’s independence. How many of the above statements are correct? a) Only one b) Only two c) Only three d) None Correct Solution (c) Kuki tribes are ethnic groups present in all Northeast Indian states except Arunachal Pradesh, Myanmar, and Bangladesh. Hence statement 1 is correct. Lal is known as their village chief and has their own customary laws. Hence statement 2 is correct. They do not marry outside their community and a majority of them follow Christianity as their religion. They resisted British hegemony and started a rebellion before India’s independence. Kuki Rebellion took place during 1917 and they were defeated by the British in 1919. Hence statement 3 is correct. Incorrect Solution (c) Kuki tribes are ethnic groups present in all Northeast Indian states except Arunachal Pradesh, Myanmar, and Bangladesh. Hence statement 1 is correct. Lal is known as their village chief and has their own customary laws. Hence statement 2 is correct. They do not marry outside their community and a majority of them follow Christianity as their religion. They resisted British hegemony and started a rebellion before India’s independence. Kuki Rebellion took place during 1917 and they were defeated by the British in 1919. Hence statement 3 is correct. Question 29 of 35 29. Question Consider the following statements about Black Sea Grain Initiative: It was set up to resume vital food and fertilizer exports from Ukraine to the rest of the world. It was brokered between Russia and Ukraine by the Food and Agriculture Organization and India. It is implemented by the Joint Coordination Centre which is hosted in Istanbul. How many of the above statements are correct? a) Only one b) Only two c) Only three d) None Correct Solution (b) Black Sea Grain Initiative was set up to resume vital food and fertilizer exports from Ukraine to the rest of the world. Hence statement 1 is correct. It allowed exports of grain, other foodstuffs, and fertilizer, including ammonia, to resume through a safe maritime humanitarian corridorfrom three key Ukrainian ports – Chornomorsk, Odesa, and Yuzhny/Pivdennyi, to the rest of the world. It was brokered between Russia and Ukraine by the United Nations and Turkey. Hence statement 2 is incorrect. It is implemented by the Joint Coordination Centre (JCC) which is hosted in Istanbul. Hence statement 3 is correct. JCC includes representatives from Russia, Türkiye, Ukraine, and the United Nations Incorrect Solution (b) Black Sea Grain Initiative was set up to resume vital food and fertilizer exports from Ukraine to the rest of the world. Hence statement 1 is correct. It allowed exports of grain, other foodstuffs, and fertilizer, including ammonia, to resume through a safe maritime humanitarian corridorfrom three key Ukrainian ports – Chornomorsk, Odesa, and Yuzhny/Pivdennyi, to the rest of the world. It was brokered between Russia and Ukraine by the United Nations and Turkey. Hence statement 2 is incorrect. It is implemented by the Joint Coordination Centre (JCC) which is hosted in Istanbul. Hence statement 3 is correct. JCC includes representatives from Russia, Türkiye, Ukraine, and the United Nations Question 30 of 35 30. Question Which of the following disease is treated using the drug, ‘Leqembi’? a) Dementias b) Parkinson’s disease c) Epilepsy d) Alzheimer’s disease Correct Solution (d) Context: Donanemab, if approved will become the second drug used to treat Alzheimer’s disease. Leqembi is a drug approved for Alzheimer’s disease that targets the fundamental pathophysiology of the disease. Hence option d is correct. Alzheimer’s disease is an irreversible, progressive brain disorder that slowly destroys memory and thinking skills and, eventually, the ability to carry out simple tasks. Incorrect Solution (d) Context: Donanemab, if approved will become the second drug used to treat Alzheimer’s disease. Leqembi is a drug approved for Alzheimer’s disease that targets the fundamental pathophysiology of the disease. Hence option d is correct. Alzheimer’s disease is an irreversible, progressive brain disorder that slowly destroys memory and thinking skills and, eventually, the ability to carry out simple tasks. Question 31 of 35 31. Question What is X in the sequence – 9, 324, 25, 256, 49, 196, 81, X? a) 121 b) 136 c) 144 d) 169 Correct Solution (c) 9 -> 32 25 -> 52 49 -> 72 81 -> 92   Similarly, 324 -> 182 256 -> 162 196 -> 142 144 -> 122. Incorrect Solution (c) 9 -> 32 25 -> 52 49 -> 72 81 -> 92   Similarly, 324 -> 182 256 -> 162 196 -> 142 144 -> 122. Question 32 of 35 32. Question If A and B are digits and A 3 + 8 B = 150, then the value of A – B is a) 1 b) 2 c) -2 d) -1 Correct Solution (d) We have A 3 + 8 B = 150 Here, 3 + B = 0 , so 3 + B is a two digit number whose unit number is 0 3 + B = 10 => B = 7   Now considering ten’s column, A + 8 + 1 = 15 A + 9 = 15 A = 6 Therefore, A – B = 6 – 7 = -1 Incorrect Solution (d) We have A 3 + 8 B = 150 Here, 3 + B = 0 , so 3 + B is a two digit number whose unit number is 0 3 + B = 10 => B = 7   Now considering ten’s column, A + 8 + 1 = 15 A + 9 = 15 A = 6 Therefore, A – B = 6 – 7 = -1 Question 33 of 35 33. Question Which of the following statements are correct? The average of first twenty multiples of 8 is 84 There are 25 prime numbers between 100 to 200 Choose the correct answer using the code given below a) 1 only b) 2 only c) Both 1 and 2 d) Neither 1 nor 2 Correct Solution (a) 1) The average of first twenty multiples of 8 is 84.   First twenty mutiples of 8 are: 8 * 1, 8*2,8*3,…… so on… So, AP = 8 + 16 + 24 + …… + 160 Required average = 8( 1 + 2 + 3 + …… 10)/20 AP = Sum of first n numbers = n/2[first term + last term] Required average of AP = [ 8 + 160 ]/2 Hence, Statement 1 is correct 2) There are 21 prime numbers between 100 and 200 101, 103, 107, 109, 113, 127, 131, 137, 139,149,151,157, 163, 167, 173, 179, 181, 191, 193, 197, 199 Hence, statement 2 is incorrect Incorrect Solution (a) 1) The average of first twenty multiples of 8 is 84.   First twenty mutiples of 8 are: 8 * 1, 8*2,8*3,…… so on… So, AP = 8 + 16 + 24 + …… + 160 Required average = 8( 1 + 2 + 3 + …… 10)/20 AP = Sum of first n numbers = n/2[first term + last term] Required average of AP = [ 8 + 160 ]/2 Hence, Statement 1 is correct 2) There are 21 prime numbers between 100 and 200 101, 103, 107, 109, 113, 127, 131, 137, 139,149,151,157, 163, 167, 173, 179, 181, 191, 193, 197, 199 Hence, statement 2 is incorrect Question 34 of 35 34. Question In a group of 6 boys and 4 girls, four children are to be selected. In how many different ways can they be selected such that at least one girl should be there? a) 150 b) 195 c) 249 d) 250 Correct Solution (b) In a group of 6 boys and 4 girls, four children are to be selected such that at least one boy should be there. Hence we have 4 options as given below We can select 4 girls (option 1) Number of ways to this = 4C4  We can select 3 girls and 1 boy (option 2) Number of ways to this = 4C3 × 6C1 We can select 2 girls and 2 boys (option 3) Number of ways to this = 4C2 × 6C2 We can select 1 girl and 3 boys (option 4) Number of ways to this = 4C1 × 6C3   Total number of ways = 4C4 + 4C3 × 6C1 + 4C2 × 6C2 + 4C1 × 6C3 = 1+ ( 4 ×  6) + ( (4 ×  3)/(2 ×  1) ×  ( (6 ×  5)/( 2 ×  1)) + ( 4 ×  ( 6 ×  5 ×  4)/( 3 ×  2 × 1)) = 1 + 24 + ( 6 × 15 ) + ( 4 ×  20 ) = 1 + 24 + 90 + 80 = 195   Incorrect Solution (b) In a group of 6 boys and 4 girls, four children are to be selected such that at least one boy should be there. Hence we have 4 options as given below We can select 4 girls (option 1) Number of ways to this = 4C4  We can select 3 girls and 1 boy (option 2) Number of ways to this = 4C3 × 6C1 We can select 2 girls and 2 boys (option 3) Number of ways to this = 4C2 × 6C2 We can select 1 girl and 3 boys (option 4) Number of ways to this = 4C1 × 6C3   Total number of ways = 4C4 + 4C3 × 6C1 + 4C2 × 6C2 + 4C1 × 6C3 = 1+ ( 4 ×  6) + ( (4 ×  3)/(2 ×  1) ×  ( (6 ×  5)/( 2 ×  1)) + ( 4 ×  ( 6 ×  5 ×  4)/( 3 ×  2 × 1)) = 1 + 24 + ( 6 × 15 ) + ( 4 ×  20 ) = 1 + 24 + 90 + 80 = 195   Question 35 of 35 35. Question How many numbers are there between 90 and 1000 such that the digit 7 occupies the units place? a) 105 b) 81 c) 65 d) 91 Correct Solution (d)  Between 100 and 200 there are 10 such numbers. They are 107, 117, 127, 137, 147, 157, 167, 177, 187, 197. Like that between 200 and 300 -> 10 300 to 400 ->10 400 to 500 ->10 500 to 600 ->10 600 to 700 ->10 700 to 800 ->10 800 to 900 ->10 900 to 1000 ->10 ∴ from 100 to 1000, 90 such numbers are there Between 90 and 100 97 is there So total such numbers is 90 + 1 = 91   Incorrect Solution (d)  Between 100 and 200 there are 10 such numbers. They are 107, 117, 127, 137, 147, 157, 167, 177, 187, 197. Like that between 200 and 300 -> 10 300 to 400 ->10 400 to 500 ->10 500 to 600 ->10 600 to 700 ->10 700 to 800 ->10 800 to 900 ->10 900 to 1000 ->10 ∴ from 100 to 1000, 90 such numbers are there Between 90 and 100 97 is there So total such numbers is 90 + 1 = 91   window.wpProQuizInitList = window.wpProQuizInitList || []; window.wpProQuizInitList.push({ id: '#wpProQuiz_3508', init: { quizId: 3508, mode: 1, globalPoints: 70, timelimit: 1800, resultsGrade: [0], bo: 704, qpp: 0, catPoints: [70], formPos: 0, lbn: "Test-summary", json: {"29639":{"type":"single","id":29639,"catId":0,"points":2,"correct":[1,0,0,0]},"29641":{"type":"single","id":29641,"catId":0,"points":2,"correct":[1,0,0,0]},"29642":{"type":"single","id":29642,"catId":0,"points":2,"correct":[1,0,0,0]},"29645":{"type":"single","id":29645,"catId":0,"points":2,"correct":[0,0,1,0]},"29646":{"type":"single","id":29646,"catId":0,"points":2,"correct":[0,0,1,0]},"29647":{"type":"single","id":29647,"catId":0,"points":2,"correct":[1,0,0,0]},"29649":{"type":"single","id":29649,"catId":0,"points":2,"correct":[0,0,1,0]},"29652":{"type":"single","id":29652,"catId":0,"points":2,"correct":[0,0,1,0]},"29654":{"type":"single","id":29654,"catId":0,"points":2,"correct":[0,0,1,0]},"29656":{"type":"single","id":29656,"catId":0,"points":2,"correct":[0,1,0,0]},"29659":{"type":"single","id":29659,"catId":0,"points":2,"correct":[0,1,0,0]},"29660":{"type":"single","id":29660,"catId":0,"points":2,"correct":[0,0,1,0]},"29662":{"type":"single","id":29662,"catId":0,"points":2,"correct":[0,0,1,0]},"29663":{"type":"single","id":29663,"catId":0,"points":2,"correct":[0,1,0,0]},"29665":{"type":"single","id":29665,"catId":0,"points":2,"correct":[0,0,1,0]},"29668":{"type":"single","id":29668,"catId":0,"points":2,"correct":[0,1,0,0]},"29669":{"type":"single","id":29669,"catId":0,"points":2,"correct":[0,0,1,0]},"29672":{"type":"single","id":29672,"catId":0,"points":2,"correct":[0,1,0,0]},"29674":{"type":"single","id":29674,"catId":0,"points":2,"correct":[1,0,0,0]},"29676":{"type":"single","id":29676,"catId":0,"points":2,"correct":[0,0,1,0]},"29677":{"type":"single","id":29677,"catId":0,"points":2,"correct":[0,1,0,0]},"29678":{"type":"single","id":29678,"catId":0,"points":2,"correct":[0,0,1,0]},"29679":{"type":"single","id":29679,"catId":0,"points":2,"correct":[1,0,0,0]},"29680":{"type":"single","id":29680,"catId":0,"points":2,"correct":[1,0,0,0]},"29683":{"type":"single","id":29683,"catId":0,"points":2,"correct":[0,1,0,0]},"29686":{"type":"single","id":29686,"catId":0,"points":2,"correct":[0,0,1,0]},"29687":{"type":"single","id":29687,"catId":0,"points":2,"correct":[0,1,0,0]},"29688":{"type":"single","id":29688,"catId":0,"points":2,"correct":[0,0,1,0]},"29689":{"type":"single","id":29689,"catId":0,"points":2,"correct":[0,1,0,0]},"29692":{"type":"single","id":29692,"catId":0,"points":2,"correct":[0,0,0,1]},"29695":{"type":"single","id":29695,"catId":0,"points":2,"correct":[0,0,1,0]},"29698":{"type":"single","id":29698,"catId":0,"points":2,"correct":[0,0,0,1]},"29701":{"type":"single","id":29701,"catId":0,"points":2,"correct":[1,0,0,0]},"29702":{"type":"single","id":29702,"catId":0,"points":2,"correct":[0,1,0,0]},"29704":{"type":"single","id":29704,"catId":0,"points":2,"correct":[0,0,0,1]}} } }); All the Best IASbaba

DAILY CURRENT AFFAIRS IAS | UPSC Prelims and Mains Exam – 6th March 2024

Archives (PRELIMS & MAINS Focus)   Black Sea Syllabus Prelims – Geography Context: Ukraine said on Tuesday that its forces destroyed a Russian military patrol ship in the Black Sea near annexed Crimea, the latest naval attack on Moscow’s fleet in the key waterway. Background:- The Black Sea has been a vital battleground in the two-year war between Russia and Ukraine About Black Sea The Black Sea is a marginal mediterranean sea lying between Europe and Asia, east of the Balkans, south of the East European Plain, west of the Caucasus, and north of Anatolia. It is bounded by Bulgaria, Georgia, Romania, Russia, Turkey, and Ukraine. The Black Sea is supplied by major rivers, principally the Danube, Dnieper and Dniester. The Bosporus strait connects it to the small Sea of Marmara which in turn is connected to the Aegean Sea via the strait of the Dardanelles. To the north, the Black Sea is connected to the Sea of Azov by the Kerch Strait. While the net flow of water through the Bosporus and Dardanelles (known collectively as the Turkish Straits) is out of the Black Sea, water generally flows in both directions simultaneously: Denser, more saline water from the Aegean flows into the Black Sea underneath the less dense, fresher water that flows out of the Black Sea. This creates a significant and permanent layer of deep water that does not drain or mix and is therefore anoxic. This anoxic layer is responsible for the preservation of ancient shipwrecks which have been found in the Black Sea. Source: NDTV Article 371(A- J) Syllabus Prelims & Mains – Polity Context: In the backdrop of protests across Ladakh, the Centre is mulling granting Article 371-like protection to the Union Territory. Background: Ladakh witnessed massive protests demanding statehood, legislature and inclusion in the Sixth Schedule, apart from concerns about environmental degradation due to possible industrialisation. About Article 371 (A-J): Article 371 of the Constitution includes special provisions for 11 states, including six of the Northeast. Articles 369 through 392 (including some that have been removed) appear in Part XXI of the Constitution, titled ‘Temporary, Transitional and Special Provisions’. Article 370 dealt with Temporary Provisions with respect to the State of Jammu and Kashmir.It was repealed in 2019; Articles 371, 371A, 371B, 371C, 371D, 371E, 371F, 371G, 371H, and 371J define special provisions with regard to another state (or states).Articles 370 and 371 were part of the Constitution at the time of its commencement on January 26, 1950; Articles 371A through 371J were incorporated subsequently. Article 371 has provisions for Maharashtra and Gujarat. Article 371A (13th Amendment Act, 1962), Nagaland: This provision was inserted after a 16-point agreement between the Centre and the Naga People’s Convention in 1960, which led to the creation of Nagaland in 1963. Parliament cannot legislate in matters of Naga religion or social practices, Naga customary law and procedure, administration of civil and criminal justice involving decisions according to Naga customary law, and ownership and transfer of land without concurrence of the state Assembly. Article 371B (22nd Amendment Act, 1969) has provisions for Assam; Article 371C (27th Amendment Act, 1971) has provisions for Manipur. Article 371D & E – has provision for Andhra Pradesh. Article 371F (36th Amendment Act, 1975) has provision for Sikkim; Article 371G (53rd Amendment Act, 1986) has provision for Mizoram, Article 371H (55th Amendment Act, 1986) has provision for Arunachal Pradesh. Article 371I deals with Goa, but it does not include any provision that can be deemed ‘special’. Article 371J (98th Amendment Act, 2012) has provision for Karnataka. Examples/details of provisions for some northeast states: Article 371G (53rd Amendment Act, 1986), Mizoram: Parliament cannot make laws on “religious or social practices of the Mizos, Mizo customary law and procedure, administration of civil and criminal justice involving decisions according to Mizo customary law, ownership and transfer of land unless the Assembly so decides”. Article 371A (13th Amendment Act, 1962), Nagaland : Parliament cannot legislate in matters of Naga religion or social practices, Naga customary law and procedure, administration of civil and criminal justice involving decisions according to Naga customary law, and ownership and transfer of land without concurrence of the state Assembly. Source: Indian Express CONSUMER PROTECTION ACT, 2019 Syllabus Prelims – Polity Context: Recently, the Supreme Court of India issued the contempt notice and passed an interim order imposing a ban on advertisements of Patanjali Ayurved’s medicines. Background: According to Section 2(28) of the Consumer Protection Act, 2019, an advertisement is deemed misleading when it gives a false guarantee or is likely to mislead consumers regarding the nature, substance, quantity, or quality of the product or service. About The Consumer Protection Act : The Consumer Protection Act, 2019 is a comprehensive legislation enacted in India to safeguard the rights and interests of consumers. Key provisions of the Act: Definition of Consumer: The Act defines a consumer as any person who: Buys goods or avails services for personal use. Uses goods with the consent of the buyer. Hires or avails services for consideration. It also covers legal heirs and representatives of consumers. Rights of Consumers Right to Safety: Protection from hazardous goods and services. Right to Information: Access accurate and transparent information about products and services. Right to Choose: Choose from a variety of goods and services. Right to Redressal: Seek redressal for unfair trade practices and defective products. Right to Consumer Education: Be informed about their rights and responsibilities. Unfair Trade Practices The Act prohibits various unfair practices, such as: False representation: Misleading advertisements or false claims. Unfair contracts: Imposing unfair terms on consumers. Defective goods: Selling goods with defects. Restrictive trade practices: Monopolistic behaviour. Consumer Disputes Redressal The Act establishes Consumer Disputes Redressal Commissions at the district, state, and national levels. Consumers can file complaints related to unfair trade practices, defective goods, or deficient services. Source: Livemint Previous Year Question Q1. With reference to consumer’s rights/privileges under the provisions of the law in India, which of the following statements is/are correct? Consumers are empowered to take samples for food testing. When a consumer files a complaint in any consumer forum, no fee is required to be paid. In case of the death of a consumer, his/her legal heir can file a complaint in the consumer forum on his/her behalf. Select the correct answer using the codes given below: 1 only 2 and 3 only 1 and 3 only 1, 2 and 3 GEOGRAPHICAL INDICATION (GI) TAG Syllabus Prelims – Current Event Context: Risa, a traditional tribal attire from Tripura, has recently received the Geographical Indication (GI) tag. Background: Risa is a handwoven cloth used primarily as a female upper garment. It also serves as headgear, a stole, or a present to express respect. This exquisite fabric is woven in colourful designs and holds significant social and religious importance. About Geographical Indication (GI) tag A Geographical Indication (GI) tag is a distinctive sign used on products that have a specific geographical origin and possess qualities or a reputation that are essentially due to that origin. It serves as a form of intellectual property protection, safeguarding the unique identity and value of these products. Significance of GI Tags: Authenticity: GI tags ensure that registered authorized users are allowed to use the popular product name. Preserving Tradition: They protect traditional methods of production and recognize the product’s historical roots. Promoting Local Artisans: GI tags benefit local artisans and promote cultural heritage. Governance of GI Tags in India: The Geographical Indications of Goods (Registration and Protection) Act, 1999 governs GI tags in India. As a member of the World Trade Organization (WTO), India implemented this Act to fulfil its obligations under the Agreement on Trade-Related Aspects of Intellectual Property Rights (TRIPS). The Geographical Indication Registry under the Department of Industry Promotion and Internal Trade, Ministry of Commerce and Industry grants these tags. Darjeeling tea holds the distinction of being the first product in India to receive the Geographical Indication (GI) tag. Source: Business Standard Previous Year Question India enacted The Geographical Indications of Goods (Registration and Protection) Act, 1999 in order to comply with the obligations to ILO IMF UNCTAD WTO CARBON CAPTURE AND STORAGE (CCS) Syllabus Prelims – Environment Context: In a significant shift from its previous stance, Germany, a major contributor to greenhouse gas emissions, has decided to permit industries to capture their carbon emissions and store them underground at offshore sites. Background: Germany aims to become carbon neutral by 2045, but it faces challenges in curbing emissions from sectors like cement production. Carbon capture and storage is now seen as a temporary solution until more sustainable innovations emerge. About Carbon Capture and Storage (CCS): Carbon Capture and Storage (CCS) is a crucial approach in the fight against climate change. Carbon capture and storage (CCS) involves capturing carbon dioxide (CO2) emissions from industrial processes and power plants. Unlike carbon dioxide removal (CDR), which removes CO2 from the atmosphere, CCS prevents CO2 from escaping into the air in the first place. The goal of CCS is to prevent a significant amount of CO2 from entering the atmosphere and contributing to global warming and climate change. Benefits Reducing Greenhouse Gas Emissions: CCS captures carbon dioxide (CO2) emissions from industrial processes and power plants, preventing them from entering the atmosphere. By storing CO2 underground, CCS helps reduce the overall concentration of greenhouse gases. Preserving Fossil Fuel Use: CCS allows us to continue using fossil fuels while minimizing their environmental impact. It provides a bridge toward cleaner energy sources by reducing emissions from existing fossil fuel infrastructure. Industrial Applications: CCS can be applied to sectors like cement production, steel manufacturing, and chemical industries. These sectors often emit substantial CO2, and CCS offers a way to address their emissions. Creating Carbon Sinks: Underground storage sites act as carbon sinks, permanently storing CO2 away from the atmosphere. Properly managed storage sites can sequester emissions for centuries. Transitioning to Clean Energy: CCS provides a transition strategy as we shift toward renewable energy sources. It buys time for renewable technologies to mature and become economically competitive. Source: Economic Times Delimitation Syllabus Prelims & Mains – Polity Context: The Delimitation of constituencies for the Lok Sabha and State Legislative Assemblies is to be carried out based on the First Census after 2026. Background: The 2021 Census was originally postponed due to the COVID-19 pandemic and subsequently due to delays on the part of the Central government. About Delimitation Delimitation means the process of fixing the number of seats and boundaries of territorial constituencies in each State for the Lok Sabha and Legislative assemblies. It also includes determining the seats to be reserved for Scheduled Castes (SC) and Scheduled Tribes (ST) in these houses. It is performed by the ‘Delimitation Commission’ that is set up under an act of Parliament. Delimitation Commissions have been set up four times — 1952, 1963, 1973, and 2002 under the Acts of 1952, 1962, 1972, and 2002. The first delimitation exercise was carried out by the President (with the help of the Election Commission) in 1950-51. Significance of Delimitation Delimitation ensures fair representation in the Lok Sabha and State Legislative Assemblies by adjusting the number of seats based on population changes which is crucial for upholding the democratic principle of one citizen-one vote-one value. By readjusting the boundaries of territorial constituencies, delimitation aims to ensure equitable distribution of seats among different regions, considering population shifts over time. This helps prevent underrepresentation or overrepresentation of specific areas. Delimitation determines the allocation of reserved seats for Scheduled Castes (SC) and Scheduled Tribes (ST) in accordance with constitutional provisions, ensuring adequate political representation for marginalized communities. Delimitation impacts federal principles by influencing the distribution of political power among states. It is essential to strike a balance between population-based representation and federal considerations to maintain harmony among diverse regions. Historically, the freezing of seats based on the 1971 Census aimed to incentivize population control measures. However, the impending delimitation exercise raises questions about the effectiveness and implications of this policy in the context of changing demographics. Issues/Concerns related to Delimitation: The disparity in representation between the northern and southern parts of India in the Lok Sabha due to population as a deciding factor. The delimitation based solely on population disregards the progress made by the southern states in population control and may lead to disparities in the federal structure. Despite having only 18% of the country’s population, the southern states contribute 35% to the country’s GDP. The northern states, which did not prioritize population control, are expected to benefit from the delimitation process due to their higher population growth. After the 15th Finance Commission used the 2011 Census as a basis for its recommendation, concerns were raised about southern states losing funding and representation in parliament. Previously, the 1971 Census was used as the base for funding and tax devolution recommendations to states. The scheduled delimitation and reallocation of seats may result in not only a loss of seats for southern states but also an increase in power for political parties with their base of support in the north. This could potentially lead to a shift of power toward the north and away from the south. The exercise will also affect the division of seats reserved for the Scheduled Castes and Scheduled Tribes in each state (under Articles 330 and 332). Source: The Hindu Practice MCQs Daily Practice MCQs Q1.) With reference to Carbon Capture and Storage (CCS), consider the following statements: CCS captures carbon dioxide (CO2) emissions from industrial processes and preventing them from entering the atmosphere. CCS allows to continue using fossil fuels while minimizing their environmental impact. CCS can be applied to sectors like cement production, steel manufacturing, and chemical industries. How many of the above given statements are not correct? Only one Only two All three None Q2.)Consider the following pairs: Products with GI Tag                   States Lanjia Saura paintings – Jammu and Kashmir Risa Textile –                          Tripura Amroha Dholak –             Uttar Pradesh How many of the pairs given above are correctly matched? Only one Only two All three None Q3.) Consider the following statements: Statement-I: The Consumer Protection Act, 2019 is a comprehensive legislation enacted in India to safeguard the rights and interests of consumers. Statement-II: The Act establishes Central Consumer Protection Authority to addresses improper trade practices, misleading advertisements, and violations of consumer rights. Which one of the following is correct in respect of the above statements? Both Statement-I and Statement-II are correct and Statement-II is the correct explanation for Statement-I Both Statement-I and Statement-II are correct and Statement-II is not the correct explanation for Statement-I Statement-I is correct but Statement-II is incorrect Statement-I is incorrect but Statement-II is correct Comment the answers to the above questions in the comment section below!! ANSWERS FOR ’  6th March  2024 – Daily Practice MCQs’ will be updated along with tomorrow’s Daily Current Affairs.st ANSWERS FOR  5th March – Daily Practice MCQs Answers- Daily Practice MCQs Q.1) – b Q.2) – c Q.3) – d

Daily Prelims CA Quiz

UPSC Quiz – 2024 : IASbaba’s Daily Current Affairs Quiz 6th March 2024

For Previous Daily Quiz (ARCHIVES) – CLICK HERE The Current Affairs questions are based on sources like ‘The Hindu’, ‘Indian Express’ and ‘PIB’, which are very important sources for UPSC Prelims Exam. The questions are focused on both the concepts and facts. The topics covered here are generally different from what is being covered under ‘Daily Current Affairs/Daily News Analysis (DNA) and Daily Static Quiz’ to avoid duplication. The questions would be published from Monday to Saturday before 2 PM. One should not spend more than 10 minutes on this initiative. Gear up and Make the Best Use of this initiative. Do remember that, “the difference between Ordinary and EXTRA-Ordinary is PRACTICE!!” Important Note: Don’t forget to post your marks in the comment section. Also, let us know if you enjoyed today’s test 🙂After completing the 5 questions, click on ‘View Questions’ to check your score, time taken, and solutions. To take the Test Click Here

[DAY 03] 60 DAY RAPID REVISION (RaRe) SERIES for UPSC Prelims 2024 – HISTORY, CURRENT AFFAIRS & CSAT TEST SERIES!

Archives Hello Friends The 60 Days Rapid Revision (RaRe) Series is IASbaba’s Flagship Initiative recommended by Toppers and loved by the aspirants’ community every year. It is the most comprehensive program which will help you complete the syllabus, revise and practice tests on a daily basis. The Programme on a daily basis includes Daily Prelims MCQs from Static (Monday – Saturday) Daily Static Quiz will cover all the topics of static subjects – Polity, History, Geography, Economics, Environment and Science and technology. 20 questions will be posted daily and these questions are framed from the topics mentioned in the schedule. It will ensure timely and streamlined revision of your static subjects. Daily Current Affairs MCQs (Monday – Saturday) Daily 5 Current Affairs questions, based on sources like ‘The Hindu’, ‘Indian Express’ and ‘PIB’, would be published from Monday to Saturday according to the schedule. Daily CSAT Quiz (Monday – Friday) CSAT has been an Achilles heel for many aspirants. Daily 5 CSAT Questions will be published. Note – Daily Test of 20 static questions, 10 current affairs, and 5 CSAT questions. (35 Prelims Questions) in QUIZ FORMAT will be updated on a daily basis. To Know More about 60 Days Rapid Revision (RaRe) Series – CLICK HERE   60 Day Rapid Revision (RaRe) Series Schedule – CLICK HERE  Important Note Comment your Scores in the Comment Section. This will keep you accountable, responsible and sincere in days to come. It will help us come out with the Cut-Off on a Daily Basis. Let us know if you enjoyed today’s test 🙂  You can post your comments in the given format  (1) Your Score (2) Matrix Meter (3) New Learning from the Test Time limit: 0 Test-summary 0 of 35 questions completed Questions: 1 2 3 4 5 6 7 8 9 10 11 12 13 14 15 16 17 18 19 20 21 22 23 24 25 26 27 28 29 30 31 32 33 34 35 Information The following Test is based on the syllabus of 60 Days Plan-2023 for UPSC IAS Prelims 2022. To view Solutions, follow these instructions: Click on – ‘Start Test’ button Solve Questions Click on ‘Test Summary’ button Click on ‘Finish Test’ button Now click on ‘View Questions’ button – here you will see solutions and links. You have already completed the test before. Hence you can not start it again. Test is loading... You must sign in or sign up to start the test. You have to finish following test, to start this test: Results 0 of 35 questions answered correctly Your time: Time has elapsed You have scored 0 points out of 0 points, (0) Average score     Your score     Categories Not categorized 0% Your result has been entered into leaderboard Loading Name: E-Mail: Captcha: maximum of 70 points Pos. Name Entered on Points Result Table is loading No data available 1 2 3 4 5 6 7 8 9 10 11 12 13 14 15 16 17 18 19 20 21 22 23 24 25 26 27 28 29 30 31 32 33 34 35 Answered Review Question 1 of 35 1. Question Which of the following can be considered as the major factors that can be attributed to the success of the English against other European powers in India? The complete government ownership of the English East India Company. The advancement of the Royal Navy of Britain. Beginning of Industrial Revolution in England. Use of debt markets by Britain to fund its wars. How many of the above statements are correct?  a) Only one b) Only two c) Only three d) All four Correct Solution (c) Statement 1 Statement 2 Statement 3 Statement 4 Incorrect Correct Correct Correct Structure and Nature of the Trading Companies: The English East India Company, formed through the amalgamation of several rival companies at home, was controlled by a board of directors whose members were elected annually. The shareholders of the company exercised considerable influence, as the votes could be bought and sold through the purchase of shares. The trading companies of France and Portugal were largely owned by the State and their nature was in many ways feudalistic. Naval Superiority: The Royal Navy of Britain was not only the largest; it was the most advanced of its times. The victory against the Spanish Armada and against the French at Trafalgar had put the Royal Navy at the peak of the European naval forces. In India too, the British were able to defeat the Portuguese and the French due to the strong and fast movement of the naval ships. The English learned from the Portuguese the importance of an efficient navy and improved their own fleet technologically. Industrial Revolution: The Industrial Revolution started in England in the early 18th century, with the invention of new machines like the spinning Jenny, steam engine, the power loom and several others. These machines greatly improved productivity in the fields of textiles, metallurgy, steam power, and agriculture. The industrial revolution reached other European nations late and this helped England to maintain its hegemony. Use of Debt Market: One of the major and innovative reasons why Britain succeeded between the mid-eighteenth century and the mid-nineteenth century, while other European nations fell, was that it used the debt markets to fund its wars. The world‘s first central bank—the Bank of England—was established to sell government debt to the money markets on the promise of a decent return on Britain‘s defeating rival countries like France and Spain. Britain was thus enabled to spend much more on its military than its rivals. Britain’s rival France could not match the expenditure of the English; between 1694 and 1812, first under the monarchs, then under the revolutionary governments, and finally under Napoleon Bonaparte, France simply went bankrupt with its outdated ways of raising money. Incorrect Solution (c) Statement 1 Statement 2 Statement 3 Statement 4 Incorrect Correct Correct Correct Structure and Nature of the Trading Companies: The English East India Company, formed through the amalgamation of several rival companies at home, was controlled by a board of directors whose members were elected annually. The shareholders of the company exercised considerable influence, as the votes could be bought and sold through the purchase of shares. The trading companies of France and Portugal were largely owned by the State and their nature was in many ways feudalistic. Naval Superiority: The Royal Navy of Britain was not only the largest; it was the most advanced of its times. The victory against the Spanish Armada and against the French at Trafalgar had put the Royal Navy at the peak of the European naval forces. In India too, the British were able to defeat the Portuguese and the French due to the strong and fast movement of the naval ships. The English learned from the Portuguese the importance of an efficient navy and improved their own fleet technologically. Industrial Revolution: The Industrial Revolution started in England in the early 18th century, with the invention of new machines like the spinning Jenny, steam engine, the power loom and several others. These machines greatly improved productivity in the fields of textiles, metallurgy, steam power, and agriculture. The industrial revolution reached other European nations late and this helped England to maintain its hegemony. Use of Debt Market: One of the major and innovative reasons why Britain succeeded between the mid-eighteenth century and the mid-nineteenth century, while other European nations fell, was that it used the debt markets to fund its wars. The world‘s first central bank—the Bank of England—was established to sell government debt to the money markets on the promise of a decent return on Britain‘s defeating rival countries like France and Spain. Britain was thus enabled to spend much more on its military than its rivals. Britain’s rival France could not match the expenditure of the English; between 1694 and 1812, first under the monarchs, then under the revolutionary governments, and finally under Napoleon Bonaparte, France simply went bankrupt with its outdated ways of raising money. Question 2 of 35 2. Question With reference to the Treaty of Allahabad in 1765, which of the following statements is/are correct? Allahabad and Banaras were surrendered to Shah Alam II by Nawab Shuja-ud-Daula. Shah Alam – II agreed to pay Rs.50 lakh to company as war indemnity. Which of the above statements is/are correct? a) 1 only b) 2 only c) Both 1 and 2 d) Neither 1 nor 2 Correct Solution (d) Statement 1 Statement 2 Incorrect Incorrect Robert Clive concluded two important treaties at Allahabad in August 1765 – one with the Nawab of Awadh and the other with the Mughal Emperor, Shah Alam II. Under this treaty, Nawab of Awadh Shuja-ud-Daula agreed to surrender Allahabad and Kara to Emperor Shah Alam II. Under this treaty, Nawab of Awadh Shuja-ud-Daula agreed to pay Rs 50 lakh to the Company as war indemnity. Incorrect Solution (d) Statement 1 Statement 2 Incorrect Incorrect Robert Clive concluded two important treaties at Allahabad in August 1765 – one with the Nawab of Awadh and the other with the Mughal Emperor, Shah Alam II. Under this treaty, Nawab of Awadh Shuja-ud-Daula agreed to surrender Allahabad and Kara to Emperor Shah Alam II. Under this treaty, Nawab of Awadh Shuja-ud-Daula agreed to pay Rs 50 lakh to the Company as war indemnity. Question 3 of 35 3. Question With reference to rule of European powers in India, who among the following introduced Blue Water Policy? a) British b) Portuguese c) French d) Danish Correct Solution (b) Blue Water Policy: In 1505, the King of Portugal appointed a governor Francisco De Almeida in India for a three-year term and he was asked to consolidate the position of the Portuguese in India and to destroy Muslim trade by seizing Aden, Ormuz and Malacca. Francisco de Almeida wanted that Portuguese should maintain supremacy on sea and confine their activities to purely commercial transactions and to realize his vision he implemented Blue Water Policy. The Blue Water Policy was the fortification of Indian Ocean, not for the security but for the establishment of Portuguese business in the Indian Ocean. The Blue water policy was reversed by Alfonso de Albuquerque. Incorrect Solution (b) Blue Water Policy: In 1505, the King of Portugal appointed a governor Francisco De Almeida in India for a three-year term and he was asked to consolidate the position of the Portuguese in India and to destroy Muslim trade by seizing Aden, Ormuz and Malacca. Francisco de Almeida wanted that Portuguese should maintain supremacy on sea and confine their activities to purely commercial transactions and to realize his vision he implemented Blue Water Policy. The Blue Water Policy was the fortification of Indian Ocean, not for the security but for the establishment of Portuguese business in the Indian Ocean. The Blue water policy was reversed by Alfonso de Albuquerque. Question 4 of 35 4. Question Consider the following pairs: Battle                                                  Emperor/Ruler associated Battle of Karnal                        Ahmed Shah Battle of Plassey                     Muhammad Shah Battle of Buxar                           Shah Alam II How many of the above pairs are correctly matched?  a) Only one b) Only two c) All three d) None Correct Solution (a) Statement 1 Statement 2 Statement 3 Incorrect Incorrect Correct Battle of Karnal: It was held between Nadir Shah and Mughals (Emperor Muhammad Shah) in 1739 and Mughals were defeated in the war and later Muhammad Shah was imprisoned and annexed areas west of the Indus into the Persian Empire. Battle of Plassey: The battle was fought between the East India Company headed by Robert Clive and the Nawab of Bengal (Siraj-Ud-Daulah) and his French Troop in 1757. This battle is often termed as the ‘decisive event’ which became the source of ultimate rule of the British in India. Mughal emperor Alamgir-II was ruling the empire when the Battle of Plassey took place. Battle of Buxar: It was a battle fought between the English Forces, and a joint army of the Nawab of Oudh, Nawab of Bengal, and the Mughal Emperor Shah Alam II in 1764. The battle was the result of misuse of trade privileges granted by the Nawab of Bengal and also the colonialist ambitions of East India Company. Incorrect Solution (a) Statement 1 Statement 2 Statement 3 Incorrect Incorrect Correct Battle of Karnal: It was held between Nadir Shah and Mughals (Emperor Muhammad Shah) in 1739 and Mughals were defeated in the war and later Muhammad Shah was imprisoned and annexed areas west of the Indus into the Persian Empire. Battle of Plassey: The battle was fought between the East India Company headed by Robert Clive and the Nawab of Bengal (Siraj-Ud-Daulah) and his French Troop in 1757. This battle is often termed as the ‘decisive event’ which became the source of ultimate rule of the British in India. Mughal emperor Alamgir-II was ruling the empire when the Battle of Plassey took place. Battle of Buxar: It was a battle fought between the English Forces, and a joint army of the Nawab of Oudh, Nawab of Bengal, and the Mughal Emperor Shah Alam II in 1764. The battle was the result of misuse of trade privileges granted by the Nawab of Bengal and also the colonialist ambitions of East India Company. Question 5 of 35 5. Question Consider the following statements regarding the Battle of Buxar: This made the English a great power in northern India and contenders for the supremacy over the whole country. The treaty of Allahabad made Nawab of Awadh a firm friend of the company as Awadh was not annexed. The treaty of Allahabad made Shah Alam II agreed to reside at Allahabad which was to be ceded to him by Nawab of Awadh under the company’s protection. How many of the above statements are correct? a) Only one b) Only two c) All three d) None Correct Solution (c) Statement 1 Statement 2 Statement 3 Correct Correct Correct In the Battle of Buxar, not only the Nawab of Bengal but also the Mughal emperor of India was defeated by the English. The victory made the English a great power in northern India and contenders for the supremacy over the whole country. Clive did not want to annex Awadh because it would have placed the company under obligation to protect an extensive land frontier from the Afghan and the Maratha invasions. The treaty made Nawab a firm friend of the company and turned Awadh into a buffer state. The battle culminated into Treaty of Allahabad according to which Shah Alam II, the Mughal emperor agreed to reside at Allahabad, to be ceded to him by the Nawab of Awadh, under the company’s protection. He issued a Farman granting the Diwani rights of Bengal, Bihar and Orissa to the East India Company in lieu of an annual payment.   Incorrect Solution (c) Statement 1 Statement 2 Statement 3 Correct Correct Correct In the Battle of Buxar, not only the Nawab of Bengal but also the Mughal emperor of India was defeated by the English. The victory made the English a great power in northern India and contenders for the supremacy over the whole country. Clive did not want to annex Awadh because it would have placed the company under obligation to protect an extensive land frontier from the Afghan and the Maratha invasions. The treaty made Nawab a firm friend of the company and turned Awadh into a buffer state. The battle culminated into Treaty of Allahabad according to which Shah Alam II, the Mughal emperor agreed to reside at Allahabad, to be ceded to him by the Nawab of Awadh, under the company’s protection. He issued a Farman granting the Diwani rights of Bengal, Bihar and Orissa to the East India Company in lieu of an annual payment.   Question 6 of 35 6. Question Consider the following statements: Under the Treaty of 1760, Mir Kasim ceded to the English the districts of Burdwan, Midnapur and Chittagong. The Mughal Emperor of India was a participant in the Battle of Buxar. The Treaty of Allahabad granted the diwani rights of Bengal, Bihar and Orissa to the East India Company. How many of the above statements are correct?  a) Only one b) Only two c) All three d) None Correct Solution (c) Statement 1 Statement 2 Statement 3 Correct Correct Correct After the Battle of Plassey, Mir Jafar was made the nawab of Bengal by the British. He was expected to show his generosity to the British. However, he got irritated with the interference of Robert Clive and entered into a conspiracy with the Dutch. The Dutch were defeated by the English in the Battle of Bedara in 1759. Hence, the Treaty of 1760 was signed between the new puppet nawab of Bengal, Mir Kasim and the English. Under the treaty, Mir Kasim agreed to cede the districts of Burdwan, Midnapore and Chittagong. The Battle of Buxar was fought between the combined armies of Mir Kasim, Shah Alam II and the Nawab of Awadh on 22nd October 1764. Shah Alam II was the then Mughal Emperor of India. The Battle of Buxar was important in the sense that for the first time, the Emperor of India was defeated by the English. Robert Clive concluded a treaty with the Mughal Emperor, Shah Alam II. Under the Treaty of Allahabad (1765), Shah Alam II granted the diwani rights of Bengal, Bihar and Orissa to the East India Company.   Incorrect Solution (c) Statement 1 Statement 2 Statement 3 Correct Correct Correct After the Battle of Plassey, Mir Jafar was made the nawab of Bengal by the British. He was expected to show his generosity to the British. However, he got irritated with the interference of Robert Clive and entered into a conspiracy with the Dutch. The Dutch were defeated by the English in the Battle of Bedara in 1759. Hence, the Treaty of 1760 was signed between the new puppet nawab of Bengal, Mir Kasim and the English. Under the treaty, Mir Kasim agreed to cede the districts of Burdwan, Midnapore and Chittagong. The Battle of Buxar was fought between the combined armies of Mir Kasim, Shah Alam II and the Nawab of Awadh on 22nd October 1764. Shah Alam II was the then Mughal Emperor of India. The Battle of Buxar was important in the sense that for the first time, the Emperor of India was defeated by the English. Robert Clive concluded a treaty with the Mughal Emperor, Shah Alam II. Under the Treaty of Allahabad (1765), Shah Alam II granted the diwani rights of Bengal, Bihar and Orissa to the East India Company.   Question 7 of 35 7. Question Consider the following statements: The French were the last Europeans to be defeated by the British in India. Burma was the last territory to be annexed by the British into India. Which of the statements given above is/are correct? a) 1 only b) 2 only c) Both 1 and 2 d) Neither 1 nor 2 Correct Solution (c) Statement 1 Statement 2 Correct Correct The treaty of Paris made after the third Anglo Carnatic war in 1763 made French to the last Europeans to be defeated by the British in India. The Third Anglo Burmese war marked the end of the Annexation of the British Indian empire. With the addition of Burma province into British India in 1886, the lieutenant governorship was established into the province. Incorrect Solution (c) Statement 1 Statement 2 Correct Correct The treaty of Paris made after the third Anglo Carnatic war in 1763 made French to the last Europeans to be defeated by the British in India. The Third Anglo Burmese war marked the end of the Annexation of the British Indian empire. With the addition of Burma province into British India in 1886, the lieutenant governorship was established into the province. Question 8 of 35 8. Question Though the Portuguese were the first to arrive in India, they were incapable of maintaining a trade monopoly in the East for a long because- Its merchants enjoyed more power than its landed aristocrats. Portuguese lagged behind in the development of shipping as compared to other European powers. They followed a policy of religious intolerance. How many of the above statements are correct?  a) Only one b) Only two c) All three d) None Correct Solution (b) Statement 1 Statement 2 Statement 3 Incorrect Correct Correct By the 18th century, the Portuguese in India lost their commercial influence, though some of them still carried on trade in their individual capacity and many took to piracy and robbery. The decline of the Portuguese was brought about by several factors. The local advantages gained by the Portuguese in India were reduced with the emergence of powerful dynasties in Egypt, Persia, and North India and the rise of the turbulent Marathas as their immediate neighbors. (The Marathas captured Salsette and Bassein in 1739 from the Portuguese). The population of Portuguese was less than a million, its Court was autocratic and decadent, and its merchants enjoyed much less power and prestige than its landed aristocrats. The earlier monopoly of knowledge of the sea route to India held by the Portuguese could not remain a secret forever; soon enough the Dutch and the English, who were learning the skills of ocean navigation, also learned it and outshined the Portuguese. They lagged behind in the development of shipping as compared to other European powers. They followed a policy of religious intolerance. The religious policies of the Portuguese, such as the activities of the Jesuits, gave rise to political fears. Their antagonism for the Muslims apart, the Portuguese policy of conversion to Christianity made Hindus also resentful. Note: As new trading communities from Europe arrived in India, there began a fierce rivalry among them. In this struggle, the Portuguese had to give way to the more powerful and enterprising competitors. The Dutch and the English had greater resources and more compulsions to expand overseas, and they overcame the Portuguese resistance. Incorrect Solution (b) Statement 1 Statement 2 Statement 3 Incorrect Correct Correct By the 18th century, the Portuguese in India lost their commercial influence, though some of them still carried on trade in their individual capacity and many took to piracy and robbery. The decline of the Portuguese was brought about by several factors. The local advantages gained by the Portuguese in India were reduced with the emergence of powerful dynasties in Egypt, Persia, and North India and the rise of the turbulent Marathas as their immediate neighbors. (The Marathas captured Salsette and Bassein in 1739 from the Portuguese). The population of Portuguese was less than a million, its Court was autocratic and decadent, and its merchants enjoyed much less power and prestige than its landed aristocrats. The earlier monopoly of knowledge of the sea route to India held by the Portuguese could not remain a secret forever; soon enough the Dutch and the English, who were learning the skills of ocean navigation, also learned it and outshined the Portuguese. They lagged behind in the development of shipping as compared to other European powers. They followed a policy of religious intolerance. The religious policies of the Portuguese, such as the activities of the Jesuits, gave rise to political fears. Their antagonism for the Muslims apart, the Portuguese policy of conversion to Christianity made Hindus also resentful. Note: As new trading communities from Europe arrived in India, there began a fierce rivalry among them. In this struggle, the Portuguese had to give way to the more powerful and enterprising competitors. The Dutch and the English had greater resources and more compulsions to expand overseas, and they overcame the Portuguese resistance. Question 9 of 35 9. Question Consider the following statements with reference to the Anglo-Dutch rivalry: The Anglo-Dutch compromise involved the Dutch withdrawal from the spice islands of Indonesia. The victory of the British in the Battle of Hooghly led to the final collapse of the Dutch in India. Which of the statements given above is/are correct? a) 1 only b) 2 only c) Both 1 and 2 d) Neither 1 nor 2 Correct Solution (b) Statement 1 Statement 2 Incorrect Correct In the middle of the seventeenth century, the English began to emerge as a big colonial power. The Anglo-Dutch rivalry lasted for about seven decades during which period the Dutch lost their settlements to the British one by one. The English were also at this time rising to prominence in the Eastern trade, and this posed a serious challenge to the commercial interests of the Dutch. A commercial rivalry soon turned into bloody warfare. After prolonged warfare, both the parties came to a compromise in 1667 by which the British agreed to withdraw all their claims on Indonesia, and the Dutch retired from India to concentrate on their more profitable trade in Indonesia. They monopolized the trade in black pepper and spices. In the Third Anglo-Dutch War (1672-74), communications between Surat and the new English settlement of Bombay got cut due to which three homebound English ships were captured in the Bay of Bengal by the Dutch forces. The retaliation by the English resulted in the defeat of the Dutch. The Battle of Chinsurah (also known as the Battle of Hooghly) took place near Chinsurah, India on 25 November 1759), which dealt a crushing blow to Dutch ambitions in India. Incorrect Solution (b) Statement 1 Statement 2 Incorrect Correct In the middle of the seventeenth century, the English began to emerge as a big colonial power. The Anglo-Dutch rivalry lasted for about seven decades during which period the Dutch lost their settlements to the British one by one. The English were also at this time rising to prominence in the Eastern trade, and this posed a serious challenge to the commercial interests of the Dutch. A commercial rivalry soon turned into bloody warfare. After prolonged warfare, both the parties came to a compromise in 1667 by which the British agreed to withdraw all their claims on Indonesia, and the Dutch retired from India to concentrate on their more profitable trade in Indonesia. They monopolized the trade in black pepper and spices. In the Third Anglo-Dutch War (1672-74), communications between Surat and the new English settlement of Bombay got cut due to which three homebound English ships were captured in the Bay of Bengal by the Dutch forces. The retaliation by the English resulted in the defeat of the Dutch. The Battle of Chinsurah (also known as the Battle of Hooghly) took place near Chinsurah, India on 25 November 1759), which dealt a crushing blow to Dutch ambitions in India. Question 10 of 35 10. Question The Battle of Wandiwash was one of the decisive battles in the struggle among European powers in India. It was fought between which of the following countries? a) The Dutch and the Portuguese b) The French and the British c) The Portuguese and the British d) The Danes and the Dutch Correct Solution (b) Battle of Wandiwash: The outbreak of the Seven Years’ War in Europe between England and France in 1756 provided the context for the third and decisive round of the Anglo-French conflict in south India. The most decisive battle of the Third Carnatic War was the Battle of Wandiwash in 1760. In the Battle of Wandiwash, General Eyre Coote of the English defeated the French army under Count Thomas Arthur de Lally and took Bussy as a prisoner. French also lost Pondicherry to the British. During the Third Carnatic War, the French lost their positions in India. However, by the Peace of Paris in 1763 France got back all the factories and settlements that it possessed in India prior to 1749. But the balance of power in India had by now decisively changed with the steady expansion of the power of the English company. Incorrect Solution (b) Battle of Wandiwash: The outbreak of the Seven Years’ War in Europe between England and France in 1756 provided the context for the third and decisive round of the Anglo-French conflict in south India. The most decisive battle of the Third Carnatic War was the Battle of Wandiwash in 1760. In the Battle of Wandiwash, General Eyre Coote of the English defeated the French army under Count Thomas Arthur de Lally and took Bussy as a prisoner. French also lost Pondicherry to the British. During the Third Carnatic War, the French lost their positions in India. However, by the Peace of Paris in 1763 France got back all the factories and settlements that it possessed in India prior to 1749. But the balance of power in India had by now decisively changed with the steady expansion of the power of the English company. Question 11 of 35 11. Question Consider the following pairs: European Commercial Companies                            Base Portuguese                                                          Panaji Dutch                                                                  Masulipatnam French                                                                 Pondicherry How many of the above pairs are correctly matched?  a) Only one b) Only two c) All three d) None Correct Solution (c) Statement 1 Statement 2 Statement 3 Correct Correct Correct Portuguese – Panaji in 1510 Dutch – Masulipatnam in 1605 French – Pondicherry in 1673 Note: The European commercial Companies had set up base in different places early during the Mughal era. With the expansion of commercial activity, towns grew around these trading centers. By the end of the eighteenth century the land-based empires in Asia were replaced by the powerful sea-based European empires. Forces of international trade, mercantilism and capitalism now came to define the nature of society. Incorrect Solution (c) Statement 1 Statement 2 Statement 3 Correct Correct Correct Portuguese – Panaji in 1510 Dutch – Masulipatnam in 1605 French – Pondicherry in 1673 Note: The European commercial Companies had set up base in different places early during the Mughal era. With the expansion of commercial activity, towns grew around these trading centers. By the end of the eighteenth century the land-based empires in Asia were replaced by the powerful sea-based European empires. Forces of international trade, mercantilism and capitalism now came to define the nature of society. Question 12 of 35 12. Question Which of the following statements is/are correct regarding Battle of Plassey? It ousted the French permanently from Bengal. It recognized the sovereignty of the English over Calcutta. It culminated into the Treaty of Alinagar. How many of the above statements are correct?  a) Only one b) Only two c) All three d) None Correct Solution (c) Statement 1 Statement 2 Statement 3 Correct Correct Correct The Battle of Plassey, 1757 had laid the foundation of British Empire in India. Robert Clive forged a secret alliance with Mir Jafar to win the battle. Due to the conspiracy of Nawab’s officials, 50,000 strong forces of Siraj was defeated by a handful of Clive’s forces. After the victory of British, Mir Jafer became the Nawab of Bengal. He gave large sums of money plus the Zamindari of 24 parganas to the English.The battle established the military supremacy of the English over Bengal and their main rivals French were ousted. The sovereignty of the English over Calcutta was recognized and the English posted a Resident at the Nawab’s court. Siraj-ud- daula had to sign treaty of Alinagar (the new name of Calcutta, practically conceding all demands of the English. The English became so confident that they attacked Chandernagar, the French settlement again challenging the sovereignty of Nawab. Incorrect Solution (c) Statement 1 Statement 2 Statement 3 Correct Correct Correct The Battle of Plassey, 1757 had laid the foundation of British Empire in India. Robert Clive forged a secret alliance with Mir Jafar to win the battle. Due to the conspiracy of Nawab’s officials, 50,000 strong forces of Siraj was defeated by a handful of Clive’s forces. After the victory of British, Mir Jafer became the Nawab of Bengal. He gave large sums of money plus the Zamindari of 24 parganas to the English.The battle established the military supremacy of the English over Bengal and their main rivals French were ousted. The sovereignty of the English over Calcutta was recognized and the English posted a Resident at the Nawab’s court. Siraj-ud- daula had to sign treaty of Alinagar (the new name of Calcutta, practically conceding all demands of the English. The English became so confident that they attacked Chandernagar, the French settlement again challenging the sovereignty of Nawab. Question 13 of 35 13. Question Consider the following statements: The French were the last Europeans to come to India with the purpose of trade. Mahe and Balasore were important trading centres of the French East India Company. Which of the statements given above is/are correct? a) 1 only b) 2 only c) Both 1 and 2 d) Neither 1 nor 2 Correct Solution (c) Statement 1 Statement 2 Correct Correct The French harbored a wish to engage in the commerce of the East since the opening years of the sixteenth century, their appearance on the Indian coasts was late. They were the last Europeans to come to India with the purpose of trade. Colbert laid the foundation of the Compagnie des Indes Orientales (French East India Company) in 1664, which was granted a 50-year monopoly on French trade in the Indian and Pacific Oceans. In 1667, Francois Caron headed an expedition to India. Consequently, the first French factory in India was established at Surat in 1668 and a later one at Masulipatnam in 1669. In 1673, Sher Khan Lodi, the governor of Valikondapuram (under the Bijapur Sultan), granted Francois Martin, the director of the Masulipatnam factory, a site for a settlement. Pondicherry was founded in 1674. In the same year, Francois Martin replaced Caron as the French governor. The French company established its factories in other parts of India also, particularly in the coastal regions. Mahe, Karaikal, Balasore and Qasim Bazar were a few important trading centres of the French East India Company.   Incorrect Solution (c) Statement 1 Statement 2 Correct Correct The French harbored a wish to engage in the commerce of the East since the opening years of the sixteenth century, their appearance on the Indian coasts was late. They were the last Europeans to come to India with the purpose of trade. Colbert laid the foundation of the Compagnie des Indes Orientales (French East India Company) in 1664, which was granted a 50-year monopoly on French trade in the Indian and Pacific Oceans. In 1667, Francois Caron headed an expedition to India. Consequently, the first French factory in India was established at Surat in 1668 and a later one at Masulipatnam in 1669. In 1673, Sher Khan Lodi, the governor of Valikondapuram (under the Bijapur Sultan), granted Francois Martin, the director of the Masulipatnam factory, a site for a settlement. Pondicherry was founded in 1674. In the same year, Francois Martin replaced Caron as the French governor. The French company established its factories in other parts of India also, particularly in the coastal regions. Mahe, Karaikal, Balasore and Qasim Bazar were a few important trading centres of the French East India Company.   Question 14 of 35 14. Question The Battle of Plassey paved the way for the British mastery of Bengal and eventually of the whole of India. Which of the following were the reasons for the Battle of Plassey? Misuse of dastaks by company’s officials in private capacity Fortification of Calcutta Abolition of duties on internal trade by the Nawab Shifting of capital from Murshidabad to Munger How many of the above statements are correct?  a) Only one b) Only two c) Only three d) All four Correct Solution (b) Statement 1 Statement 2 Statement 3 Statement 4 Correct Correct Incorrect Incorrect This Farman was a perpetual source of conflict between the Company and the Nawabs of Bengal. For one, it meant the loss of revenue to the Bengal Government. Secondly, the power to issue dastaks for the Company’s goods was misused by the Company’s servants to evade taxes on their private trade. Without taking the Nawab’s permission, the Company began to fortify Calcutta in expectation of the coming struggle with the French, who were stationed at this time at Chandernagore. Mir Qasim treated Indian merchants and English as same, without granting any special privileges for the latter. Therefore, abolition of duties on internal trade by the Nawab Mir Qasim was one of the reasons to Battle of Buxar. Mir Qasim wanted to be independent and shifted his capital to Munger Fort from Calcutta. This was one of the reasons to Battle of Buxar.   Causes of the Battle of Plassey: The Company had secured valuable privileges in 1717 under a royal farman by the Mughal Emperor, which had granted the Company the freedom to export and import their goods in Bengal without paying taxes and the right to issue passes or dastaks for the movement of such goods. The Company’s servants were also permitted to trade but were not covered by this Farman and were required to pay the same taxes as Indian merchants. This Farman was a perpetual source of conflict between the Company and the Nawabs of Bengal. For one, it meant the loss of revenue to the Bengal Government. Secondly, the power to issue dastaks for the Company’s goods was misused by the Company’s servants to evade taxes on their private trade. Without taking the Nawab’s permission, the Company began to fortify Calcutta in expectation of the coming struggle with the French, who were stationed at this time at Chandernagore. Siraj-ud-Daulah interpreted this as an attack on his sovereignty and ordered both the English and French to demolish their fortifications. While the French obliged, English refused to do so. This set the stage for a battle that took place on the fi eld of Plassey on 23rd June 1757. The fateful battle of Plassey was a battle only in name. In all, the English lost 29 men while the Nawab lost nearly 500. The major part of the Nawab’s army, led by the traitors Mir Jafar and Rai Durlabh, took no part in the fighting. After the battle, Mir Jafar was proclaimed the Nawab of Bengal and the company was granted undisputed right to free trade in Bengal, Bihar and Orissa. It also received the zamindari of the 24 Parganas near Calcutta. Incorrect Solution (b) Statement 1 Statement 2 Statement 3 Statement 4 Correct Correct Incorrect Incorrect This Farman was a perpetual source of conflict between the Company and the Nawabs of Bengal. For one, it meant the loss of revenue to the Bengal Government. Secondly, the power to issue dastaks for the Company’s goods was misused by the Company’s servants to evade taxes on their private trade. Without taking the Nawab’s permission, the Company began to fortify Calcutta in expectation of the coming struggle with the French, who were stationed at this time at Chandernagore. Mir Qasim treated Indian merchants and English as same, without granting any special privileges for the latter. Therefore, abolition of duties on internal trade by the Nawab Mir Qasim was one of the reasons to Battle of Buxar. Mir Qasim wanted to be independent and shifted his capital to Munger Fort from Calcutta. This was one of the reasons to Battle of Buxar.   Causes of the Battle of Plassey: The Company had secured valuable privileges in 1717 under a royal farman by the Mughal Emperor, which had granted the Company the freedom to export and import their goods in Bengal without paying taxes and the right to issue passes or dastaks for the movement of such goods. The Company’s servants were also permitted to trade but were not covered by this Farman and were required to pay the same taxes as Indian merchants. This Farman was a perpetual source of conflict between the Company and the Nawabs of Bengal. For one, it meant the loss of revenue to the Bengal Government. Secondly, the power to issue dastaks for the Company’s goods was misused by the Company’s servants to evade taxes on their private trade. Without taking the Nawab’s permission, the Company began to fortify Calcutta in expectation of the coming struggle with the French, who were stationed at this time at Chandernagore. Siraj-ud-Daulah interpreted this as an attack on his sovereignty and ordered both the English and French to demolish their fortifications. While the French obliged, English refused to do so. This set the stage for a battle that took place on the fi eld of Plassey on 23rd June 1757. The fateful battle of Plassey was a battle only in name. In all, the English lost 29 men while the Nawab lost nearly 500. The major part of the Nawab’s army, led by the traitors Mir Jafar and Rai Durlabh, took no part in the fighting. After the battle, Mir Jafar was proclaimed the Nawab of Bengal and the company was granted undisputed right to free trade in Bengal, Bihar and Orissa. It also received the zamindari of the 24 Parganas near Calcutta. Question 15 of 35 15. Question Which of the following statements is incorrect with regard to Subsidiary Alliance? a) Indian rulers were not allowed to have their independent Armed forces. b) The company was supposed to provide protection to the Indian States in lieu of Diwani Rights within the States. c) If the Indian rulers failed to make the payment, then part of their territory was taken away as penalty. d) It was used by Lord Wellesley to further the process of expansion of the Company’s rule in India. Correct Solution (b) a)       b)       c)       d)       Correct Incorrect Correct Correct According to the terms of Subsidiary Alliance, the Indian rulers were not allowed to have their independent armed forces. They were to be protected by the Company, but had to pay for the ‘subsidiary forces’ that the Company was supposed to maintain for the purpose of this protection. The States had to pay the company for the protection under the Subsidiary Alliance in the form of cash or territory. Diwani rights were not given to the company. According to the terms of Subsidiary Alliance, if the Indian rulers failed to make the payment, then part of their territory was taken away as penalty. The policy of Subsidiary alliance was used by Lord Wellesley to further the process of expansion of the Company’s rule in India. When he was Governor General (1798-1805), the Nawab of Awadh was forced to give over half of his territory to the Company in 1801, as he failed to pay for the ‘subsidiary forces’. Hyderabad was also forced to cede territories on similar grounds.   Incorrect Solution (b) a)       b)       c)       d)       Correct Incorrect Correct Correct According to the terms of Subsidiary Alliance, the Indian rulers were not allowed to have their independent armed forces. They were to be protected by the Company, but had to pay for the ‘subsidiary forces’ that the Company was supposed to maintain for the purpose of this protection. The States had to pay the company for the protection under the Subsidiary Alliance in the form of cash or territory. Diwani rights were not given to the company. According to the terms of Subsidiary Alliance, if the Indian rulers failed to make the payment, then part of their territory was taken away as penalty. The policy of Subsidiary alliance was used by Lord Wellesley to further the process of expansion of the Company’s rule in India. When he was Governor General (1798-1805), the Nawab of Awadh was forced to give over half of his territory to the Company in 1801, as he failed to pay for the ‘subsidiary forces’. Hyderabad was also forced to cede territories on similar grounds.   Question 16 of 35 16. Question With reference to the policy of annexation by diplomacy and administrative mechanisms by British, consider the following pairs: Governor-General                                          Policy Lord Hastings                                     Ring fence Lord Wellesley                                Subsidiary alliance Lord Dalhousie                              Doctrine of lapse How many of the above pairs are correctly matched?  a) Only one b) Only two c) All three d) None Correct Solution (c) Statement 1 Statement 2 Statement 3 Correct Correct Correct Policy of Ring Fence: It is followed by Warren Hastings, under which aimed at creating buffer zones to defend the Company’s frontiers. Broadly speaking, it was the policy of defence of their neighbours’ frontiers for safeguarding their own territories. This policy of Warren Hastings was reflected in his war against the Marathas and Mysore. Policy of Subsidiary Alliance: It is followed by Lord Wellesley, under which the allying Indian state’s ruler was compelled to accept the permanent stationing of a British force within his territory and to pay a subsidy for its maintenance. Also, the Indian ruler had to agree to the posting of a British resident in his court. Under the system, the Indian ruler could not employ any European in his service without the prior approval of the British. Nor could he negotiate with any other Indian ruler without consulting the governor-general. In return for all this, the British would defend the ruler from his enemies and adopt a policy of noninterference in the internal matters of the allied state. Policy of Doctrine of Lapse: It is followed by Lord Dalhousie, under which the adopted son could be the heir to his foster father’s private property, but not the state; it was for the paramount power (the British) to decide whether to bestow the state on the adopted son or to annex it. Incorrect Solution (c) Statement 1 Statement 2 Statement 3 Correct Correct Correct Policy of Ring Fence: It is followed by Warren Hastings, under which aimed at creating buffer zones to defend the Company’s frontiers. Broadly speaking, it was the policy of defence of their neighbours’ frontiers for safeguarding their own territories. This policy of Warren Hastings was reflected in his war against the Marathas and Mysore. Policy of Subsidiary Alliance: It is followed by Lord Wellesley, under which the allying Indian state’s ruler was compelled to accept the permanent stationing of a British force within his territory and to pay a subsidy for its maintenance. Also, the Indian ruler had to agree to the posting of a British resident in his court. Under the system, the Indian ruler could not employ any European in his service without the prior approval of the British. Nor could he negotiate with any other Indian ruler without consulting the governor-general. In return for all this, the British would defend the ruler from his enemies and adopt a policy of noninterference in the internal matters of the allied state. Policy of Doctrine of Lapse: It is followed by Lord Dalhousie, under which the adopted son could be the heir to his foster father’s private property, but not the state; it was for the paramount power (the British) to decide whether to bestow the state on the adopted son or to annex it. Question 17 of 35 17. Question Which of the statements are correct regarding the Ring-Fencing policy? It was meant to keep India safe from Napoleonic danger. The states brought under the system were assured of military assistance against external aggression at their own expense. The forces in the allied states were organized, equipped and commanded by the officers of the company. How many of the above statements are correct?  a) Only one b) Only two c) All three d) None Correct Solution (b) Statement 1 Statement 2 Statement 3 Incorrect Correct Correct Warren Hastings followed this policy to counter the powerful combination of the Marathas, Mysore and Hyderabad. He followed a policy of Ring-fence (1773-1785) which aimed at creating buffer zones to defend the company’s frontiers. It was the subsidiary alliance policy adopted by Wellesley to keep India safe from Napoleonic danger. The states brought under the ring-fence system were assured of military assistance against external aggression at their own expense. To safeguard against the dangers from afghan invaders and the Marathas, the East India company undertook to organize the defence of the frontiers. For example, to defend the Bengal, it required to defend the frontier of Awadh. Thus, in ring fence policy the Allies were required to maintain forces which were to be organized, equipped and commanded by the officers of the company who in turn were to be paid by the rulers of these states. Incorrect Solution (b) Statement 1 Statement 2 Statement 3 Incorrect Correct Correct Warren Hastings followed this policy to counter the powerful combination of the Marathas, Mysore and Hyderabad. He followed a policy of Ring-fence (1773-1785) which aimed at creating buffer zones to defend the company’s frontiers. It was the subsidiary alliance policy adopted by Wellesley to keep India safe from Napoleonic danger. The states brought under the ring-fence system were assured of military assistance against external aggression at their own expense. To safeguard against the dangers from afghan invaders and the Marathas, the East India company undertook to organize the defence of the frontiers. For example, to defend the Bengal, it required to defend the frontier of Awadh. Thus, in ring fence policy the Allies were required to maintain forces which were to be organized, equipped and commanded by the officers of the company who in turn were to be paid by the rulers of these states. Question 18 of 35 18. Question Consider the following pairs: Treaty                                      War Treaty of Yandabo                   First Anglo-Burmese War Treaty of Surat                        First Anglo-Maratha War Treaty of Lahore                      First Anglo-Sikh War How many of the pairs given above is/are correctly matched? a) Only one b) Only two c) All three d) None Correct Solution (b) Statement 1 Statement 2 Statement 3 Correct Incorrect Correct The Treaty of Yandabo was the peace treaty that ended the First Anglo-Burmese War. The treaty was signed in 1826, nearly two years after the war formally broke out. It was signed by General Sir Archibald Campbell on the British side, and the Governor of Legaing Maha Min Hla Kyaw Htin from the Burmese side, without any due permission and consent of the Ahom kingdom, Kachari kingdom or the other territories covered in the treaty. The Treaty of Salbai was signed in 1782, by representatives of the Maratha Empire and the British East India Company after long negotiations to settle the outcome of the First Anglo-Maratha War (1775-82). It was signed between Warren Hastings and Mahadaji Scindia. The Treaty of Surat was signed in 1775 between Raghunath Rao, a claimant to the throne of the Peshwa and the British East India Company at Bombay. The Treaty of Lahore, signed in 1846, was a peace treaty marking the end of the First Anglo-Sikh War (1845-46). The control of the rivers Sutlej and Beas and part of the Indus passed to the British, with the provison that this was not to interfere with the passage of passenger boats owned by the Lahore Government. Also, provision was made for the separate sale of all the hilly regions between River Beas and Indus, including Kashmir, by the East India Company at a later date to Gulab Singh, the Raja of Jammu.   Incorrect Solution (b) Statement 1 Statement 2 Statement 3 Correct Incorrect Correct The Treaty of Yandabo was the peace treaty that ended the First Anglo-Burmese War. The treaty was signed in 1826, nearly two years after the war formally broke out. It was signed by General Sir Archibald Campbell on the British side, and the Governor of Legaing Maha Min Hla Kyaw Htin from the Burmese side, without any due permission and consent of the Ahom kingdom, Kachari kingdom or the other territories covered in the treaty. The Treaty of Salbai was signed in 1782, by representatives of the Maratha Empire and the British East India Company after long negotiations to settle the outcome of the First Anglo-Maratha War (1775-82). It was signed between Warren Hastings and Mahadaji Scindia. The Treaty of Surat was signed in 1775 between Raghunath Rao, a claimant to the throne of the Peshwa and the British East India Company at Bombay. The Treaty of Lahore, signed in 1846, was a peace treaty marking the end of the First Anglo-Sikh War (1845-46). The control of the rivers Sutlej and Beas and part of the Indus passed to the British, with the provison that this was not to interfere with the passage of passenger boats owned by the Lahore Government. Also, provision was made for the separate sale of all the hilly regions between River Beas and Indus, including Kashmir, by the East India Company at a later date to Gulab Singh, the Raja of Jammu.   Question 19 of 35 19. Question With reference to Anglo-Mysore wars, consider the following statements: The Treaty of Seringapatam was concluded between Haidar Ali and the English. The Nizam of Hyderabad and the Marathas supported the state of Mysore in the four Anglo-Mysore wars. Both Arthur Wellesley and Lord Wellesley were involved in the fourth Anglo-Mysore war. How many of the above statements are correct?  a) Only one b) Only two c) All three d) None Correct Solution (a) Statement 1 Statement 2 Statement 3 Incorrect Incorrect Correct The Treaty of Seringapatam was signed between Tippu Sultan and the English in 1792. Under the treaty, nearly half of the Mysorean territory was taken over by the victors of the third Anglo-Mysore war- the state of Travancore, the British, the Nizam of Hyderabad and the Marathas. The Nizam of Hyderabad and the Marathas kept changing their strategies with time. During the first Anglo Mysore war, Haidar Ali turned the Nizam and the Marathas against the British through his skillful diplomacy. Later however, Marathas attacked Mysore in 1771. In the second and third Anglo-Mysore war, both Nizam and the British sided with the English forces against the state of Mysore. In 1798, Lord Wellesley succeeded Sir John Shore as the new Governor General of Bengal. He was concerned about the growing friendship between Tippu Sultan and the French. He accused Tippu Sultan of conspiracy against the British. The fourth Anglo-Mysore war that followed was also participated by Arthur Wellesley, the brother of Lord Wellesley. Hence, both Wellesley brothers were involved in the fourth Anglo-Mysore war. Incorrect Solution (a) Statement 1 Statement 2 Statement 3 Incorrect Incorrect Correct The Treaty of Seringapatam was signed between Tippu Sultan and the English in 1792. Under the treaty, nearly half of the Mysorean territory was taken over by the victors of the third Anglo-Mysore war- the state of Travancore, the British, the Nizam of Hyderabad and the Marathas. The Nizam of Hyderabad and the Marathas kept changing their strategies with time. During the first Anglo Mysore war, Haidar Ali turned the Nizam and the Marathas against the British through his skillful diplomacy. Later however, Marathas attacked Mysore in 1771. In the second and third Anglo-Mysore war, both Nizam and the British sided with the English forces against the state of Mysore. In 1798, Lord Wellesley succeeded Sir John Shore as the new Governor General of Bengal. He was concerned about the growing friendship between Tippu Sultan and the French. He accused Tippu Sultan of conspiracy against the British. The fourth Anglo-Mysore war that followed was also participated by Arthur Wellesley, the brother of Lord Wellesley. Hence, both Wellesley brothers were involved in the fourth Anglo-Mysore war. Question 20 of 35 20. Question Consider the following pairs: Land Revenue System                        Associated Areas Permanent Settlement              Bengal and Bihar Mahalwari System                      Madras and Bombay Ryotwari Settlement                   North-West Provinces and Punjab How many of the pairs given above is/are correctly matched? a) Only one b) Only two c) All three d) None Correct Solution (a) Statement 1 Statement 2 Statement 3 Correct Incorrect Incorrect Permanent Settlement: It was introduced in Bengal and Bihar in 1793 by Lord Cornwallis through the Permanent Settlement Act. Under the settlement, Zamindars were to give 10/11th of the rental they derived keeping only 1/11th for themselves and the sums to be paid were fixed in perpetuity. It was later extended to Orissa, The Northern districts of Madras, and the Districts of Varanasi.   Mahalwari system: It was a modified version of the zamindari settlement introduced in the Gangetic valley, the northwest Provinces, parts of central India, and Punjab. Cultivation of land was done on a co-sharing basis and the assessment was fixed for the entire village or estate. The responsibility of paying taxes lay with landlords or heads of families who collectively claimed to be the landlords of the village or the estate. The settlement was also periodically revised.   Ryotwari system: The British introduced a new form of revenue settlement in Madras, Bombay, and Berar regions called the Ryotwari system. Under the Ryotwari system, a direct tax contact was established between the ryot (the cultivator) and the state. The cultivator was to be recognized as the owner of his plot of land subject to the payment of land revenue. It was not permanent and was renegotiated periodically after 20 to 30 years.   Incorrect Solution (a) Statement 1 Statement 2 Statement 3 Correct Incorrect Incorrect Permanent Settlement: It was introduced in Bengal and Bihar in 1793 by Lord Cornwallis through the Permanent Settlement Act. Under the settlement, Zamindars were to give 10/11th of the rental they derived keeping only 1/11th for themselves and the sums to be paid were fixed in perpetuity. It was later extended to Orissa, The Northern districts of Madras, and the Districts of Varanasi.   Mahalwari system: It was a modified version of the zamindari settlement introduced in the Gangetic valley, the northwest Provinces, parts of central India, and Punjab. Cultivation of land was done on a co-sharing basis and the assessment was fixed for the entire village or estate. The responsibility of paying taxes lay with landlords or heads of families who collectively claimed to be the landlords of the village or the estate. The settlement was also periodically revised.   Ryotwari system: The British introduced a new form of revenue settlement in Madras, Bombay, and Berar regions called the Ryotwari system. Under the Ryotwari system, a direct tax contact was established between the ryot (the cultivator) and the state. The cultivator was to be recognized as the owner of his plot of land subject to the payment of land revenue. It was not permanent and was renegotiated periodically after 20 to 30 years.   Question 21 of 35 21. Question Consider the following statements regarding Sovereign Gold Bond (SGB) Scheme:  SGBs are issued by the Reserve Bank of India on behalf of the Government of India. Its objective is to mobilize the gold held by households and institutions in the country. The tenure of the SGB will be for a period of five years with an option of premature redemption after 3rd year. How many of the above statements are correct? a) Only one b) Only two c) All three d) None Correct Solution (b) SGBs are issued by the Reserve Bank of India on behalf of the Government of India. Hence statement 1 is correct. The main objective of the SGB Scheme is to mobilize the gold held by households and institutions in the country, reduce reliance on the import of gold over time to meet the domestic demand, and promote FDI in the gold and jewellery sector. Hence statement 2 is correct. The tenure of the SGB will befor a period of eight years with an option of premature redemption after 5th year. Hence statement 3 is incorrect. The minimum permissible investment will beone gram of gold and the maximum limit of subscription shall be 4 Kg for individuals, 4 Kg for HUF, and 20 Kg for trusts and similar entities per fiscal year (April-March) notified by the Government from time to time. SGBs will berestricted for sale to resident individuals, HUFs (Hindu Undivided Family), trusts, universities, and charitable institutions. Incorrect Solution (b) SGBs are issued by the Reserve Bank of India on behalf of the Government of India. Hence statement 1 is correct. The main objective of the SGB Scheme is to mobilize the gold held by households and institutions in the country, reduce reliance on the import of gold over time to meet the domestic demand, and promote FDI in the gold and jewellery sector. Hence statement 2 is correct. The tenure of the SGB will befor a period of eight years with an option of premature redemption after 5th year. Hence statement 3 is incorrect. The minimum permissible investment will beone gram of gold and the maximum limit of subscription shall be 4 Kg for individuals, 4 Kg for HUF, and 20 Kg for trusts and similar entities per fiscal year (April-March) notified by the Government from time to time. SGBs will berestricted for sale to resident individuals, HUFs (Hindu Undivided Family), trusts, universities, and charitable institutions. Question 22 of 35 22. Question Consider the following statements regarding the consent principle for the Central Bureau of Investigation (CBI): The consent of the state government given to the CBI that can be either case-specific or general. No states have withdrawn the general consent given to the CBI till today since 1946. Choose the correct code: a) 1 only b) 2 only c) Both 1 and 2 d) Neither 1 nor 2 Correct Solution (a) The Central Bureau of Investigation (CBI) was established by a resolution of the Ministry of Home Affairs and later transferred to the Ministry of Personnel, Public Grievances, and Pensions, currently functioning as an attached office. Its establishment was recommended by the Santhanam Committee on the Prevention of Corruption. The CBI operates under the Delhi Special Police Establishment (DSPE) Act of 1946. It is neither a constitutional nor a statutory body. It investigates cases related to bribery, governmental corruption, breaches of central laws, multi-state organized crime, and multi-agency or international cases. The consent of the state government to the CBI can be either case-specific or general. Hence statement 1 is correct. General consent is normally given by states to help the CBI in the seamless investigation of cases of corruption against central government employees in their states. This is essentially consent by default, which means CBI may begin investigations taking consent as having been already given. In the absence of general consent, CBI would have to apply to the state government for its consent in every individual case, and before taking even small actions. Mizoram, West Bengal, Chhattisgarh, Rajasthan, Maharashtra, Kerala, Jharkhand, Punjab, Tamil Nadu, and Meghalaya have withdrawn their general consent given to the CBI. The states can withdraw the general consent given to the Central Bureau of Investigation (CBI), under Section 6 of the Delhi Special Police Establishment (DSPE) Act, 1946. Hence statement 2 is incorrect. Incorrect Solution (a) The Central Bureau of Investigation (CBI) was established by a resolution of the Ministry of Home Affairs and later transferred to the Ministry of Personnel, Public Grievances, and Pensions, currently functioning as an attached office. Its establishment was recommended by the Santhanam Committee on the Prevention of Corruption. The CBI operates under the Delhi Special Police Establishment (DSPE) Act of 1946. It is neither a constitutional nor a statutory body. It investigates cases related to bribery, governmental corruption, breaches of central laws, multi-state organized crime, and multi-agency or international cases. The consent of the state government to the CBI can be either case-specific or general. Hence statement 1 is correct. General consent is normally given by states to help the CBI in the seamless investigation of cases of corruption against central government employees in their states. This is essentially consent by default, which means CBI may begin investigations taking consent as having been already given. In the absence of general consent, CBI would have to apply to the state government for its consent in every individual case, and before taking even small actions. Mizoram, West Bengal, Chhattisgarh, Rajasthan, Maharashtra, Kerala, Jharkhand, Punjab, Tamil Nadu, and Meghalaya have withdrawn their general consent given to the CBI. The states can withdraw the general consent given to the Central Bureau of Investigation (CBI), under Section 6 of the Delhi Special Police Establishment (DSPE) Act, 1946. Hence statement 2 is incorrect. Question 23 of 35 23. Question Consider the following statements about Cargo Release Time: It is the time taken from the arrival of the cargo at the customs station to its out-of-charge. It is a key indicator of trade efficiency and ease of doing business. It is a performance measurement tool recommended by the World Trade Organisation. How many of the above statements are correct? a) Only one b) Only two c) All three d) None Correct Solution (b) Cargo release time is defined as the time taken from the arrival of the cargo at the customs station to its out-of-charge. Hence statement 1 is correct. For domestic clearance in case of imports and arrival of the cargo at the customs station to the eventual departure of the carrier in case of exports. Cargo release time is a key indicator of trade efficiency and ease of doing business, as it reflects the effectiveness of customs procedures and other regulatory processes involved in cross-border trade. Hence statement 2 is correct. Cargo release time is measured using Time Release Study (TRS), a performance measurement tool recommended by the World Customs Organization (WCO). Hence statement 3 is incorrect.   Note: The World Customs Organization (WCO) is an independent intergovernmental body whose mission is to enhance the effectiveness and efficiency of customs administrations. It is the only international organization with competence in customs matters which can rightly call itself the voice of the international customs community. It has its headquarters in Brussels, Belgium. Incorrect Solution (b) Cargo release time is defined as the time taken from the arrival of the cargo at the customs station to its out-of-charge. Hence statement 1 is correct. For domestic clearance in case of imports and arrival of the cargo at the customs station to the eventual departure of the carrier in case of exports. Cargo release time is a key indicator of trade efficiency and ease of doing business, as it reflects the effectiveness of customs procedures and other regulatory processes involved in cross-border trade. Hence statement 2 is correct. Cargo release time is measured using Time Release Study (TRS), a performance measurement tool recommended by the World Customs Organization (WCO). Hence statement 3 is incorrect.   Note: The World Customs Organization (WCO) is an independent intergovernmental body whose mission is to enhance the effectiveness and efficiency of customs administrations. It is the only international organization with competence in customs matters which can rightly call itself the voice of the international customs community. It has its headquarters in Brussels, Belgium. Question 24 of 35 24. Question Consider the following statements regarding Saksham Anganwadi and Poshan 2.0: It is a centrally sponsored scheme implemented by states and union territories. It seeks to address the challenges of malnutrition in children, adolescent girls, pregnant women, and lactating mothers. It is being implemented by the Ministry of Health and Family Welfare. How many of the above statements are correct? a) Only one b) Only two c) All three d) None Correct Solution (b) Saksham Anganwadi and Poshan 2.0 is a centrally sponsored scheme implemented by states and union territories. Hence statement 1 is correct. It seeks to address the challenges of malnutrition in children, adolescent girls, pregnant women, and lactating mothers through a strategic shift in nutrition content and delivery and by the creation of a convergent ecosystem to develop and promote practices that nurture health, wellness, and immunity. Hence statement 2 is correct. It provides a package of six services, namely, supplementary nutrition, pre-school non-formal education, nutrition and health education, immunization, health check-up, and referral services. It provides services to all eligible beneficiaries, namely, children in the age group of 0-6 years, pregnant women, and lactating mothers through the platform of Anganwadi Centres across the country. It is being implemented by the Ministry of Women and Child Development. Hence statement 3 is incorrect. Incorrect Solution (b) Saksham Anganwadi and Poshan 2.0 is a centrally sponsored scheme implemented by states and union territories. Hence statement 1 is correct. It seeks to address the challenges of malnutrition in children, adolescent girls, pregnant women, and lactating mothers through a strategic shift in nutrition content and delivery and by the creation of a convergent ecosystem to develop and promote practices that nurture health, wellness, and immunity. Hence statement 2 is correct. It provides a package of six services, namely, supplementary nutrition, pre-school non-formal education, nutrition and health education, immunization, health check-up, and referral services. It provides services to all eligible beneficiaries, namely, children in the age group of 0-6 years, pregnant women, and lactating mothers through the platform of Anganwadi Centres across the country. It is being implemented by the Ministry of Women and Child Development. Hence statement 3 is incorrect. Question 25 of 35 25. Question Consider the following statements about the characteristics of Mesolithic Rock Paintings: They mainly saw the use of red colour. The hunting scenes were predominant in the period. Animals depicted include elephants, bison, tiger, and boar. How many of the above statements are correct? a) Only one b) Only two c) All three d) None Correct Solution (c) The characteristics of Mesolithic Rock Paintings: This period mainly saw the use of red colour. Though Hence statement 1 is correct. It has a larger variety of themes but the paintings are smaller in size. The hunting scenes were predominant in the period. Hence statement 2 is correct. The paintings depict people hunting in groups and hunting armed with barbed spears, pointed sticks, arrows, and bows. Animals depicted include elephants, bison, tiger, boar, deer, antelope, leopard, panther, rhinoceros, fish, frog, lizard, squirrel, and at times birds. Hence statement 3 is correct. Animals were painted in a naturalistic style and humans were depicted in a stylistic manner. The young, old, children, and women equally find a place in these paintings. Women are painted both nude and clothed. In many of the rock shelters handprints, fist prints, and dots made by the fingertips are found. One of the most prominent examples of the Mesolithic rock painting is found in Bhimbetka, Madhya Pradesh. Incorrect Solution (c) The characteristics of Mesolithic Rock Paintings: This period mainly saw the use of red colour. Though Hence statement 1 is correct. It has a larger variety of themes but the paintings are smaller in size. The hunting scenes were predominant in the period. Hence statement 2 is correct. The paintings depict people hunting in groups and hunting armed with barbed spears, pointed sticks, arrows, and bows. Animals depicted include elephants, bison, tiger, boar, deer, antelope, leopard, panther, rhinoceros, fish, frog, lizard, squirrel, and at times birds. Hence statement 3 is correct. Animals were painted in a naturalistic style and humans were depicted in a stylistic manner. The young, old, children, and women equally find a place in these paintings. Women are painted both nude and clothed. In many of the rock shelters handprints, fist prints, and dots made by the fingertips are found. One of the most prominent examples of the Mesolithic rock painting is found in Bhimbetka, Madhya Pradesh. Question 26 of 35 26. Question The Canary Islands are located in? a) Italy b) France c) Germany d) Spain Correct Solution (d) The Canary Islands-a Spanish archipelago off the coast of northwestern Africa. It is part of Spain. It is about 1300 km South of mainland Spain and 115 km West of the African coast (Morocco). Hence option d is correct. It is located in the Atlantic Ocean. It is an archipelago. These Islands were formed by volcanic eruptions millions of years ago. Incorrect Solution (d) The Canary Islands-a Spanish archipelago off the coast of northwestern Africa. It is part of Spain. It is about 1300 km South of mainland Spain and 115 km West of the African coast (Morocco). Hence option d is correct. It is located in the Atlantic Ocean. It is an archipelago. These Islands were formed by volcanic eruptions millions of years ago. Question 27 of 35 27. Question Consider the following statements about the ‘United Nations Educational, Scientific and Cultural Organisation’ (UNESCO): India is a founding member of UNESCO. India got re-elected to UNESCO executive board for 2021-25 term. United States of America is not a member of UNCESO. How many of the above statements are correct? a) Only one b) Only two c) All three d) None Correct Solution (b) It is a specialised agency of the United Nations (UN). UN: is an international organization founded in 1945. It works for maintaining international peace and security, protect human rights, deliver humanitarian aid, promote sustainable development and uphold international law. Objective: promoting world peace and security through international cooperation in education, the sciences, and culture. HQ: Paris, France. Members:- It has 194 member states and 12 associate members, as well as partners in the nongovernmental, intergovernmental, and private sectors. Recently USA rejoined UNESCO. Hence statement 3 is not correct. UN member states not UNESCO members: Israel, Liechtenstein. Issue of USA Leaving UNESCO In 2011, UNESCO inducted Palestine as a member. This led to the US halting the agency’s funding, worth millions of dollars, under then-President Barack Obama. Reason to stop funding UNESCO:- Israel and the United States termed the inclusion of Palestine and UNESCO’s naming of what Israel claims were ancient Jewish sites as Palestinian heritage sites as examples of anti–Israel bias. US laws, owing to the country’s historical ties with Israel, prohibit funding to any UN agency that implies recognition of the Palestinians’ demands for their own state. Consequently, in 2019 the US and Israel pulled out of UNESCO citing bias in the organisation in the Palestine issue. The US had pulled out of UNESCO once earlier in 1984 and then rejoined in 2003. Recent Developments: it was negotiated recently through an agreement in 2022 that the USA will begin giving UNESCO funds again. Issue of Palestine:- The Palestinians claim the West Bank, east Jerusalem and Gaza Strip territories captured by Israel in the 1967 war for an independent state. Israel disagrees with Palestine’s claims. Palestine is not recognised as a sovereign state by the United Nations. India and UNESCO India has been a founding member of UNESCO. Hence statement 1 is correct It had ratified UNESCO’s Constitution in 1946, while still under colonial rule. India has been continuously re-elected to the UNESCO Executive Board since 1946. Recently, India won the re-election to the executive board of the UN’s cultural and education organisation for the 2021-25 term. Hence statement 2 is correct. Incorrect Solution (b) It is a specialised agency of the United Nations (UN). UN: is an international organization founded in 1945. It works for maintaining international peace and security, protect human rights, deliver humanitarian aid, promote sustainable development and uphold international law. Objective: promoting world peace and security through international cooperation in education, the sciences, and culture. HQ: Paris, France. Members:- It has 194 member states and 12 associate members, as well as partners in the nongovernmental, intergovernmental, and private sectors. Recently USA rejoined UNESCO. Hence statement 3 is not correct. UN member states not UNESCO members: Israel, Liechtenstein. Issue of USA Leaving UNESCO In 2011, UNESCO inducted Palestine as a member. This led to the US halting the agency’s funding, worth millions of dollars, under then-President Barack Obama. Reason to stop funding UNESCO:- Israel and the United States termed the inclusion of Palestine and UNESCO’s naming of what Israel claims were ancient Jewish sites as Palestinian heritage sites as examples of anti–Israel bias. US laws, owing to the country’s historical ties with Israel, prohibit funding to any UN agency that implies recognition of the Palestinians’ demands for their own state. Consequently, in 2019 the US and Israel pulled out of UNESCO citing bias in the organisation in the Palestine issue. The US had pulled out of UNESCO once earlier in 1984 and then rejoined in 2003. Recent Developments: it was negotiated recently through an agreement in 2022 that the USA will begin giving UNESCO funds again. Issue of Palestine:- The Palestinians claim the West Bank, east Jerusalem and Gaza Strip territories captured by Israel in the 1967 war for an independent state. Israel disagrees with Palestine’s claims. Palestine is not recognised as a sovereign state by the United Nations. India and UNESCO India has been a founding member of UNESCO. Hence statement 1 is correct It had ratified UNESCO’s Constitution in 1946, while still under colonial rule. India has been continuously re-elected to the UNESCO Executive Board since 1946. Recently, India won the re-election to the executive board of the UN’s cultural and education organisation for the 2021-25 term. Hence statement 2 is correct. Question 28 of 35 28. Question Consider the following statements about the composition of the National Human Rights Commission (NHRC): It is a multi-member body consisting of a chairperson, five full-time members, and seven deemed members. They serve for five years or until they attain the age of 70 years, whichever is earlier. They can be removed only on the charges of proven misbehavior or incapacity. How many of the above statements are correct? a) Only one b) Only two c) Only three d) None Correct Solution (b) The National Human Rights Commission (NHRC) is a multi-member body consisting of a chairperson, five full-time members, and seven deemed members. Hence statement 1 is correct. Chairperson will be a retired chief justice of India or a judge of the Supreme Court. The chairman and members are appointed by the President on the recommendations of a six-member committee consisting of:- Prime Minister (head) Speaker of the Lok Sabha Deputy Chairman of the Rajya Sabha Leaders of the Opposition in both Houses of Parliament Union Home Minister. They serve for three yearsor until they attain the age of 70 years, whichever is earlier. Hence statement 2 is incorrect. The President can remove them from office under specific circumstances. They can be removed only on the charges of proven misbehavior or incapacity if proved by an inquiry conducted by a Supreme Court Judge. Hence statement 3 is correct. Incorrect Solution (b) The National Human Rights Commission (NHRC) is a multi-member body consisting of a chairperson, five full-time members, and seven deemed members. Hence statement 1 is correct. Chairperson will be a retired chief justice of India or a judge of the Supreme Court. The chairman and members are appointed by the President on the recommendations of a six-member committee consisting of:- Prime Minister (head) Speaker of the Lok Sabha Deputy Chairman of the Rajya Sabha Leaders of the Opposition in both Houses of Parliament Union Home Minister. They serve for three yearsor until they attain the age of 70 years, whichever is earlier. Hence statement 2 is incorrect. The President can remove them from office under specific circumstances. They can be removed only on the charges of proven misbehavior or incapacity if proved by an inquiry conducted by a Supreme Court Judge. Hence statement 3 is correct. Question 29 of 35 29. Question Consider the following statements about the Food Corporation of India (FCI): It is a statutory body under the Food Corporation Act of 1964. It works under the Ministry of Consumer Affairs, Food, and Public Distribution. Choose the correct code: a) 1 only b) 2 only c) Both 1 and 2 d) Neither 1 nor 2 Correct Solution (c) The Food Corporation of India is a statutory body under the Food Corporation Act of 1964. Hence statement 1 is correct. It works under the Ministry of Consumer Affairs, Food, and Public Distribution. Hence statement 2 is correct. It is headquartered in New Delhi. Objectives of FCI: Effective price support operations for safeguarding the interests of the farmers. Distribution of food grains throughout the country for the public distribution system. Maintaining a satisfactory level of operational and buffer stocks of foodgrains to ensure National Food Security. Effective Price Support Operations for safeguarding the interest of farmers.   Incorrect Solution (c) The Food Corporation of India is a statutory body under the Food Corporation Act of 1964. Hence statement 1 is correct. It works under the Ministry of Consumer Affairs, Food, and Public Distribution. Hence statement 2 is correct. It is headquartered in New Delhi. Objectives of FCI: Effective price support operations for safeguarding the interests of the farmers. Distribution of food grains throughout the country for the public distribution system. Maintaining a satisfactory level of operational and buffer stocks of foodgrains to ensure National Food Security. Effective Price Support Operations for safeguarding the interest of farmers.   Question 30 of 35 30. Question Consider the following statements: Debrigarh Wildlife Sanctuary is located in Assam. Hirakud Dam is on the Mahanadi River. Simlipal is the largest national park in Odisha. How many of the above statements are correct? a) Only one b) Only two c) All three d) None Correct Solution (b) The Debrigarh wildlife sanctuary is located in the Bargarh district in the Indian state of Odisha, covering a total area of 346.91 km2. It is situated near the city of Sambalpur’s Hirakud Dam. Hence statement 1 is incorrect. It was declared a wildlife sanctuary in 1985. It has dry deciduous forests. Hirakud Dam is on the Mahanadi River. Hence statement 2 is correct.    Note: Major Protected Areas in Odisha National Parks: Bhitarkanika National Park Simlipal National Park-Simlipal is the largest national park in Odisha and is also considered as one of the principal tiger projects in India. Hence statement 3 is correct   Incorrect Solution (b) The Debrigarh wildlife sanctuary is located in the Bargarh district in the Indian state of Odisha, covering a total area of 346.91 km2. It is situated near the city of Sambalpur’s Hirakud Dam. Hence statement 1 is incorrect. It was declared a wildlife sanctuary in 1985. It has dry deciduous forests. Hirakud Dam is on the Mahanadi River. Hence statement 2 is correct.    Note: Major Protected Areas in Odisha National Parks: Bhitarkanika National Park Simlipal National Park-Simlipal is the largest national park in Odisha and is also considered as one of the principal tiger projects in India. Hence statement 3 is correct   Question 31 of 35 31. Question Eight friends P, Q, R, S, T, U, V and W are sitting around a circular table facing the centre. V sits second to the right of P, whose third to the left is T. R sits third to the left of Q, and S sits immediately between V and R. U and V cannot sit opposite each other. Three of the following four are similar in a certain way based on their positions in the seating arrangement and so form a group. Which of the following does not belong to that group? a) RW b) TQ c) SU d) VR Correct Solution (c) According to the data given, the arrangement of 8 friends will be like this Venn diagram By analyzing the combinations, We find that RW, TQ, VR are in clockwise direction with one person in between SU is in anti-clockwise direction, so it is the odd one out. Incorrect Solution (c) According to the data given, the arrangement of 8 friends will be like this Venn diagram By analyzing the combinations, We find that RW, TQ, VR are in clockwise direction with one person in between SU is in anti-clockwise direction, so it is the odd one out. Question 32 of 35 32. Question In a class of 120 students, 62 like English, 52 like Mathematics and 24 like both English and Mathematics. What percentage of students in the class do not like both the subjects?   a) 30% b) 27% c) 25% d) 21 % Correct Solution (c) Total Students in a class = 120 Number of students who like English = 62 Number of students who like Maths = 52 Number of students who like both English and Maths = 24 Number of students who like neither English nor Maths = 120 – (62 + 52 – 24) = 120 – 90 = 30 Percentage of students who do not like both English and Maths = (30/120) × 100 = 1/4 = 25% Hence, option (c) is the correct answer. Incorrect Solution (c) Total Students in a class = 120 Number of students who like English = 62 Number of students who like Maths = 52 Number of students who like both English and Maths = 24 Number of students who like neither English nor Maths = 120 – (62 + 52 – 24) = 120 – 90 = 30 Percentage of students who do not like both English and Maths = (30/120) × 100 = 1/4 = 25% Hence, option (c) is the correct answer. Question 33 of 35 33. Question Consider a 3-digit integer x with distinct digits. Let y be the integer formed by swapping x’s units and hundreds digits. What is the greatest prime number less than 30 that perfectly divides the absolute difference between x and y? a) 7 b) 11 c) 13 d) 17 Correct Solution (b) Let 3-digit integer x with distinct digits.   = abc 100a + 10b + c y be the integer formed by swapping x’s units and hundreds digits. => cba 100c + 10b + a Difference  =  100a + 10b + c – (100c + 10b + a) = 99a  – 99c = 99(a – c) = 3 * 3 *11 (a – c) 11  is the greatest prime number that perfectly divides the absolute difference between x and y Incorrect Solution (b) Let 3-digit integer x with distinct digits.   = abc 100a + 10b + c y be the integer formed by swapping x’s units and hundreds digits. => cba 100c + 10b + a Difference  =  100a + 10b + c – (100c + 10b + a) = 99a  – 99c = 99(a – c) = 3 * 3 *11 (a – c) 11  is the greatest prime number that perfectly divides the absolute difference between x and y Question 34 of 35 34. Question In the English alphabet, the first 4 letters are written in opposite order; and the next 4 letters are written in opposite order and so on; and at the end Y and Z are interchanged. Which will be the fourth letter to the right of the 13th letter ?   a) N b) T c) H d) I Correct Solution (b) English alphabets are: ABCD EFGH IJKL MNOP QRST UVWX YZ As per the question, the alphabets are arranged as follows: DCBA HGFE LKJI PONM TSRQ XWVU ZY 13TH letter is P. And the fourth letter right to P is T. Incorrect Solution (b) English alphabets are: ABCD EFGH IJKL MNOP QRST UVWX YZ As per the question, the alphabets are arranged as follows: DCBA HGFE LKJI PONM TSRQ XWVU ZY 13TH letter is P. And the fourth letter right to P is T. Question 35 of 35 35. Question Abhishek was conducting an experiment in which the average of 11 observations came to be 90, while the average of first five observations was 87, and that of the last five was 84. What was the measure of 6th observation?   a) 165 b) 150 c) 145 d) 135 Correct Solution (d) Explanation Let the 6th observation be ‘x’ Average of first five is 87 and last five is 84 First five observation total = 87 x 5 = 435 Last five observation total = 84 x 5 = 420 Sum total of 11 observation = 11 x 90 = 990 ∴ 435 + 420 + x = 990 => x = 990 – 435 – 420 = 135 Incorrect Solution (d) Explanation Let the 6th observation be ‘x’ Average of first five is 87 and last five is 84 First five observation total = 87 x 5 = 435 Last five observation total = 84 x 5 = 420 Sum total of 11 observation = 11 x 90 = 990 ∴ 435 + 420 + x = 990 => x = 990 – 435 – 420 = 135 window.wpProQuizInitList = window.wpProQuizInitList || []; window.wpProQuizInitList.push({ id: '#wpProQuiz_3505', init: { quizId: 3505, mode: 1, globalPoints: 70, timelimit: 1800, resultsGrade: [0], bo: 704, qpp: 0, catPoints: [70], formPos: 0, lbn: "Test-summary", json: {"29557":{"type":"single","id":29557,"catId":0,"points":2,"correct":[0,0,1,0]},"29558":{"type":"single","id":29558,"catId":0,"points":2,"correct":[0,0,0,1]},"29560":{"type":"single","id":29560,"catId":0,"points":2,"correct":[0,1,0,0]},"29561":{"type":"single","id":29561,"catId":0,"points":2,"correct":[1,0,0,0]},"29562":{"type":"single","id":29562,"catId":0,"points":2,"correct":[0,0,1,0]},"29563":{"type":"single","id":29563,"catId":0,"points":2,"correct":[0,0,1,0]},"29564":{"type":"single","id":29564,"catId":0,"points":2,"correct":[0,0,1,0]},"29567":{"type":"single","id":29567,"catId":0,"points":2,"correct":[0,1,0,0]},"29570":{"type":"single","id":29570,"catId":0,"points":2,"correct":[0,1,0,0]},"29572":{"type":"single","id":29572,"catId":0,"points":2,"correct":[0,1,0,0]},"29575":{"type":"single","id":29575,"catId":0,"points":2,"correct":[0,0,1,0]},"29576":{"type":"single","id":29576,"catId":0,"points":2,"correct":[0,0,1,0]},"29577":{"type":"single","id":29577,"catId":0,"points":2,"correct":[0,0,1,0]},"29579":{"type":"single","id":29579,"catId":0,"points":2,"correct":[0,1,0,0]},"29580":{"type":"single","id":29580,"catId":0,"points":2,"correct":[0,1,0,0]},"29582":{"type":"single","id":29582,"catId":0,"points":2,"correct":[0,0,1,0]},"29584":{"type":"single","id":29584,"catId":0,"points":2,"correct":[0,1,0,0]},"29585":{"type":"single","id":29585,"catId":0,"points":2,"correct":[0,1,0,0]},"29586":{"type":"single","id":29586,"catId":0,"points":2,"correct":[1,0,0,0]},"29588":{"type":"single","id":29588,"catId":0,"points":2,"correct":[1,0,0,0]},"29590":{"type":"single","id":29590,"catId":0,"points":2,"correct":[0,1,0,0]},"29593":{"type":"single","id":29593,"catId":0,"points":2,"correct":[1,0,0,0]},"29596":{"type":"single","id":29596,"catId":0,"points":2,"correct":[0,1,0,0]},"29599":{"type":"single","id":29599,"catId":0,"points":2,"correct":[0,1,0,0]},"29602":{"type":"single","id":29602,"catId":0,"points":2,"correct":[0,0,1,0]},"29603":{"type":"single","id":29603,"catId":0,"points":2,"correct":[0,0,0,1]},"29605":{"type":"single","id":29605,"catId":0,"points":2,"correct":[0,1,0,0]},"29608":{"type":"single","id":29608,"catId":0,"points":2,"correct":[0,1,0,0]},"29610":{"type":"single","id":29610,"catId":0,"points":2,"correct":[0,0,1,0]},"29613":{"type":"single","id":29613,"catId":0,"points":2,"correct":[0,1,0,0]},"29616":{"type":"single","id":29616,"catId":0,"points":2,"correct":[0,0,1,0]},"29619":{"type":"single","id":29619,"catId":0,"points":2,"correct":[0,0,1,0]},"29622":{"type":"single","id":29622,"catId":0,"points":2,"correct":[0,1,0,0]},"29624":{"type":"single","id":29624,"catId":0,"points":2,"correct":[0,1,0,0]},"29625":{"type":"single","id":29625,"catId":0,"points":2,"correct":[0,0,0,1]}} } }); All the Best IASbaba

DAILY CURRENT AFFAIRS IAS | UPSC Prelims and Mains Exam – 5th March 2024

Archives (PRELIMS & MAINS Focus)   Prototype Fast Breeder Reactor Syllabus Prelims and Mains – Science Context: In a historic milestone marking entry into the vital second stage of India’s three stage nuclear program, Prime Minister, Shri Narendra Modi witnessed , commencement of “Core Loading” at India’s first indigenous Fast Breeder Reactor (500 MWe) at Kalpakkam, Tamil Nadu. Background:- The government had approved in 2003, the creation of Bharatiya Nabhikiya Vidyut Nigam Ltd (BHAVINI) to construct and operate India’s most advanced nuclear reactor-Prototype Fast Breeder Reactor (PFBR). About PFBR and Indias Three stage nuclear program The PFBR is a machine that produces more nuclear fuel than it consumes. Its core-loading event is being hailed as a “milestone” because the operationalisation of the PFBR will mark the start of stage II of India’s three-stage nuclear power programme. In the first stage, India used pressurised heavy water reactors (PHWRs) and natural uranium-238 (U-238), which contains minuscule amounts of U-235, as the fissile material. In nuclear fission, the nucleus of an atom absorbs a neutron, destabilises, and breaks into two while releasing some energy. If the destabilised nucleus releases more neutrons, the reactor’s facilities will attempt to use them to instigate more fission reactions. The heavy water in PHWR – water molecules containing the deuterium isotope of hydrogen – slows neutrons released by one fission reaction enough to be captured by other U-238 and U-235 nuclei and cause new fission. The heavy water is pressurised to keep it from boiling. The reactions produce plutonium-239 (Pu-239) and energy. Only U-235, not U-238, can sustain a chain reaction but it is consumed fully in stage I. In stage II, India will use Pu-239 together with U-238 in the PFBR to produce energy, U-233, and more Pu-239. In stage III, Pu-239 will be combined with thorium-232 (Th-232) in reactors to produce energy and U-233. Homi J. Bhabha designed the three-stage programme because India hosts roughly a quarter of the world’s thorium. The three stages are expected to allow the country complete self-sufficiency in nuclear energy. How does the PFBR work? PHWRs use natural or low-enriched U-238 as the fissile material and produce Pu-239 as a byproduct. This Pu-239 is combined with more U-238 into a mixed oxide and loaded into the core of a new reactor together with a blanket. This is a material the fission products in the core react with to produce more Pu-239. A breeder reactor is a nuclear reactor that produces more fissile material than it consumes. In a ‘fast’ breeder reactor, the neutrons aren’t slowed, allowing them to trigger specific fission reactions. The PFBR is designed to produce more Pu-239 than it consumes. It uses liquid sodium, a highly reactive substance, as coolant in two circuits. Coolant in the first circuit enters the reactor and leaves with (heat) energy and radioactivity. Via heat-exchangers, it transfers only the heat to the coolant in a secondary circuit. The latter transfers the heat to generators to produce electricity. Source: The Hindu SIMILIPAL TIGER RESERVE(STR) Syllabus Prelims -Environment Context: The Odisha government has sought the National Tiger Conservation Authority’s (NTCA) nod to introduce some female tigers to the Similipal Tiger Reserve. Background: Odisha wants to bring in female big cats from the Central Indian landscape, which includes areas like Madhya Pradesh, Rajasthan and Maharashtra to improve the tigers’ gene pool and improve the sex ratio. About Similipal Tiger Reserve: Similipal Tiger Reserve is a national park and tiger reserve located in the Mayurbhanj district of Odisha, India. It is part of the Mayurbhanj Elephant Reserve. The reserve derives its name from the abundance of red silk cotton trees, known as ‘Simul’ in the local language. It is home to a diverse range of flora and fauna, including the Bengal tiger, Asian elephant, gaur, and chausingha. Notably, it is Asia’s second-largest biosphere and the country’s only wild habitat for melanistic royal Bengal tigers. The tigers of Similipal belong to a unique lineage with higher-than-normal levels of melanin, which gives them black-and-yellow-interspersed stripes on their coats. These tigers are not entirely black, and are therefore more accurately described as pseudo-melanistic. The reserve features two impressive waterfalls: the Barehipani Falls and the Joranda Falls. Similipal was declared a wildlife sanctuary in 1979 and a biosphere reserve in 1994. It has been part of the UNESCO World Network of Biosphere Reserves since 2009. The region around STR is home to a variety of tribes including Kolha, Santhala, Bhumija, Bhatudi, Gondas, Khadia, Mankadia and Sahara. Tiger Relocation Process The National Tiger Conservation Authority (NTCA) ’s approval is mandatory for any tiger relocation project within India. Before relocating tigers, the state government must seek permission from the NTCA. In the case of Similipal Tiger Reserve, the NTCA technical committee is expected to visit the reserve soon to study its landscape, climate, whether it has the required prey base and other aspects before giving its go-ahead. Source: Indian Express Previous Year Question Q1. Consider the following protected areas: Bandipur Bhitarkanika Manas Sunderbans Which of the above are declared Tiger Reserves? 1 and 2 only 1, 3 and 4 only 2, 3 and 4 only 1, 2, 3 and 4 NATIONAL URBAN COOPERATIVE FINANCE AND DEVELOPMENT CORPORATION LIMITED (NUCFDC) Syllabus Prelims – Current Event Context: Recently, Shri Amit Shah inaugurated the National Urban Cooperative Finance and Development Corporation Limited (NUCFDC). Background: The minister suggested the Board of the NUCFDC to set up one urban cooperative bank in each unrepresented town/city in a time-bound program. About NATIONAL URBAN COOPERATIVE FINANCE AND DEVELOPMENT CORPORATION LIMITED (NUCFDC) : The National Urban Cooperative Finance and Development Corporation Limited (NUCFDC) is an umbrella organization for Urban Cooperative Banks (UCBs) in India. Its primary role is to modernize and strengthen the Urban Cooperative Banking Sector in India. The National Urban Cooperative Finance and Development Corporation Limited (NUCFDC) has received Certificate of Registration (CoR) from the Reserve Bank of India (RBI) to operate as a Non-Banking Finance Company (NBFC). Additionally, the NUCFDC is allowed to function as a Self-Regulatory Organization (SRO) for the sector. Key Objectives: Facilitate financial assistance and support to UCBs. Promote cooperative institutions in urban areas. Enhance financial inclusion and accessibility for citizens. Benefits and Services: The NUCFDC aims to provide UCBs with facilities such as ATMs, credit/debit cards, clearing systems, SLR limits, and refinancing. It contributes to the overall development and well-being of cooperative banks and their customers.  Source: PIB NATIONAL COMMISSION FOR PROTECTION OF CHILD RIGHTS (NCPCR) Syllabus Prelims -Current Event Context: Recently, National Commission for Protection of Child Rights (NCPCR) has filed a complaint with the Ministry of Electronics and Information Technology against an App. Background: The commission has requested MeitY to take action against Google and Apple and also urged the ministry to mandate stringent KYC norms for users accessing Ullu and similar apps on the marketplace About National Commission for Protection of Child Rights (NCPCR) National Commission for Protection of Child Rights (NCPCR) is a statutory body established by the Government of India under the Commission for Protection of Child Rights (CPCR) Act, 2005. NCPCR is established to safeguard the rights of children aged 0 to 18 years. Its mission is to ensure that all laws, policies, and programs align with the vision of child rights. It works towards promoting a child-centric approach in all regulations, guidelines, and executive actions in India. NCPCR operates under the Ministry of Women & Child Development of the Central Government. The Commission is further mandated to monitor the proper and effective implementation of Protection of Children from Sexual Offences (POCSO) Act, 2012, Juvenile Justice (Care and Protection of Children) Act, 2015, and Right to Free and Compulsory Education (RTE) Act, 2009. Source: inc42 Bannerghatta National Park Syllabus Prelims – Environment Context: Days after the clearance from the National Board of Wildlife (NBWL), the National Highway Authority of India (NHAI) has sought to clear the path for building a six-lane elevated highway inside the Bannerghatta National Park (BNP). Background: Faced with very high man-animal conflict and encroachments, BNP is already reeling under severe threats of fragmentation About Bannerghatta National Park: Bannerghatta National Park is located near Bangalore, Karnataka. It was declared as a national park in 1974. In 2002, a small portion of the park became a zoological garden, the Bannerghatta Biological Park Between 1995 and 2021, the dry deciduous forests in the National Park shrunk approximately by 44% due to uncontrolled diversion of forest land for agricultural purpose, urbanisation, etc. Other national parks of Karnataka are Bandipur National Park, Anshi National Park, Kudremukh National Park, Nagarhole National Park. Source: The Hindu Nano DAP Syllabus Prelims & Mains – GS 3 Context: The finance minister in the Interim Budget 2024-25 has announced the expansion of the application of Nano DAP (Di-Ammonium Phosphate) as a fertilizer on various crops in all agro-climatic zones. Background: If Russia were to develop and deploy such a weapon, it would be in violation of Outer Space Treaty as well as the Partial Nuclear Test Ban Treaty of 1963 that prohibits nuclear explosions in space. Russia is party to both treaties. About DAP DAP is the second most commonly used fertilizer in India after urea. DAP is a preferred fertilizer in India because it contains both nitrogen and phosphorus which are primary macro-nutrients and part of 18 essential plant nutrients. Fertilizer grade DAP contains 18% nitrogen and 46% phosphorus. It is manufactured by reacting ammonia with phosphoric acid under controlled conditions in fertilizer plants. What is Nano DAP? Nano DAP is a specialised form of DAP designed to improve the fertiliser’s effectiveness in promoting plant growth and development. In 2023 Indian Farmers Fertiliser Cooperative (IFFCO) launched its Nano DAP, containing 8% Nitrogen and 16% Phosphorus by volume. Unlike conventional DAP, which comes in granular form, IFFCO’s Nano DAP is in liquid form. Source: PIB BIOTRIG Syllabus Prelims & Mains – Environment and Science Context: A recent study introduces a waste management technology called ‘BioTRIG,’ utilizing pyrolysis at the community level in rural India. Background: The BioTRIG represents a sustainable solution that can improve the quality of life for rural communities, especially those living below the poverty line. About BioTrig: BioTRIG is a community-level pyrolysis system designed to run on the waste generated by villagers. Pyrolysis is a chemical recycling process that transforms leftover organic materials into their component molecules. The system works by sealing the waste inside an oxygen-free chamber and heating it to temperatures above 400 degrees Celsius. During pyrolysis, useful chemicals are produced, including bio-oil, syngas, and biochar fertiliser. Benefits for Rural Communities: Indoor Air Pollution Reduction: By utilizing BioTRIG, rural Indians can significantly reduce indoor air pollution. Cooking with fossil fuels in unventilated households disproportionately affects women’s and children’s health. Improved Soil Health: Biochar, one of the products of pyrolysis, can enhance soil fertility. It acts as a carbon sink while improving agricultural land quality. Clean Power Generation: The syngas and bio-oil produced by BioTRIG can power the pyrolysis system in future cycles. Surplus electricity can also benefit local homes and businesses. Productive Farmland: By using the biochar and other pyrolysis products, farmers can have more productive farmland. Sources : Down To Earth Practice MCQs Daily Practice MCQs Q1.) In India, melanistic royal Bengal tigers can be found in their natural habitat in: Ranthambore Tiger Reserve Similipal Tiger Reserve Bandipur Tiger Reserve Kaziranga Tiger Reserve Q2.)Consider the following statements: Statement-I: The National Urban Cooperative Finance and Development Corporation Limited (NUCFDC) is an umbrella organization for Urban Cooperative Banks in India. Statement-II: Its primary role is to modernize and strengthen the Rural Cooperative Banking Sector in India. Which one of the following is correct in respect of the above statements? Both Statement-I and Statement-II are correct and Statement-II is the correct explanation for Statement-I Both Statement-I and Statement-II are correct and Statement-II is not the correct explanation for Statement-I Statement-I is correct but Statement-II is incorrect Statement-I is incorrect but Statement-II is correct Q3.) With reference to National Commission for Protection of Child Rights (NCPCR), consider the following statements: NCPCR is a statutory body established by the Government of India. NCPCR operates under the Ministry of Women & Child Development of the Central Government. Chairperson and members are appointed by the Central Government. The Chairperson and other members serve a term of three years. How many statements given above are correct? Only one Only two Only three All four Comment the answers to the above questions in the comment section below!! ANSWERS FOR ’  5th March  2024 – Daily Practice MCQs’ will be updated along with tomorrow’s Daily Current Affairs.st ANSWERS FOR  4th March – Daily Practice MCQs Answers- Daily Practice MCQs Q.1) – c Q.2) – c Q.3) – b

Daily Prelims CA Quiz

UPSC Quiz – 2024 : IASbaba’s Daily Current Affairs Quiz 5th March 2024

For Previous Daily Quiz (ARCHIVES) – CLICK HERE The Current Affairs questions are based on sources like ‘The Hindu’, ‘Indian Express’ and ‘PIB’, which are very important sources for UPSC Prelims Exam. The questions are focused on both the concepts and facts. The topics covered here are generally different from what is being covered under ‘Daily Current Affairs/Daily News Analysis (DNA) and Daily Static Quiz’ to avoid duplication. The questions would be published from Monday to Saturday before 2 PM. One should not spend more than 10 minutes on this initiative. Gear up and Make the Best Use of this initiative. Do remember that, “the difference between Ordinary and EXTRA-Ordinary is PRACTICE!!” Important Note: Don’t forget to post your marks in the comment section. Also, let us know if you enjoyed today’s test 🙂After completing the 5 questions, click on ‘View Questions’ to check your score, time taken, and solutions. To take the Test Click Here

[DAY 02] 60 DAY RAPID REVISION (RaRe) SERIES for UPSC Prelims 2024 – POLITY, CURRENT AFFAIRS & CSAT TEST SERIES!

Archives Hello Friends The 60 Days Rapid Revision (RaRe) Series is IASbaba’s Flagship Initiative recommended by Toppers and loved by the aspirants’ community every year. It is the most comprehensive program which will help you complete the syllabus, revise and practice tests on a daily basis. The Programme on a daily basis includes Daily Prelims MCQs from Static (Monday – Saturday) Daily Static Quiz will cover all the topics of static subjects – Polity, History, Geography, Economics, Environment and Science and technology. 20 questions will be posted daily and these questions are framed from the topics mentioned in the schedule. It will ensure timely and streamlined revision of your static subjects. Daily Current Affairs MCQs (Monday – Saturday) Daily 5 Current Affairs questions, based on sources like ‘The Hindu’, ‘Indian Express’ and ‘PIB’, would be published from Monday to Saturday according to the schedule. Daily CSAT Quiz (Monday – Friday) CSAT has been an Achilles heel for many aspirants. Daily 5 CSAT Questions will be published. Note – Daily Test of 25 static questions, 5 current affairs, and 5 CSAT questions. (35 Prelims Questions) in QUIZ FORMAT will be updated on a daily basis. To Know More about 60 Days Rapid Revision (RaRe) Series – CLICK HERE   60 Day Rapid Revision (RaRe) Series Schedule – CLICK HERE  Important Note Comment your Scores in the Comment Section. This will keep you accountable, responsible and sincere in days to come. It will help us come out with the Cut-Off on a Daily Basis. Let us know if you enjoyed today’s test 🙂  You can post your comments in the given format  (1) Your Score (2) Matrix Meter (3) New Learning from the Test Time limit: 0 Test-summary 0 of 35 questions completed Questions: 1 2 3 4 5 6 7 8 9 10 11 12 13 14 15 16 17 18 19 20 21 22 23 24 25 26 27 28 29 30 31 32 33 34 35 Information The following Test is based on the syllabus of 60 Days Plan-2023 for UPSC IAS Prelims 2022. To view Solutions, follow these instructions: Click on – ‘Start Test’ button Solve Questions Click on ‘Test Summary’ button Click on ‘Finish Test’ button Now click on ‘View Questions’ button – here you will see solutions and links. You have already completed the test before. Hence you can not start it again. Test is loading... You must sign in or sign up to start the test. You have to finish following test, to start this test: Results 0 of 35 questions answered correctly Your time: Time has elapsed You have scored 0 points out of 0 points, (0) Average score     Your score     Categories Not categorized 0% Your result has been entered into leaderboard Loading Name: E-Mail: Captcha: maximum of 70 points Pos. Name Entered on Points Result Table is loading No data available 1 2 3 4 5 6 7 8 9 10 11 12 13 14 15 16 17 18 19 20 21 22 23 24 25 26 27 28 29 30 31 32 33 34 35 Answered Review Question 1 of 35 1. Question Consider the following statements with regard to the Fundamental Right provided under Part III of the Indian Constitution: All Fundamental rights are provided to citizens only. Some fundamental rights provide protections against the private persons. Constitution does not allow the Parliament to make any changes to the fundamental rights. How many of the above statements are correct?  a) Only one b) Only two c) All three d) None Correct Solution (a) Statement 1 Statement 2 Statement 3 Incorrect Correct Incorrect Certain Fundamental Rights are also provided to foreigners (except enemy aliens). Article 14, Article 20, Article 21, Article 21A, Article 22, Article 23, Article 24, Article 25, Article 26, Article 27 & Article 28.   Certain fundamental rights provide protection against private citizens. Example – Article 15 (2), Article 17, Article 23, Article 24 The Supreme Court’s position on constitutional amendments laid out in its judgment of Kesavananda Bharati Case (1973) is that Parliament can amend Part III of the Constitution under article 386 but cannot destroy its ‘basic structure’. Incorrect Solution (a) Statement 1 Statement 2 Statement 3 Incorrect Correct Incorrect Certain Fundamental Rights are also provided to foreigners (except enemy aliens). Article 14, Article 20, Article 21, Article 21A, Article 22, Article 23, Article 24, Article 25, Article 26, Article 27 & Article 28.   Certain fundamental rights provide protection against private citizens. Example – Article 15 (2), Article 17, Article 23, Article 24 The Supreme Court’s position on constitutional amendments laid out in its judgment of Kesavananda Bharati Case (1973) is that Parliament can amend Part III of the Constitution under article 386 but cannot destroy its ‘basic structure’. Question 2 of 35 2. Question Consider the following statement regarding Equality of Opportunity as enshrined in Indian Constitution: It provides for equality of opportunity for all the residents in the country in matters of employment to any office under the State. Government can implement special schemes and measures for improving the conditions of certain sections of society. Which of the above statements is/are correct? a) 1 only b) 2 only c) Both 1 and 2 d) Neither 1 nor 2 Correct Solution (b) Statement 1 Statement 2 Incorrect Correct It provides for equality of opportunity for all citizens and not resident (resident could be foreigner) in matters of employment or appointment to any office under the state. The Constitution clarifies that the government can implement special schemes and measures for improving the conditions of certain sections of society: children, women, and the socially and educationally backward classes.   Incorrect Solution (b) Statement 1 Statement 2 Incorrect Correct It provides for equality of opportunity for all citizens and not resident (resident could be foreigner) in matters of employment or appointment to any office under the state. The Constitution clarifies that the government can implement special schemes and measures for improving the conditions of certain sections of society: children, women, and the socially and educationally backward classes.   Question 3 of 35 3. Question Consider the following statements regarding Right to Freedom as mentioned under Article 19-22? No citizen can be denied his or her life except by procedure as laid down under the law. It protects freedom of speech and expression. It makes elementary education a Fundamental Right. How many of the above statements are correct?  a) Only one b) Only two c) All three d) None Correct Solution (c) Statement 1 Statement 2 Statement 3 Correct Correct Correct According to Article 21: “Protection of Life and Personal Liberty: No person shall be deprived of his life or personal liberty except according to procedure established by law.” This fundamental right is available to every person, citizens and foreigners alike. There is Protection of six rights regarding freedom of: (i) speech and expression, (ii) assembly, (iii) association, (iv) movement, (v) residence, and (vi) profession (Article 19). The Constitution (Eighty-sixth Amendment) Act, 2002 inserted Article 21-A in the Constitution of India to provide free and compulsory education of all children in the age group of six to fourteen years as a Fundamental Right in such a manner as the State may, by law, determine. Incorrect Solution (c) Statement 1 Statement 2 Statement 3 Correct Correct Correct According to Article 21: “Protection of Life and Personal Liberty: No person shall be deprived of his life or personal liberty except according to procedure established by law.” This fundamental right is available to every person, citizens and foreigners alike. There is Protection of six rights regarding freedom of: (i) speech and expression, (ii) assembly, (iii) association, (iv) movement, (v) residence, and (vi) profession (Article 19). The Constitution (Eighty-sixth Amendment) Act, 2002 inserted Article 21-A in the Constitution of India to provide free and compulsory education of all children in the age group of six to fourteen years as a Fundamental Right in such a manner as the State may, by law, determine. Question 4 of 35 4. Question Which among the following statement is/are correct regarding protection offered under Article 22 if a person is arrested under ordinary law? Right to be informed of the grounds of arrest. Right to consult and be defended by a legal practitioner. Right to be produced before a magistrate within 24 hours including the journey time. How many of the above statements are correct?  a) Only one b) Only two c) All three d) None Correct Solution (b) Statement 1 Statement 2 Statement 3 Correct Correct Incorrect Right to be informed of the grounds of arrest. Any person who is in custody has to be informed as to why he has been arrested. Right to consult and be defended by a legal practitioner. These safeguards are, however, not applicable to Enemy aliens People arrested under preventive detention law. Right to be produced before a magistrate within 24 hours excluding the journey time. Incorrect Solution (b) Statement 1 Statement 2 Statement 3 Correct Correct Incorrect Right to be informed of the grounds of arrest. Any person who is in custody has to be informed as to why he has been arrested. Right to consult and be defended by a legal practitioner. These safeguards are, however, not applicable to Enemy aliens People arrested under preventive detention law. Right to be produced before a magistrate within 24 hours excluding the journey time. Question 5 of 35 5. Question Which of the following rights emanate from Right to Life under Article 21 of the Constitution? Right to Privacy Right to Livelihood Right to Clean Environment Right against Public Hanging How many of the above statements are correct? a) Only one b) Only two c) All three d) All four Correct Solution (d) Judicial intervention has ensured that the scope of Article 21 is not narrow and restricted. It has been widening by several landmark judgements. After Maneka Gandhi Case the scope of article 21 has been ever widening. Statement Analysis: Statement 1 Statement 2 Statement 3 Statement 4 Correct Correct Correct Correct Right to privacy Right to livelihood Right to clean environment Right against public hanging Incorrect Solution (d) Judicial intervention has ensured that the scope of Article 21 is not narrow and restricted. It has been widening by several landmark judgements. After Maneka Gandhi Case the scope of article 21 has been ever widening. Statement Analysis: Statement 1 Statement 2 Statement 3 Statement 4 Correct Correct Correct Correct Right to privacy Right to livelihood Right to clean environment Right against public hanging Question 6 of 35 6. Question The ‘86th Constitutional Amendment Act of 2002’ added articles to which of the following provisions of the Constitution of India? Fundamental Duties Fundamental Rights Directive Principles of State Policy How many of the above statements are correct?  a) Only one b) Only two c) All three d) None Correct Solution (c) Statement 1 Statement 2 Statement 3 Correct Correct Correct The 86th Constitutional Amendment Act, 2002 added the eleventh Fundamental duty to Part IV-A of the constitution under article 51-A. It says ‘To provide opportunities for education to his child or ward between the age of six and fourteen years’. The 86th Constitutional Amendment Act also added the ‘Right to Education as a fundamental right to Part III of the constitution under Article 21A. It says ‘The State shall provide free and compulsory education to all children of the age of six to fourteen years in such manner as the State may, by law, determine’. It also added new article 45 to the constitution under Directive Principles of State Policy. It says ‘The state shall endeavor to provide early childhood care and education for all children until they complete the age of six years.’ Incorrect Solution (c) Statement 1 Statement 2 Statement 3 Correct Correct Correct The 86th Constitutional Amendment Act, 2002 added the eleventh Fundamental duty to Part IV-A of the constitution under article 51-A. It says ‘To provide opportunities for education to his child or ward between the age of six and fourteen years’. The 86th Constitutional Amendment Act also added the ‘Right to Education as a fundamental right to Part III of the constitution under Article 21A. It says ‘The State shall provide free and compulsory education to all children of the age of six to fourteen years in such manner as the State may, by law, determine’. It also added new article 45 to the constitution under Directive Principles of State Policy. It says ‘The state shall endeavor to provide early childhood care and education for all children until they complete the age of six years.’ Question 7 of 35 7. Question Over the years Supreme Courts and High Courts have given a wide interpretation to the Fundamental Rights. In this context which of the following are Fundamental Rights? Right to Shelter Right to Apply for Bail Right to not mention the name of one’s father. How many of the above statements are correct?  a) Only one b) Only two c) All three d) None Correct Solution (c) RIGHT TO SHELTER In 2022, in Samarpal versus Union of India, the Delhi high court held that the right to shelter was a fundamental right under Article 21 of the Constitution. Right to Shelter is a Fundamental Right and Even an Encroacher Can’t be Removed Without Adopting Procedure Establish by Law: Jharkhand HC. High Court stated that “in a country like India which professes high democratic values, the Constitution of India stands like a lighthouse illuminating life aspirations of the people of India that every State action must follow the procedure established by law. RMC being an instrumentality of the State under Article 12 of the Constitution of India is governed by the rule of law in a welfare State and cannot arrogate to itself a status beyond what is provided by the Constitution.” The fundamental right to shelter was declared by the court not as a ‘systemic right’ but as a ‘conditional right’. A claim under the right to shelter would stand conditional upon the existence of a particular policy/statute which must provide for the means to facilitate exercising the right. Allahabad HC also held that Right to shelter is a fundamental right, a right to all infrastructure necessary to live and develop as a human being in Rajesh Yadav v. State of UP.  Right to Apply for Bail is an Individual Fundamental Right implicit in Articles 14, 21, and 19: Supreme Court. The Kerala High Court has ruled that citizens have the fundamental right to not mention the name of their father in documents needed to prove identity. Incorrect Solution (c) RIGHT TO SHELTER In 2022, in Samarpal versus Union of India, the Delhi high court held that the right to shelter was a fundamental right under Article 21 of the Constitution. Right to Shelter is a Fundamental Right and Even an Encroacher Can’t be Removed Without Adopting Procedure Establish by Law: Jharkhand HC. High Court stated that “in a country like India which professes high democratic values, the Constitution of India stands like a lighthouse illuminating life aspirations of the people of India that every State action must follow the procedure established by law. RMC being an instrumentality of the State under Article 12 of the Constitution of India is governed by the rule of law in a welfare State and cannot arrogate to itself a status beyond what is provided by the Constitution.” The fundamental right to shelter was declared by the court not as a ‘systemic right’ but as a ‘conditional right’. A claim under the right to shelter would stand conditional upon the existence of a particular policy/statute which must provide for the means to facilitate exercising the right. Allahabad HC also held that Right to shelter is a fundamental right, a right to all infrastructure necessary to live and develop as a human being in Rajesh Yadav v. State of UP.  Right to Apply for Bail is an Individual Fundamental Right implicit in Articles 14, 21, and 19: Supreme Court. The Kerala High Court has ruled that citizens have the fundamental right to not mention the name of their father in documents needed to prove identity. Question 8 of 35 8. Question With reference to the Right to Equality in the Indian Constitution, consider the following statements regarding Articles 15 and 16: Under both the articles, the state cannot discriminate against any citizen on grounds only of religion, race, caste, descent, sex, or place of birth. Under both the articles, it is mentioned that the state is permitted to make any special provision for women and children. Which of the statements given above is/are correct? a) 1 only b) 2 only c) Both 1 and 2 d) Neither 1 nor 2 Correct Solution (d) Statement 1 Statement 2 Incorrect Incorrect While Article 15 provides that the State shall not discriminate against any citizen on grounds only of religion, race, caste, sex or place of birth, Article 16 states that no citizen can be discriminated against or be ineligible for any employment or office under the State on grounds of only religion, race, caste, sex, descent, place of birth or residence. Article 15 states that nothing in this Article shall prevent the state from making any special provision for women and children. There is no such provision in Article 16. Incorrect Solution (d) Statement 1 Statement 2 Incorrect Incorrect While Article 15 provides that the State shall not discriminate against any citizen on grounds only of religion, race, caste, sex or place of birth, Article 16 states that no citizen can be discriminated against or be ineligible for any employment or office under the State on grounds of only religion, race, caste, sex, descent, place of birth or residence. Article 15 states that nothing in this Article shall prevent the state from making any special provision for women and children. There is no such provision in Article 16. Question 9 of 35 9. Question The Doctrine of Essentiality was invented by a seven-judge Bench of the Supreme Court in the context of: a) Religious Rights b) Right to Vote c) Right to Travel Abroad d) Right to Life Correct Solution (a) Essential religious practice test is a doctrine evolved by the supreme court (SC) to protect only such religious practices under fundamental rights, which are essential and integral to religion. The doctrine of “essentiality” was invented by a seven-judge Bench of the SC in the ‘Shirur Mutt’ case in 1954. The court then took the responsibility of determining the essential practices of a religion upon itself. When the exercise of religious practices came in conflict with other fundamental rights guaranteed under the constitution. The doctrine of essentiality was utilized to decide on the constitutionality of such practices. Incorrect Solution (a) Essential religious practice test is a doctrine evolved by the supreme court (SC) to protect only such religious practices under fundamental rights, which are essential and integral to religion. The doctrine of “essentiality” was invented by a seven-judge Bench of the SC in the ‘Shirur Mutt’ case in 1954. The court then took the responsibility of determining the essential practices of a religion upon itself. When the exercise of religious practices came in conflict with other fundamental rights guaranteed under the constitution. The doctrine of essentiality was utilized to decide on the constitutionality of such practices. Question 10 of 35 10. Question Consider the following statements regarding Cultural and Educational Rights: All religious minorities can set up their own educational institutions but this right is not extended to linguistic minorities. The Government cannot discriminate any educational institution on the basis that it is under the management of minority community while granting aid. Which of the above statements is/are correct? a) 1 only b) 2 only c) Both 1 and 2 d) Neither 1 nor 2 Correct Solution (b) Statement 1 Statement 2 Incorrect Correct Article 30(1): All religious and linguistic minorities have the right to establish and administer educational institutions of their choice.   Article 30(2): The State shall not, when granting aid to educational institutions, discriminate against any educational institution on the ground that it is under the management of a minority, whether based on religion or language.. Incorrect Solution (b) Statement 1 Statement 2 Incorrect Correct Article 30(1): All religious and linguistic minorities have the right to establish and administer educational institutions of their choice.   Article 30(2): The State shall not, when granting aid to educational institutions, discriminate against any educational institution on the ground that it is under the management of a minority, whether based on religion or language.. Question 11 of 35 11. Question Which of the following are Constitutional Rights? No tax shall be levied or collected except by the authority of law. No person shall be deprived of his property except by the authority of law. Right to vote in elections. How many of the above statements are correct?  a) Only one b) Only two c) All three d) None Correct Solution (c) Statement 1 Statement 2 Statement 3 Correct Correct Correct Article 265 provides that – No tax shall be levied or collected except by authority of law. Right to Property is no longer a fundamental right, rather it is a Constitutional Right and now exists in Article 300A. Article 300A states that – No person shall be deprived of his property saved by the authority of law. Therefore, the article protects an individual from interference by the State and dispossesses a person of the property unless it is in accordance with the procedure established by law. Article 326 of the Constitution provides for- The elections to the House of the People and to the Legislative Assembly of every State shall be on the basis of adult suffrage.   Incorrect Solution (c) Statement 1 Statement 2 Statement 3 Correct Correct Correct Article 265 provides that – No tax shall be levied or collected except by authority of law. Right to Property is no longer a fundamental right, rather it is a Constitutional Right and now exists in Article 300A. Article 300A states that – No person shall be deprived of his property saved by the authority of law. Therefore, the article protects an individual from interference by the State and dispossesses a person of the property unless it is in accordance with the procedure established by law. Article 326 of the Constitution provides for- The elections to the House of the People and to the Legislative Assembly of every State shall be on the basis of adult suffrage.   Question 12 of 35 12. Question With regard to Martial Law, consider the following statements: It has been borrowed in India from the English common law. It has no specific provision in the Constitution and is implicit in nature. Which of the statements given above is/are correct? a) 1 only b) 2 only c) Both 1 and 2 d) Neither 1 nor 2 Correct Solution (c) Statement 1 Statement 2 Correct Correct The concept of martial law has been borrowed in India from the English common law. The expression ‘martial law’ has not been defined anywhere in the Constitution. It means ‘military rule’. It refers to a situation where civil administration is run by the military authorities according to their own rules and regulations framed outside the ordinary law. There is no specific or express provision in the Constitution that authorizes the executive to declare martial law. However, it is implicit in Article 34 under which martial law can be declared in any area within the territory of India. The martial law is imposed under the extraordinary circumstances like war, invasion, insurrection, rebellion, riot or any violent resistance to law. Its justification is to repel force by force for maintaining or restoring order in the society. Incorrect Solution (c) Statement 1 Statement 2 Correct Correct The concept of martial law has been borrowed in India from the English common law. The expression ‘martial law’ has not been defined anywhere in the Constitution. It means ‘military rule’. It refers to a situation where civil administration is run by the military authorities according to their own rules and regulations framed outside the ordinary law. There is no specific or express provision in the Constitution that authorizes the executive to declare martial law. However, it is implicit in Article 34 under which martial law can be declared in any area within the territory of India. The martial law is imposed under the extraordinary circumstances like war, invasion, insurrection, rebellion, riot or any violent resistance to law. Its justification is to repel force by force for maintaining or restoring order in the society. Question 13 of 35 13. Question Which of the following statements regarding ‘Minorities’ is/are correct: The term ‘minority’ has not been defined in the Indian Constitution. Linguistic minorities are identified on state to state basis. Select the correct answer using the code given below: a) 1 only b) 2 only c) Both 1 and 2 d) Neither 1 nor 2 Correct Solution (c) Statement 1 Statement 2 Correct Correct The Constitution uses the word ‘minorities’ in some articles but does not define it anywhere. Article 29 has the word ‘minorities’ in its marginal heading but speaks of any section of citizens having a distinct language script and culture. Article 30 speaks specifically of two categories of minorities — religious and linguistic. A linguistic minority is a group of people whose mother tongue is different from that of the majority in the state or part of a state. Currently, the linguistic minorities are identified on a state-wise basis thus determined by the state government.   Incorrect Solution (c) Statement 1 Statement 2 Correct Correct The Constitution uses the word ‘minorities’ in some articles but does not define it anywhere. Article 29 has the word ‘minorities’ in its marginal heading but speaks of any section of citizens having a distinct language script and culture. Article 30 speaks specifically of two categories of minorities — religious and linguistic. A linguistic minority is a group of people whose mother tongue is different from that of the majority in the state or part of a state. Currently, the linguistic minorities are identified on a state-wise basis thus determined by the state government.   Question 14 of 35 14. Question Which of the following Articles was introduced in the Constitution of India by the Twenty-Fifth Amendment Act, 1971? a) Article 31A b) Article 31B c) Article 31C d) Article 31D Correct Solution (c) The Twenty-Fifth Amendment Act, 1971 – Curtailed the fundamental right to property. Inserted new Article 31C: Provided that any law made to give effect to the Directive Principles contained in Article 39 (b) or (c) cannot be challenged on the ground of violation of the rights guaranteed by Articles 14, 19 and 31. Incorrect Solution (c) The Twenty-Fifth Amendment Act, 1971 – Curtailed the fundamental right to property. Inserted new Article 31C: Provided that any law made to give effect to the Directive Principles contained in Article 39 (b) or (c) cannot be challenged on the ground of violation of the rights guaranteed by Articles 14, 19 and 31. Question 15 of 35 15. Question Consider the following statements: Article 25 forms the foundation of the Indian secularism. Indian secularism allows the state to regulate secular activities associated with a religious practice. Article 25 forbids imparting of religious instructions from an institution wholly funded by the state. How many of the above statements are correct?  a) Only one b) Only two c) All three d) None Correct Solution (b) Statement 1 Statement 2 Statement 3 Correct Correct Incorrect Article 25 is the foundation of the Indian secularism, not only because it guarantees religious freedom of individuals and equality of all religions, it also makes a clear distinction of religious affairs of the people and the secular activities of the state. Indian secularism allows for the state to regulate secular activities associated with a religious practice. The state is entitled to regulate the secular activities, including economic, financial and political. The state is entitled to undertake social welfare and social reform of all sections of the people. Article 28 forbids the imparting of religious instructions from an institution wholly funded by the state. Incorrect Solution (b) Statement 1 Statement 2 Statement 3 Correct Correct Incorrect Article 25 is the foundation of the Indian secularism, not only because it guarantees religious freedom of individuals and equality of all religions, it also makes a clear distinction of religious affairs of the people and the secular activities of the state. Indian secularism allows for the state to regulate secular activities associated with a religious practice. The state is entitled to regulate the secular activities, including economic, financial and political. The state is entitled to undertake social welfare and social reform of all sections of the people. Article 28 forbids the imparting of religious instructions from an institution wholly funded by the state. Question 16 of 35 16. Question Consider the following statements: Article 29 holds that no citizens shall be denied admission into a school, maintained or aided by the state on the ground of religion, race, caste, sex, place of birth, or any of them. Article 30 holds that only the religious minorities have the right to set up or maintain educational institutions of their own choice. Which of the statements given above is/are correct? a) 1 only b) 2 only c) Both 1 and 2 d) Neither 1 nor 2 Correct Solution (d) Statement 1 Statement 2 Incorrect Incorrect No citizens shall be denied admission into a school, maintained or aided by the state on the ground of religion, race, caste, language, or any of them (Article 29). All religious or linguistic minorities have the right to set up or maintain educational institutions of their own choice. The state, while making grants to the educational institutions, shall not discriminate against a religious or linguistic minority (Article 30). Incorrect Solution (d) Statement 1 Statement 2 Incorrect Incorrect No citizens shall be denied admission into a school, maintained or aided by the state on the ground of religion, race, caste, language, or any of them (Article 29). All religious or linguistic minorities have the right to set up or maintain educational institutions of their own choice. The state, while making grants to the educational institutions, shall not discriminate against a religious or linguistic minority (Article 30). Question 17 of 35 17. Question Consider the following pairs: Article-Description Article 42 – Just and humane conditions of work and maternity relief. Article 43 – Living wages for the workers. Article 43A – Promotion of the co-operative societies. Article 43B – Participation of the workers in the management of the industries. How many of the pairs given above is/are correctly matched? a) One pair only b) Two pairs only c) Three pairs only d) All four pairs Correct Solution (b) Statement 1 Statement 2 Statement 3 Statement 4 Correct Correct Incorrect Incorrect Article 42 – Just and humane conditions of work and maternity relief. Article 43 – Living wages for the workers. Article 43A – Participation of the workers in the management of the Industries. Article 43B – Promotion of the co-operative societies. Incorrect Solution (b) Statement 1 Statement 2 Statement 3 Statement 4 Correct Correct Incorrect Incorrect Article 42 – Just and humane conditions of work and maternity relief. Article 43 – Living wages for the workers. Article 43A – Participation of the workers in the management of the Industries. Article 43B – Promotion of the co-operative societies. Question 18 of 35 18. Question With reference to the Directive Principles of State Policy (DPSP) in the Indian Constitution, consider the following pairs: Directive Principles pf State Policy (DPSP) – Category To secure the participation of workers – Liberal Intellectual. in the management of industries. To prohibit the consumption of intoxicating drinks – Socialist To secure for all citizens a uniform civil code – Gandhian How many of the pairs given above is/are correctly matched? a) One pair only b) Two pairs only c) Three pairs only d) None of the pairs Correct Solution (d) Statement 1 Statement 2 Statement 3 Incorrect Incorrect Incorrect To secure the participation of workers in the management of industries (Article 43A) – Socialist To prohibit the consumption of intoxicating drinks (Article 47) – Gandhian To secure for all citizens a uniform civil code (Article 44) – Liberal Intellectual Incorrect Solution (d) Statement 1 Statement 2 Statement 3 Incorrect Incorrect Incorrect To secure the participation of workers in the management of industries (Article 43A) – Socialist To prohibit the consumption of intoxicating drinks (Article 47) – Gandhian To secure for all citizens a uniform civil code (Article 44) – Liberal Intellectual Question 19 of 35 19. Question Since independence, many laws have been enacted by the state to give effect to the various Directive Principles under part IV of the Constitution. In this context consider the following statements: The Wildlife (Protection) Act, 1972 and the Forest (Conservation) Act, 1980, enactments give effect to Article 50. Khadi and Village Industries Board gives effect to Article 43. 73rd Constitutional amendment Act gives effect to Article 40. How many of the above statements are correct?  a) Only one b) Only two c) All three d) None Correct Solution (b) Statement 1 Statement 2 Statement 3 Incorrect Correct Correct Article 48A seeks to protect and improve the environment and to safeguard forests and wildlife. The Wildlife (Protection) Act, 1972 and the Forest (Conservation) Act, 1980, have been enacted to give effect to Article 48A, not Article 50. Article 43 seeks to promote cottage industries on an individual or cooperation basis in rural areas. Khadi and Village Industries Board gives effect to Article 43. Article 40 seeks to organize village panchayats and endow them with the necessary powers and authority to enable them to function as units of self-government. 73rd Constitutional Amendment Act gives effect to Article 40. Incorrect Solution (b) Statement 1 Statement 2 Statement 3 Incorrect Correct Correct Article 48A seeks to protect and improve the environment and to safeguard forests and wildlife. The Wildlife (Protection) Act, 1972 and the Forest (Conservation) Act, 1980, have been enacted to give effect to Article 48A, not Article 50. Article 43 seeks to promote cottage industries on an individual or cooperation basis in rural areas. Khadi and Village Industries Board gives effect to Article 43. Article 40 seeks to organize village panchayats and endow them with the necessary powers and authority to enable them to function as units of self-government. 73rd Constitutional Amendment Act gives effect to Article 40. Question 20 of 35 20. Question In which of the following landmark cases, Supreme Court described that the Indian Constitution is founded on the bedrock of the balance between Fundamental Rights and Directive Principles of State Policy? a) Champakam Dorairajan vs State of Madras b) Minerva Mills vs Union of India c) Kesavananda Bharati vs State of Kerala d) Maneka Gandhi vs Union of India Correct Solution (b) In Minerva Mills case, 1980, the Supreme Court described that ‘The Indian Constitution is founded on the bedrock of the balance between Fundamental Rights and Directive Principles of State Policy. To give absolute primacy to one over the other is to disturb the harmony of the Constitution. This harmony and balance is an essential feature of the basic structure of the Constitution.   Incorrect Solution (b) In Minerva Mills case, 1980, the Supreme Court described that ‘The Indian Constitution is founded on the bedrock of the balance between Fundamental Rights and Directive Principles of State Policy. To give absolute primacy to one over the other is to disturb the harmony of the Constitution. This harmony and balance is an essential feature of the basic structure of the Constitution.   Question 21 of 35 21. Question Consider the following statements about the PM Programme for Restoration, Awareness, Nourishment, and Amelioration of Mother Earth (PM-PRANAM) Scheme: Its objective is to encourage the balanced use of fertilizers inconjunction with biofertilizers and organic fertilizers. It will be financed by the savings of existing fertilizer subsidiesunder schemes run by the Department of Fertilizers, Ministry of Chemicals & Fertilizers. The reduction in urea consumption by a state will be compared to its average consumption of urea over the previous ten years. How many of the above statements are correct? a) Only one b) Only two c) All three d) None Correct Solution (b) The PM Programme for Restoration, Awareness, Nourishment, and Amelioration of Mother Earth (PM-PRANAM) Scheme’s objective is to encourage the balanced use of fertilizers inconjunction with biofertilizers and organic fertilizers and to reduce the subsidy burden on chemical fertilizers, which was around Rs 2.25 lakh crores in 2022-2023. Hence statement 1 is correct. It will be financed by the savings of existing fertilizer subsidiesunder schemes run by the Department of Fertilizers, Ministry of Chemicals & Fertilizers. Hence statement 2 is correct. The Centre will provide 50% of the subsidy savings to the states as a grant. Out of the grant, 70% can be used to create assets related to the technological adoption of alternative fertilizers and production units at various levels. The remaining 30% can be used to reward and encourage farmers, panchayats, and other stakeholders involved in fertilizer reduction and awareness generation. The reduction in urea consumption by a state will be compared to its average consumption of urea over the previous three years. This calculation will determine the eligibility for subsidy savings and grants. Hence statement 3 is incorrect.   Note: Biofertilizer consists of a carrier medium rich in live microorganisms. When applied to seed, soil, or living plants, it increases soil nutrients or makes them biologically available. Biofertilizers contain different types of fungi, root bacteria, or other microorganisms. They form a mutually beneficial or symbiotic relationship with host plants as they grow in the soil. Incorrect Solution (b) The PM Programme for Restoration, Awareness, Nourishment, and Amelioration of Mother Earth (PM-PRANAM) Scheme’s objective is to encourage the balanced use of fertilizers inconjunction with biofertilizers and organic fertilizers and to reduce the subsidy burden on chemical fertilizers, which was around Rs 2.25 lakh crores in 2022-2023. Hence statement 1 is correct. It will be financed by the savings of existing fertilizer subsidiesunder schemes run by the Department of Fertilizers, Ministry of Chemicals & Fertilizers. Hence statement 2 is correct. The Centre will provide 50% of the subsidy savings to the states as a grant. Out of the grant, 70% can be used to create assets related to the technological adoption of alternative fertilizers and production units at various levels. The remaining 30% can be used to reward and encourage farmers, panchayats, and other stakeholders involved in fertilizer reduction and awareness generation. The reduction in urea consumption by a state will be compared to its average consumption of urea over the previous three years. This calculation will determine the eligibility for subsidy savings and grants. Hence statement 3 is incorrect.   Note: Biofertilizer consists of a carrier medium rich in live microorganisms. When applied to seed, soil, or living plants, it increases soil nutrients or makes them biologically available. Biofertilizers contain different types of fungi, root bacteria, or other microorganisms. They form a mutually beneficial or symbiotic relationship with host plants as they grow in the soil. Question 22 of 35 22. Question Consider the following statements about the National Sickle Cell Anaemia Elimination Mission: It aims to eliminate sickle cell disease as a public health problem in India by the year 2035. It is implemented in all the states and union territories across the country. Choose the correct code: a) 1 only b) 2 only c) Both 1 and 2 d) Neither 1 nor 2 Correct Solution (d) Sickle cell disease (SCD) is a chronic single-gene disorder causing a debilitating systemic syndrome characterized by chronic anemia, acute painful episodes, organ infarction, and chronic organ damage and by a significant reduction in life expectancy. The National Sickle Cell Anaemia Elimination Mission aims to improve the care of all sickle cell disease patients for a better future and to lower the prevalence of the disease through a multi-faced coordinated approach towards screening and awareness strategies. It aims to eliminate sickle cell disease as a public health problem in India by 2047. Hence statement 1 is incorrect. It is implemented in 17 high-focus states across the country, this program aims to improve the care and prospects of all sickle cell disease patients while reducing the prevalence of the disease. The 17 states are- Gujarat, Maharashtra, Rajasthan, Madhya Pradesh, Jharkhand, Chhattisgarh, West Bengal, Odisha, Tamil Nadu, Telangana, Andhra Pradesh, Karnataka, Assam, Uttar Pradesh, Kerala, Bihar, and Uttarakhand. Hence statement 2 is incorrect. Incorrect Solution (d) Sickle cell disease (SCD) is a chronic single-gene disorder causing a debilitating systemic syndrome characterized by chronic anemia, acute painful episodes, organ infarction, and chronic organ damage and by a significant reduction in life expectancy. The National Sickle Cell Anaemia Elimination Mission aims to improve the care of all sickle cell disease patients for a better future and to lower the prevalence of the disease through a multi-faced coordinated approach towards screening and awareness strategies. It aims to eliminate sickle cell disease as a public health problem in India by 2047. Hence statement 1 is incorrect. It is implemented in 17 high-focus states across the country, this program aims to improve the care and prospects of all sickle cell disease patients while reducing the prevalence of the disease. The 17 states are- Gujarat, Maharashtra, Rajasthan, Madhya Pradesh, Jharkhand, Chhattisgarh, West Bengal, Odisha, Tamil Nadu, Telangana, Andhra Pradesh, Karnataka, Assam, Uttar Pradesh, Kerala, Bihar, and Uttarakhand. Hence statement 2 is incorrect. Question 23 of 35 23. Question Consider the following statements: The Sanskrit language is included in the eighth schedule of the Indian constitution. Viswa Samskrita Dinam is an annual event celebrated on22nd August. Sudharma is the only Sanskrit newspaper in the world. How many of the above statements are correct? a) Only one b) Only two c) All three d) None Correct Solution (c) Sanskrit is one of the oldest languages in the world. It belongs to the Indo-Aryan language group and was used to write Vedas. The Eighth Schedule to the Constitution of India lists the official languages of the Republic of India. Part XVII, Articles 343 to 351 of the Indian Constitution deals with the official languages. There are 22 scheduled languages in the Eighth Schedule. They are – Assamese, Bengali, Gujarati, Hindi, Kannada, Kashmiri, Konkani, Malayalam, Manipuri, Marathi, Nepali, Odia, Punjabi, Sanskrit, Sindhi, Tamil, Telugu, Urdu, Bodo, Santhali, Maithili and Dogri. The Sanskrit language is included in the eighth schedule of the Indian constitution. Hence statement 1 is correct. Viswa Samskrita Dinam is an annual event celebrated on22nd August. Its objective is to promote the revival and maintenance of the Sanskrit language. Hence statement 2 is correct. It is celebrated for the first time in the year 1969. It is celebrated on Poornima day of the Shraavana monthin the Hindu calendar. Sudharma is the only Sanskrit newspaper in the world. It is published since 1970 from Mysore in Karnataka. It is also available in online mode. Hence statement 3 is correct. Mattur, a village in the Shimoga district of Karnataka is believed to have preserved the Sanskrit language. Incorrect Solution (c) Sanskrit is one of the oldest languages in the world. It belongs to the Indo-Aryan language group and was used to write Vedas. The Eighth Schedule to the Constitution of India lists the official languages of the Republic of India. Part XVII, Articles 343 to 351 of the Indian Constitution deals with the official languages. There are 22 scheduled languages in the Eighth Schedule. They are – Assamese, Bengali, Gujarati, Hindi, Kannada, Kashmiri, Konkani, Malayalam, Manipuri, Marathi, Nepali, Odia, Punjabi, Sanskrit, Sindhi, Tamil, Telugu, Urdu, Bodo, Santhali, Maithili and Dogri. The Sanskrit language is included in the eighth schedule of the Indian constitution. Hence statement 1 is correct. Viswa Samskrita Dinam is an annual event celebrated on22nd August. Its objective is to promote the revival and maintenance of the Sanskrit language. Hence statement 2 is correct. It is celebrated for the first time in the year 1969. It is celebrated on Poornima day of the Shraavana monthin the Hindu calendar. Sudharma is the only Sanskrit newspaper in the world. It is published since 1970 from Mysore in Karnataka. It is also available in online mode. Hence statement 3 is correct. Mattur, a village in the Shimoga district of Karnataka is believed to have preserved the Sanskrit language. Question 24 of 35 24. Question Which of the following cases can be taken up by the National Human Rights Commission (NHRC): Abuse of the legal system in the trafficking of girls Rape of a visually impaired girl Death of a boy in the observation home How many of the above statements are correct? a) Only one b) Only two c) All three d) None Correct Solution (c) The National Human Right Commission was established under the Protection of Human Rights Act (PHRA) of 1993. A few major issues which are taken up by NHRC are: Custodial Torture Right to Work and Labour Rights Arbitrary Arrest and Detention Excessive Powers of the Armed Forces and the Police Sexual Violence Child Labour Violence and discrimination against Women, Children Lesbian, Gay, Bisexual, and Transgender Rights   Incorrect Solution (c) The National Human Right Commission was established under the Protection of Human Rights Act (PHRA) of 1993. A few major issues which are taken up by NHRC are: Custodial Torture Right to Work and Labour Rights Arbitrary Arrest and Detention Excessive Powers of the Armed Forces and the Police Sexual Violence Child Labour Violence and discrimination against Women, Children Lesbian, Gay, Bisexual, and Transgender Rights   Question 25 of 35 25. Question Consider the following statements regarding the provisions of the Places of Worship Act of 1991: It declares that the religious character of a place of worship shall continue to be the same as it existed on August 15, 1947. It does not apply to the Ram Janmabhoomi-Babri Masjid case, and any suit, appeal, or proceeding relating to it. It prescribes a punishment of a maximum of three years imprisonment along with a fine for contravening the provisions of the Act. How many of the above statements are correct? a) Only one b) Only two c) All three d) None Correct Solution (c) The Places of Worship Act of 1991 was enacted to prohibit conversion of any place of worship and to provide for the maintenance of the religious character of any place of worship as it existed on the 15th of August 1947. Its major provisions are: Section 3 of the Act bars the conversion, in full or part, of a place of worship of any religious denomination into a place of worship of a different religious denomination, or even a different segment of the same religious denomination. Section 4(1) of the Act declares that the religious character of a place of worship shall continue to be the same as it existed on August 15, 1947. Hence statement 1 is correct. Section 5 of the Act shall not apply to the Ram Janmabhoomi-Babri Masjid case, and any suit, appeal, or proceeding relating to it. Hence statement 2 is correct. Section 6 of the Act prescribes a punishment of a maximum of three years imprisonment along with a fine for contravening the provisions of the Act. Hence statement 3 is correct. Section 4(2) of the Act declares any suit or legal proceeding with respect to the conversion of the religious character of any place of worship existing on August 15, 1947, pending before any court, shall abate — and no fresh suit or legal proceedings shall be instituted. Incorrect Solution (c) The Places of Worship Act of 1991 was enacted to prohibit conversion of any place of worship and to provide for the maintenance of the religious character of any place of worship as it existed on the 15th of August 1947. Its major provisions are: Section 3 of the Act bars the conversion, in full or part, of a place of worship of any religious denomination into a place of worship of a different religious denomination, or even a different segment of the same religious denomination. Section 4(1) of the Act declares that the religious character of a place of worship shall continue to be the same as it existed on August 15, 1947. Hence statement 1 is correct. Section 5 of the Act shall not apply to the Ram Janmabhoomi-Babri Masjid case, and any suit, appeal, or proceeding relating to it. Hence statement 2 is correct. Section 6 of the Act prescribes a punishment of a maximum of three years imprisonment along with a fine for contravening the provisions of the Act. Hence statement 3 is correct. Section 4(2) of the Act declares any suit or legal proceeding with respect to the conversion of the religious character of any place of worship existing on August 15, 1947, pending before any court, shall abate — and no fresh suit or legal proceedings shall be instituted. Question 26 of 35 26. Question Consider the following statements about Alluri Sitarama Raju: He is considered as Manyam Veerudufor his bravery and sacrifice. He led a guerrilla campaign in the Eastern Ghats regionof Andhra Pradesh. He opposed the British in response to the Madras Forest Act of 1882 in the backdrop of the Swadeshi movement. How many of the above statements are correct? a) Only one b) Only two c) All three d) None Correct Solution (b) 125th Birth Anniversary of Alluri Sitarama Raju. Alluri Sitarama Raju is considered as Manyam Veerudufor his bravery and sacrifice. Hence statement 1 is correct. He led a guerrilla campaign in the Eastern Ghats regionof Andhra Pradesh. Hence statement 2 is correct. He opposed the British in response to the Madras Forest Act of 1882 in the backdrop of the Non-cooperation movement. Hence statement 3 is incorrect. The Madras Forest Act of 1882 effectively restricted the free movement of Adivasis in their forest habitats and prevented them from practicing their traditional form of agriculture called ‘podu’, which threatened their very way of life. It led to the Rampa rebellion (1922-1924) in which Alluri Sitarama Raju played the major role as its leader.   Incorrect Solution (b) 125th Birth Anniversary of Alluri Sitarama Raju. Alluri Sitarama Raju is considered as Manyam Veerudufor his bravery and sacrifice. Hence statement 1 is correct. He led a guerrilla campaign in the Eastern Ghats regionof Andhra Pradesh. Hence statement 2 is correct. He opposed the British in response to the Madras Forest Act of 1882 in the backdrop of the Non-cooperation movement. Hence statement 3 is incorrect. The Madras Forest Act of 1882 effectively restricted the free movement of Adivasis in their forest habitats and prevented them from practicing their traditional form of agriculture called ‘podu’, which threatened their very way of life. It led to the Rampa rebellion (1922-1924) in which Alluri Sitarama Raju played the major role as its leader.   Question 27 of 35 27. Question Consider the following statements about National Bank for Agriculture and Rural Development (NABARD): It is a statutory body formed in 1982 under the Reserve Bank of India of 1934 Act. It deals with the operations of giving credit foragriculture and other economic activities in rural areas. Choose the correct code: a) 1 only b) 2 only c) Both 1 and 2 d) Neither 1 nor 2 Correct Solution (b) National Bank for Agriculture and Rural Development (NABARD) is a statutory body formed in 1982 under the National Bank for Agriculture and Rural Development Act of 1981. Hence statement 1 is incorrect. It is fully owned by the Government of India and is headquartered in Mumbai. It deals with the operations of giving credit foragriculture and other economic activities in rural areas. Hence statement 2 is correct. It is a refinancing agency for those institutions that provide investment and production credit for promoting several developmental programs for rural development. Incorrect Solution (b) National Bank for Agriculture and Rural Development (NABARD) is a statutory body formed in 1982 under the National Bank for Agriculture and Rural Development Act of 1981. Hence statement 1 is incorrect. It is fully owned by the Government of India and is headquartered in Mumbai. It deals with the operations of giving credit foragriculture and other economic activities in rural areas. Hence statement 2 is correct. It is a refinancing agency for those institutions that provide investment and production credit for promoting several developmental programs for rural development. Question 28 of 35 28. Question Consider the following statements about Non-Fungible Tokens (NFTs): They are assets in the digital worldthat can be bought and sold like any other piece of property. They can be tweets, drawings, photos, videos, GIFs, music, in-game items, and selfies. They are backed by blockchain technology and can have multiple owners at a time. They can be bought by anyone who holds a cryptocurrency wallet. How many of the above statements are correct? a) Only one b) Only two c) Only three d) All four Correct Solution (c) Non-Fungible Tokens (NFTs) are assets in the digital worldthat can be bought and sold like any other piece of property. Hence statement 1 is correct. They can be tweets, drawings, photos, videos, GIFs, music, in-game items, and selfies. Hence statement 2 is correct. They are backed by blockchain technology and can have only one owner at a time. Hence statement 3 is incorrect. NFT owners can also digitally sign their artwork and store specific information in their NFTs’ metadata. This will be only viewable to the individual who bought the NFT. They can be bought by anyone who holds a cryptocurrency wallet. Hence statement 4 is correct. Incorrect Solution (c) Non-Fungible Tokens (NFTs) are assets in the digital worldthat can be bought and sold like any other piece of property. Hence statement 1 is correct. They can be tweets, drawings, photos, videos, GIFs, music, in-game items, and selfies. Hence statement 2 is correct. They are backed by blockchain technology and can have only one owner at a time. Hence statement 3 is incorrect. NFT owners can also digitally sign their artwork and store specific information in their NFTs’ metadata. This will be only viewable to the individual who bought the NFT. They can be bought by anyone who holds a cryptocurrency wallet. Hence statement 4 is correct. Question 29 of 35 29. Question Consider the following statements about the Directorate of Revenue Intelligence: It is the apex anti-smuggling agency of India. It works under the Ministry of Commerce and Industry. It enforces the provisions of the Customs Act, Wildlife Act, and Antiquities Act. How many of the above statements are correct? a) Only one b) Only two c) All three d) None Correct Solution (b) The Directorate of Revenue Intelligence (DRI) is the apex anti-smuggling agency of India. Hence statement 1 is correct. It works under the Department of Revenue, Ministry of Finance, Government of India. Hence statement 2 is incorrect. DRI enforces the provisions of the Customs Act, Arms Act, NDPS Act, COFEPOSA, Wildlife Act, Antiquities Act, etc. Hence statement 3 is correct. DRI undertakes the collection, collation, analysis, and dissemination of intelligence relating to smuggling, carries out investigations, adjudication of cases, and prosecution of the arrested persons. Incorrect Solution (b) The Directorate of Revenue Intelligence (DRI) is the apex anti-smuggling agency of India. Hence statement 1 is correct. It works under the Department of Revenue, Ministry of Finance, Government of India. Hence statement 2 is incorrect. DRI enforces the provisions of the Customs Act, Arms Act, NDPS Act, COFEPOSA, Wildlife Act, Antiquities Act, etc. Hence statement 3 is correct. DRI undertakes the collection, collation, analysis, and dissemination of intelligence relating to smuggling, carries out investigations, adjudication of cases, and prosecution of the arrested persons. Question 30 of 35 30. Question Consider the following statements about Nitrogen Dioxide (NO2): It is an odourless, acidic, and extremely corrosive gas. It causes air pollution, water pollution, and acid rain. It does not occur naturally but can be produced industrially. How many of the above statements are correct? a) Only one b) Only two c) All three d) None Correct Solution (b) Nitrogen Dioxide (NO2) is an odourless, acidic, and extremely corrosive gas. It is a highly reactive gas. Hence statement 1 is correct. It causes air pollution, water pollution, and acid rain. Hence statement 2 is correct. It occurs naturally and industrially. It forms from emissions from cars, trucks and buses, power plants, and off-road equipment. It is created by natural sources that include volcanoes and microbes. Hence statement 3 is incorrect. Incorrect Solution (b) Nitrogen Dioxide (NO2) is an odourless, acidic, and extremely corrosive gas. It is a highly reactive gas. Hence statement 1 is correct. It causes air pollution, water pollution, and acid rain. Hence statement 2 is correct. It occurs naturally and industrially. It forms from emissions from cars, trucks and buses, power plants, and off-road equipment. It is created by natural sources that include volcanoes and microbes. Hence statement 3 is incorrect. Question 31 of 35 31. Question If every alternative letter of English alphabet from B onwards (including B) is written in lower case (small letters) and the remaining letters are capitalized, then how will the first month of the second half of the year be written? a) JuLY b) jULy c) jUly d) jUlY Correct Solution (d) According to the question, every alternative letter of the English alphabet from B onwards (including B) is written in lower case and the remaining letters are capitalized. A b C d E f G h I j K l M n O p Q r S t U v W x Y z First month of second half of the year is July. As per the given pattern it will be written as : jUlY Hence, jUlY is the correct answer.   Incorrect Solution (d) According to the question, every alternative letter of the English alphabet from B onwards (including B) is written in lower case and the remaining letters are capitalized. A b C d E f G h I j K l M n O p Q r S t U v W x Y z First month of second half of the year is July. As per the given pattern it will be written as : jUlY Hence, jUlY is the correct answer.   Question 32 of 35 32. Question A bus travels at 54km/h while moving but after accounting for stopping time, to let passengers on and off the coach, it averages a speed of 45km/h. How many minutes does the bus stop for each hour?   a) 9 minutes b) 10 minutes c) 11 minutes d) 12 minutes Correct Solution (b) In one hour without stopping, the bus would have travelled 54km. Once stopping is factored in, the bus actually travels 45km. So it travels 9km less far as a consequence. Distance/Speed = Time 9km/54kmph = 0.16 hours 0.16 x 60 = 10 minutes So the bus stops for 10 minutes in every hour on average.   Incorrect Solution (b) In one hour without stopping, the bus would have travelled 54km. Once stopping is factored in, the bus actually travels 45km. So it travels 9km less far as a consequence. Distance/Speed = Time 9km/54kmph = 0.16 hours 0.16 x 60 = 10 minutes So the bus stops for 10 minutes in every hour on average.   Question 33 of 35 33. Question The product of two positive consecutive odd integers is 1 less than six times their sum. One of the integers is a) 7 b) 9 c) 15 d) 13 Correct Solution (d) Product of two positive consecutive odd integers = (6 * sum of two positive consecutive odd integers ) – 1 Calculation Let the two positive consecutive odd integers be x and (x+2)   Therefore, we have x * (x+2) = [6*[x + x + 2]-1 x^2 + 2x = 12x + 12 -1 x^2-10x-11 = 0 x^2 + x – 11x – 11=0 x(x + 1) – 11(x + 1) = 0 x+1 = 0 or x-11 = 0 x – 11 = 0 , x = 11 Hence, positive integers are ( x, x+2) = (11,13) 13 is the answer Incorrect Solution (d) Product of two positive consecutive odd integers = (6 * sum of two positive consecutive odd integers ) – 1 Calculation Let the two positive consecutive odd integers be x and (x+2)   Therefore, we have x * (x+2) = [6*[x + x + 2]-1 x^2 + 2x = 12x + 12 -1 x^2-10x-11 = 0 x^2 + x – 11x – 11=0 x(x + 1) – 11(x + 1) = 0 x+1 = 0 or x-11 = 0 x – 11 = 0 , x = 11 Hence, positive integers are ( x, x+2) = (11,13) 13 is the answer Question 34 of 35 34. Question Replace the incorrect term by the correct term in the given sequence 3, 6, 7, 10, 13, 16, 21, 24, 31, 36, 43, 46 where odd terms and even terms follow the same pattern.   a) 13 b) 16 c) 34 d) 43 Correct Solution (c) On separating the Odd and the Even Series, we observe the following pattern: ODD SERIES   EVEN SERIES 3 3 + 4 =7                        6 7 + 6 =13                      6+4 = 10 13 + 8=21                     10+6 =16 21 +10=31                    16+8=24 31 +12=43                    24+10=34 34+12=46 On observing the given two series, we observe that the fifth term in EVEN SERIES should be replaced by 34. Incorrect Solution (c) On separating the Odd and the Even Series, we observe the following pattern: ODD SERIES   EVEN SERIES 3 3 + 4 =7                        6 7 + 6 =13                      6+4 = 10 13 + 8=21                     10+6 =16 21 +10=31                    16+8=24 31 +12=43                    24+10=34 34+12=46 On observing the given two series, we observe that the fifth term in EVEN SERIES should be replaced by 34. Question 35 of 35 35. Question What is X in the sequence. 153, 146, 137, 126, 113, 98, 81, X a) 60 b) 61 c) 62 d) 60 Correct Solution (c) 153, 146, 137, 126, 113, 98, 81, X 7         9     11     13    15  17  19 – > difference between the numbers   So ,subtracting 19 from 81 gives the answer, which is 62.   Incorrect Solution (c) 153, 146, 137, 126, 113, 98, 81, X 7         9     11     13    15  17  19 – > difference between the numbers   So ,subtracting 19 from 81 gives the answer, which is 62.   window.wpProQuizInitList = window.wpProQuizInitList || []; window.wpProQuizInitList.push({ id: '#wpProQuiz_3500', init: { quizId: 3500, mode: 1, globalPoints: 70, timelimit: 1800, resultsGrade: [0], bo: 704, qpp: 0, catPoints: [70], formPos: 0, lbn: "Test-summary", json: {"29472":{"type":"single","id":29472,"catId":0,"points":2,"correct":[1,0,0,0]},"29473":{"type":"single","id":29473,"catId":0,"points":2,"correct":[0,1,0,0]},"29475":{"type":"single","id":29475,"catId":0,"points":2,"correct":[0,0,1,0]},"29477":{"type":"single","id":29477,"catId":0,"points":2,"correct":[0,1,0,0]},"29478":{"type":"single","id":29478,"catId":0,"points":2,"correct":[0,0,0,1]},"29480":{"type":"single","id":29480,"catId":0,"points":2,"correct":[0,0,1,0]},"29483":{"type":"single","id":29483,"catId":0,"points":2,"correct":[0,0,1,0]},"29485":{"type":"single","id":29485,"catId":0,"points":2,"correct":[0,0,0,1]},"29487":{"type":"single","id":29487,"catId":0,"points":2,"correct":[1,0,0,0]},"29488":{"type":"single","id":29488,"catId":0,"points":2,"correct":[0,1,0,0]},"29489":{"type":"single","id":29489,"catId":0,"points":2,"correct":[0,0,1,0]},"29490":{"type":"single","id":29490,"catId":0,"points":2,"correct":[0,0,1,0]},"29493":{"type":"single","id":29493,"catId":0,"points":2,"correct":[0,0,1,0]},"29494":{"type":"single","id":29494,"catId":0,"points":2,"correct":[0,0,1,0]},"29497":{"type":"single","id":29497,"catId":0,"points":2,"correct":[0,1,0,0]},"29499":{"type":"single","id":29499,"catId":0,"points":2,"correct":[0,0,0,1]},"29501":{"type":"single","id":29501,"catId":0,"points":2,"correct":[0,1,0,0]},"29503":{"type":"single","id":29503,"catId":0,"points":2,"correct":[0,0,0,1]},"29505":{"type":"single","id":29505,"catId":0,"points":2,"correct":[0,1,0,0]},"29508":{"type":"single","id":29508,"catId":0,"points":2,"correct":[0,1,0,0]},"29511":{"type":"single","id":29511,"catId":0,"points":2,"correct":[0,1,0,0]},"29514":{"type":"single","id":29514,"catId":0,"points":2,"correct":[0,0,0,1]},"29515":{"type":"single","id":29515,"catId":0,"points":2,"correct":[0,0,1,0]},"29516":{"type":"single","id":29516,"catId":0,"points":2,"correct":[0,0,1,0]},"29519":{"type":"single","id":29519,"catId":0,"points":2,"correct":[0,0,1,0]},"29521":{"type":"single","id":29521,"catId":0,"points":2,"correct":[0,1,0,0]},"29524":{"type":"single","id":29524,"catId":0,"points":2,"correct":[0,1,0,0]},"29527":{"type":"single","id":29527,"catId":0,"points":2,"correct":[0,0,1,0]},"29529":{"type":"single","id":29529,"catId":0,"points":2,"correct":[0,1,0,0]},"29530":{"type":"single","id":29530,"catId":0,"points":2,"correct":[0,1,0,0]},"29533":{"type":"single","id":29533,"catId":0,"points":2,"correct":[0,0,0,1]},"29536":{"type":"single","id":29536,"catId":0,"points":2,"correct":[0,1,0,0]},"29539":{"type":"single","id":29539,"catId":0,"points":2,"correct":[0,0,0,1]},"29541":{"type":"single","id":29541,"catId":0,"points":2,"correct":[0,0,1,0]},"29544":{"type":"single","id":29544,"catId":0,"points":2,"correct":[0,0,1,0]}} } }); All the Best IASbaba

DAILY CURRENT AFFAIRS IAS | UPSC Prelims and Mains Exam – 4th March 2024

Archives (PRELIMS & MAINS Focus)   BUREAU OF ENERGY EFFICIENCY (BEE) Syllabus Prelims – Current event Context: Recently, 22nd Foundation Day of the Bureau of Energy Efficiency was celebrated in New Delhi. Background:- It was created in March 2002 under the provisions of the Energy Conservation Act 2001. About Bureau of Energy Efficiency:-   The The Bureau of Energy Efficiency (BEE) is a statutory body under the Ministry of Power, Government of India. The agency’s primary function is to encourage the efficient use of energy in India by developing programs and strategies that promote energy conservation. BEE focuses on self-regulation and market principles to enhance energy efficiency across various sectors. Key Initiatives of BEE Standards & Labelling Scheme: BEE provides consumers with information on energy-efficient appliances and equipment, allowing them to make informed choices. Energy Conservation Building Code: BEE promotes energy-efficient practices in building construction and design. Perform, Achieve, and Trade (PAT): This program targets energy-intensive industries and encourages them to achieve specific energy-saving targets. Energy Efficiency in Large Industry: BEE collaborates with large industries to enhance energy efficiency. Energy Efficiency in Small & Medium Industry: Similar to large industries, BEE works with small and medium-sized enterprises to improve energy efficiency. Energy Efficiency in States: BEE supports state-level initiatives to promote energy conservation. The State Energy Efficiency Index 2023 was released by the Bureau of Energy Efficiency (BEE) in India. This index evaluates and ranks states based on their energy efficiency efforts, policies, and programs. Demand Side Management (DSM): BEE focuses on managing energy demand through various strategies. National Energy Conservation Awards: Recognizing outstanding efforts in energy conservation. Awareness Campaigns: BEE runs campaigns to raise awareness about energy efficiency, such as the recent #RaiseItBy1Degree campaign encouraging optimal space cooling settings. Source: PIB Kaveri and Tungabhadra Rivers Syllabus Prelims – Geography Context: Acute scarcity of water has been reported in various parts of Karnataka as rivers carry less and less water Background: Most of the areas in the arid Kalayana Karnataka region that are dependent on Krishna and Tungabhadra rivers are bracing for a crisis as south west monsoon failed. About Kaveri River: The Cauvery River (Kaveri) is designated as the ‘Dakshin Bharat ki Ganga’ or ‘the Ganga of the South’. The Cauvery River rises at Talakaveri on the Brahmagiri range near Cherangala village, Kodagu (Coorg), Karnataka. It flows through the states of Karnataka and Tamil Nadu and descends the Eastern Ghats in a series of great falls. Before emptying into the Bay of Bengal south of Cuddalore, Tamil Nadu the river breaks into a large number of distributaries forming a wide delta called the “garden of southern India” It is bounded by the Western Ghats on the west, by the Eastern Ghats on the east and the south, and by the ridges separating it from the Krishna basin and Pennar basin on the north. About Tungabhadra River The Tungabhadra River is a river in India that starts and flows through the state of Karnataka during most of its course, Andhra Pradesh and ultimately joining the Krishna River near Murvakonda in Andhra Pradesh. The river Tungabhadra derives its name from two streams viz., the Tunga, about 147 km long and the Bhadra, about 178 km long. The Tunga and the Bhadra rise at Gangamoola, in Varaha Parvatha in the Western Ghats at an elevation of 1458 metres The river after the confluence of the two streams near Shimoga, runs for about 531 km till it joins the river Krishna at Sangamaleshwaram in Andhra Pradesh. It runs for 382 km in Karnataka, forms the boundary between Karnataka and Andhra Pradesh for 58 km and further runs for the next 91 km in Andhra Pradesh. The confluence of Tungabhadra and Krishna River is a holy pilgrimage site – The Sangameswaram Temple. Hampi one of the important heritage locations being listed by UNESCO is on the banks of the Tungabhadra River. It is influenced chiefly by the South-West monsoon. It is a perennial river but the summer flows dwindle to as low as 2.83 to 1.42 cumec Source: The Hindu Hangul Syllabus Prelims- Environment Context: Mating calls of endangered hangul  indicate record uptick in population Background: Kashmir’s highly shy and sensitive animal, hangul, has reported one of the healthiest rutting or mating season in the previous autumn. It is the state animal of Jammu & Kashmir. About Hangul: It is the state animal of Jammu & Kashmir. The Kashmir stag also called hangul, is a subspecies of Central Asian red deer endemic to Kashmir and surrounding areas. The subspecies is battling for its survival in its last bastion: they are now scattered within 141sq km of the Dachigam National Park located on foothills of Zabarwan range on the outskirts of The Hangul was once widely distributed in the mountains of Kashmir and parts of Chamba district in neighbouring Himachal Pradesh. A survey in 2019 conducted by collaring the hangul has revealed that the species is no longer confined within the walls of Dachigam National Park. The endangered subspecies has now begun to use an old migratory route which spread through Sind Valley up to Tulail in Gurez Valley. The corridor was last known to be active in the early 1900s. As of 2023, of the 289 Hanguls, 275 are in Dachigam National Park and 14 are inTral Wildlife Sanctuary, which is considered as the second home of Hangul Conservation Status:- IUCN’s Red List:Critically Endangered. Previous Year Question Q1. What is/are unique about ‘Kharai Camel’, a breed found in India? (2016) It is capable of swimming up to three kilometres in seawater It survives by grazing on mangroves It lives in the wild and cannot be domesticated Select the correct answer using the code given below. 1 and 2 only 3 only 1 and 3 only 1, 2 and 3 Source: The Hindu NANO UREA Syllabus Prelims : Science Context: India will replace the consumption of 2.5 million tonnes of conventional urea with nano urea in FY24, the union chemicals and fertilizers minister said at a press conference recently. Background: This is in line with India’s goal of achieving self-sufficiency in urea production by 2025. About UREA: Urea is a significant component in Indian agriculture due to its high nitrogen content and affordability. Urea is the most important nitrogenous fertilizer in the country. Over use of urea leads to nitrate leaching, water contamination, and greenhouse gas emissions, soil acidification and loss of biodiversity. Nano urea has been developed to address several challenges associated with conventional urea fertilizers in agriculture. Nano Urea Nano Urea is a nanotechnology-based fertiliser that is used to provide a sufficient amount of nitrogen to plants. Development and Approval: Nano Urea is developed and patented by the Indian Farmers Fertiliser Cooperative Limited (IFFCO). It is the only nano fertilizer approved by the Government of India and included in the Fertilizer Control Order (FCO). Benefits: Energy-Efficient Production: Nano Urea is produced using an energy-efficient and environmentally friendly process with reduced carbon footprints. Increased Nutrient Availability: It enhances nutrient availability to crops by more than 80%, resulting in higher nutrient use efficiency. Improved Crop Productivity: Nano Urea is expected to improve crop yields, soil health, and the nutritional quality of produce. Addressing Imbalanced Use: It aims to address the issue of excessive use of conventional fertilizers. Nano Urea reduces nitrate leaching, water contamination, and greenhouse gas emissions compared to traditional urea. Source: iffco Previous Year Question Q1. With reference to chemical fertilizers in India, consider the following statements: At present, the retail price of chemical fertilizers is market-driven and not administered by the Government. Ammonia, which is an input of urea, is produced from natural gas. Sulphur, which is a raw material for phosphoric acid fertilizer, is a by-product of oil refineries. Which of the statements given above is/are correct? 1 only 2 and 3 only 2 only 1, 2 and 3 India - Bhutan Relations Syllabus Mains – GS 2 Context: Some sources have disclosed the presence of a Chinese backed disinformation campaign circulating false narratives about India Bhutan relationship. Background: The increasing presence of China in India’s neighbourhood is a matter of concern. Significance of Bhutan for India: Bhutan shares its borders with India and China, and its strategic location makes it an important buffer state for India’s security interests. India has provided Bhutan with assistance in areas such as defence, infrastructure, and communication, which has helped to maintain Bhutan’s sovereignty and territorial integrity. India has helped Bhutan build and maintain its border infrastructure, such as roads and bridges, to strengthen its defence capabilities and ensure its territorial integrity. In 2017, during the Doklam standoff between India and China, Bhutan played a crucial role in allowing Indian troops to enter its territory to resist Chinese incursions. India is Bhutan’s largest trading partner and Bhutan’s major export destination. Bhutan’s hydropower potential is a significant source of revenue for the country, and India has been instrumental in assisting Bhutan in developing its hydropower projects. Bhutan and India share strong cultural ties, as both countries are predominantly Buddhist. India has assisted Bhutan in preserving its cultural heritage, and many Bhutanese students come to India for higher education. Bhutan is one of the few countries in the world that has pledged to remain carbon-neutral, and India has been a key partner in helping Bhutan achieve this goal. India has assisted Bhutan in areas such as renewable energy, forest conservation, and sustainable tourism. Challenges in India-Bhutan Relations: China’s increasing presence in Bhutan, particularly along the disputed border between Bhutan and China, has raised concerns in India. India has been Bhutan’s closest ally and has played a key role in protecting Bhutan’s sovereignty and security. However, China’s growing economic and military influence in the region poses a challenge to India’s strategic interests in Bhutan. India and Bhutan share a 699 km long border, which has been largely peaceful. However, there have been some incidents of border incursions by Chinese forces in recent years. The Doklam standoff in 2017 was a major flashpoint in the India-China-Bhutan tri-junction. Any escalation of such disputes could strain India-Bhutan relations. Bhutan’s hydropower sector is a key pillar of its economy, and India has been a major partner in its development. However, there have been concerns in Bhutan over the terms of some of the hydropower projects, which have been seen as too favourable to India. This has led to some public opposition in Bhutan to Indian involvement in the sector. India is Bhutan’s largest trading partner, accounting for over 80% of Bhutan’s total imports and exports. However, there have been some concerns in Bhutan over the trade imbalance, with Bhutan importing more from India than it exports. Bhutan has been seeking greater access to the Indian market for its products, which could help to reduce the trade deficit. Source: The Hindu PRADHAN MANTRI ANUSUCHIT JAATI ABHYUDAY YOJANA(PM-AJAY) Syllabus Mains – GS 2 Context: The Ministry of Social Justice and Empowerment inaugurated and laid the foundation stone for 34 residential hostels across various states and central institutes (under PM-AJAY). Background: These hostels are specifically designed to address the accommodation requirements of Scheduled Castes (SC) and Other Backward Classes (OBC) students. This initiative aims to provide a conducive living environment, promote education, and empower these marginalized communities. About PRADHAN MANTRI ANUSUCHIT JAATI ABHYUDAY YOJANA(PM-AJAY): Pradhan Mantri Anusuchit Jaati Abhyuday Yojana (PM-AJAY) strives to reduce poverty, enhance education, and empower marginalized communities in India. PM-AJAY is a comprehensive scheme that amalgamates three centrally sponsored schemes: Pradhan Mantri Adarsh Gram Yojana (PMAGY): Focuses on developing “Adarsh Grams” where basic services are accessible to all sections of society, reducing disparities. Aims to create an environment where everyone can utilize their potential to the fullest. Provides infrastructure facilities and services necessary for dignified living. Special Central Assistance to Scheduled Castes Sub Plan (SCA to SCSP): Aims to increase income through comprehensive livelihood projects. Improves socio-economic developmental indicators by ensuring adequate infrastructure in SC-dominated villages. Encourages SC enrolment in schools by providing residential schools where needed. Babu Jagjivan Ram Chhatrawas Yojana (BJRCY): Focuses on constructing hostels for Scheduled Caste students, especially those from rural and remote areas. Provides accommodation and support for SC students pursuing education. Source: PIB Practice MCQs Daily Practice MCQs Q1.) With reference to the Bureau of Energy Efficiency (BEE), consider the following statements: The Bureau of Energy Efficiency is a statutory body. It focuses on self-regulation and market principles to enhance energy efficiency across various sectors. The State Energy Efficiency Index 2023 was released by the Bureau of Energy Efficiency in India. How many of the statements given above are correct? Only one Only two All three None Q2.) Consider the following statements about Nano Urea? It is the only nano fertilizer approved by the Government of India. Nano Urea is developed and patented by the Indian Farmers Fertiliser Cooperative Limited. Nano Urea reduces nitrate leaching and water contamination compared to traditional urea. How many of the statements given above are correct? Only one Only two All three None Q3.)Consider the following centrally sponsored schemes: Pradhan Mantri Adarsh Gram Yojana Pradhan Mantri Awas Yojna Babu Jagjivan Ram Chhatrawas Yojana How many of the above are the components of Pradhan Mantri Anusuchit Jaati Abhyuday Yojana (PM-AJAY)? Only one Only two All three None Comment the answers to the above questions in the comment section below!! ANSWERS FOR ’  4th March  2024 – Daily Practice MCQs’ will be updated along with tomorrow’s Daily Current Affairs.st ANSWERS FOR  2nd March – Daily Practice MCQs Answers- Daily Practice MCQs Q.1) – c Q.2) – d Q.3) – b

Daily Prelims CA Quiz

UPSC Quiz – 2024 : IASbaba’s Daily Current Affairs Quiz 4th March 2024

For Previous Daily Quiz (ARCHIVES) – CLICK HERE The Current Affairs questions are based on sources like ‘The Hindu’, ‘Indian Express’ and ‘PIB’, which are very important sources for UPSC Prelims Exam. The questions are focused on both the concepts and facts. The topics covered here are generally different from what is being covered under ‘Daily Current Affairs/Daily News Analysis (DNA) and Daily Static Quiz’ to avoid duplication. The questions would be published from Monday to Saturday before 2 PM. One should not spend more than 10 minutes on this initiative. Gear up and Make the Best Use of this initiative. Do remember that, “the difference between Ordinary and EXTRA-Ordinary is PRACTICE!!” Important Note: Don’t forget to post your marks in the comment section. Also, let us know if you enjoyed today’s test 🙂After completing the 5 questions, click on ‘View Questions’ to check your score, time taken, and solutions. To take the Test Click Here

[DAY 01] 60 DAY RAPID REVISION (RaRe) SERIES for UPSC Prelims 2024 – POLITY, CURRENT AFFAIRS & CSAT TEST SERIES!

Archives Hello Friends The 60 Days Rapid Revision (RaRe) Series is IASbaba’s Flagship Initiative recommended by Toppers and loved by the aspirants’ community every year. It is the most comprehensive program which will help you complete the syllabus, revise and practice tests on a daily basis. The Programme on a daily basis includes Daily Prelims MCQs from Static (Monday – Saturday) Daily Static Quiz will cover all the topics of static subjects – Polity, History, Geography, Economics, Environment and Science and technology. 20 questions will be posted daily and these questions are framed from the topics mentioned in the schedule. It will ensure timely and streamlined revision of your static subjects. Daily Current Affairs MCQs (Monday – Saturday) Daily 5 Current Affairs questions, based on sources like ‘The Hindu’, ‘Indian Express’ and ‘PIB’, would be published from Monday to Saturday according to the schedule. Daily CSAT Quiz (Monday – Friday) CSAT has been an Achilles heel for many aspirants. Daily 5 CSAT Questions will be published. Note – Daily Test of 25 static questions, 5 current affairs, and 5 CSAT questions. (35 Prelims Questions) in QUIZ FORMAT will be updated on a daily basis. To Know More about 60 Days Rapid Revision (RaRe) Series – CLICK HERE   60 Day Rapid Revision (RaRe) Series Schedule – CLICK HERE  60 Day Rapid Revision (RaRe) Series Questions & Solutions DAY 1–CLICK HERE  Important Note Comment your Scores in the Comment Section. This will keep you accountable, responsible and sincere in days to come. It will help us come out with the Cut-Off on a Daily Basis. Let us know if you enjoyed today’s test 🙂  You can post your comments in the given format  (1) Your Score (2) Matrix Meter (3) New Learning from the Test Time limit: 0 Test-summary 0 of 35 questions completed Questions: 1 2 3 4 5 6 7 8 9 10 11 12 13 14 15 16 17 18 19 20 21 22 23 24 25 26 27 28 29 30 31 32 33 34 35 Information To view Solutions, follow these instructions: Click on – ‘Start Test’ button Solve Questions Click on ‘Test Summary’ button Click on ‘Finish Test’ button Now click on ‘View Questions’ button – here you will see solutions and links. You have already completed the test before. Hence you can not start it again. Test is loading... You must sign in or sign up to start the test. You have to finish following test, to start this test: Results 0 of 35 questions answered correctly Your time: Time has elapsed You have scored 0 points out of 0 points, (0) Average score     Your score     Categories Not categorized 0% Your result has been entered into leaderboard Loading Name: E-Mail: Captcha: maximum of 70 points Pos. Name Entered on Points Result Table is loading No data available 1 2 3 4 5 6 7 8 9 10 11 12 13 14 15 16 17 18 19 20 21 22 23 24 25 26 27 28 29 30 31 32 33 34 35 Answered Review Question 1 of 35 1. Question With reference to Government of India Act 1919, Consider the following statements: The element of Dyarchy was introduced in the provinces under this, leading to the division of subjects into two categories. The Act introduced the concept of separate electorates for Sikhs, Indian Christians, Anglo-Europeans, and Europeans. The Provincial Budget was separated from the Central Budget under this act, allowing provinces to enact their own budget. How many of the above statements are correct?  a) Only one b) Only two c) All three d) None Correct Solution (c) The Government of India Act of 1919 implemented Dyarchy in the Provinces, dividing subjects into Reserved and Transferred categories. The executive council, not responsible to the legislative council, managed Reserved Subjects, while Transferred Subjects were administered by the governor with legislative council responsibility. (Hence statement 1 is correct). The Government of India Act of 1919 introduced separate electorates, reflecting Communal Representation. Sikhs, Indian Christians, Anglo-Europeans, and Europeans had distinct electorates, emphasizing communal identities in the electoral process. (Hence statement 2 is correct). The Government of India Act of 1919 established financial autonomy for provinces by separating Provincial and Central Budgets. Provinces gained the authority to formulate and enact their own budgets independently, providing a degree of fiscal decentralization. (Hence statement 3 is correct). Incorrect Solution (c) The Government of India Act of 1919 implemented Dyarchy in the Provinces, dividing subjects into Reserved and Transferred categories. The executive council, not responsible to the legislative council, managed Reserved Subjects, while Transferred Subjects were administered by the governor with legislative council responsibility. (Hence statement 1 is correct). The Government of India Act of 1919 introduced separate electorates, reflecting Communal Representation. Sikhs, Indian Christians, Anglo-Europeans, and Europeans had distinct electorates, emphasizing communal identities in the electoral process. (Hence statement 2 is correct). The Government of India Act of 1919 established financial autonomy for provinces by separating Provincial and Central Budgets. Provinces gained the authority to formulate and enact their own budgets independently, providing a degree of fiscal decentralization. (Hence statement 3 is correct). Question 2 of 35 2. Question Consider the following statements regarding Constitutionalism: It seeks to limit the power of the government. The rule of law, a key component of constitutionalism, emphasizes the supremacy of individuals over the law. The concept of the rule of law ensures that everyone, including the government, is subject to the law. How many of the above statements are correct?  a) Only one b) Only two c) All three d) None Correct Solution (b) Constitutionalism is a political philosophy emphasizing the importance of limiting government authority through a defined set of laws or a constitution. It aims to prevent arbitrary rule, safeguard individual rights, and establish a framework that outlines the powers and responsibilities of the government. By establishing a constitution, a government’s authority becomes subject to the principles and rules laid out in that constitution, ensuring a system of checks and balances. (Hence statement 1 is correct). The rule of law emphasizes that no one, including individuals and the government, is above the law. It signifies that the law is supreme and applies equally to all, ensuring fairness, justice, and accountability. Rather than emphasizing the supremacy of individuals, the rule of law places importance on the equal application of legal principles to protect individual rights and prevent arbitrary actions by those in authority. Constitutionalism and the rule of law work in tandem to create a just and accountable governance structure. (Hence statement 2 is incorrect). The rule of law is a foundational principle in constitutionalism that establishes the idea that all individuals, including government officials, are subject to and accountable under the law. It prevents the concentration of power in the hands of a few by ensuring that legal principles and standards apply universally. (Hence statement 3 is correct). Incorrect Solution (b) Constitutionalism is a political philosophy emphasizing the importance of limiting government authority through a defined set of laws or a constitution. It aims to prevent arbitrary rule, safeguard individual rights, and establish a framework that outlines the powers and responsibilities of the government. By establishing a constitution, a government’s authority becomes subject to the principles and rules laid out in that constitution, ensuring a system of checks and balances. (Hence statement 1 is correct). The rule of law emphasizes that no one, including individuals and the government, is above the law. It signifies that the law is supreme and applies equally to all, ensuring fairness, justice, and accountability. Rather than emphasizing the supremacy of individuals, the rule of law places importance on the equal application of legal principles to protect individual rights and prevent arbitrary actions by those in authority. Constitutionalism and the rule of law work in tandem to create a just and accountable governance structure. (Hence statement 2 is incorrect). The rule of law is a foundational principle in constitutionalism that establishes the idea that all individuals, including government officials, are subject to and accountable under the law. It prevents the concentration of power in the hands of a few by ensuring that legal principles and standards apply universally. (Hence statement 3 is correct). Question 3 of 35 3. Question With reference to different forms of Government, Consider the following statements: Theocracy is a form of government where a specific religious ideology determines leadership with religious clergy occupying key leadership roles. In a theocracy, there is often little to no distinction between scriptural laws and legal codes. Oligarchies are forms of government where a small group of individuals rules over a nation, with power often based on qualities like wealth, heredity, and race. How many of the above statements are correct?  a) Only one b) Only two c) All three d) None Correct Solution (c) Theocracy is a form of government where a specific religious ideology dictates leadership, laws, and customs. In such systems, religious clergy may hold key leadership roles, including the highest office in the nation. For example, Iran operates as a theocracy with an Islamic system of governance led by religious figures. (Hence statement 1 is correct). In theocratic systems, there is often minimal distinction between scriptural laws and legal codes. The laws are heavily influenced by religious doctrines, and religious authorities play a significant role in interpreting and implementing them within the legal framework. This integration of religious principles into the legal system can be observed in various theocratic states. (Hence statement 2 is correct). Oligarchies are forms of government where a small group of individuals holds power over a nation. The selection of this group is often based on specific qualities such as wealth, heredity, or race. In oligarchies, there tends to be an absence of democratic practices, and decision-making authority is concentrated in the hands of this privileged minority. Oligarchies may have authoritative rulers, and individual rights can be limited. (Hence statement 3 is correct). Incorrect Solution (c) Theocracy is a form of government where a specific religious ideology dictates leadership, laws, and customs. In such systems, religious clergy may hold key leadership roles, including the highest office in the nation. For example, Iran operates as a theocracy with an Islamic system of governance led by religious figures. (Hence statement 1 is correct). In theocratic systems, there is often minimal distinction between scriptural laws and legal codes. The laws are heavily influenced by religious doctrines, and religious authorities play a significant role in interpreting and implementing them within the legal framework. This integration of religious principles into the legal system can be observed in various theocratic states. (Hence statement 2 is correct). Oligarchies are forms of government where a small group of individuals holds power over a nation. The selection of this group is often based on specific qualities such as wealth, heredity, or race. In oligarchies, there tends to be an absence of democratic practices, and decision-making authority is concentrated in the hands of this privileged minority. Oligarchies may have authoritative rulers, and individual rights can be limited. (Hence statement 3 is correct). Question 4 of 35 4. Question With reference to Committees of Constituent Assembly, Consider the following statements: Dr. Rajendra Prasad chaired the Advisory Committee on Fundamental Rights, Minorities, and Tribal and Excluded Areas in the Constituent Assembly. Jawaharlal Nehru chaired both the Union Powers Committee and the States Committee in the Constituent Assembly. S.K. Dar was a member of the Constituent Assembly and headed the Linguistic Provinces Commission. How many of the above statements are correct?  a) Only one b) Only two c) All three d) None Correct Solution (a) While Dr. Rajendra Prasad made significant contributions to the Constituent Assembly, it was Sardar Patel who chaired the Advisory Committee on Fundamental Rights, Minorities, and Tribal and Excluded Areas. Sardar Patel’s leadership in this committee demonstrated his commitment to addressing critical issues related to fundamental rights and the protection of minority and tribal communities. (Hence statement 3 is incorrect). Jawaharlal Nehru’s dual leadership in the Union Powers Committee and the States Committee showcased his comprehensive understanding of the intricacies involved in both central and state governance. This dual responsibility reflects Nehru’s instrumental role in negotiating and defining the distribution of powers between the Union and the States, contributing significantly to the constitutional framework that emerged from the Constituent Assembly’s deliberations. (Hence statement 2 is correct). S.K. Dar was not a member of the Constituent Assembly but headed the Linguistic Provinces Commission. The presence of S.K. Dar’s direct involvement in these roles indicates that the leadership of the Linguistic Provinces Commission was entrusted to someone else within the Constituent Assembly, highlighting the diverse contributions of various members to the formation of linguistic provinces in post-independence India. (Hence statement 3 is incorrect). Important Points/ Value Additions: No. Committee Name Chairman 1 Union Powers Committee Jawaharlal Nehru 2 Union Constitution Committee Jawaharlal Nehru 3 Provincial Constitution Committee Sardar Patel 4 Drafting Committee Dr. B.R. Ambedkar 5 Advisory Committee on Fundamental Rights, Minorities, and Tribal and Excluded Areas Sardar Patel 6 Rules of Procedure Committee Dr. Rajendra Prasad 7 States Committee (Committee for Negotiating with States) Jawaharlal Nehru 8 Steering Committee Dr. Rajendra Prasad Incorrect Solution (a) While Dr. Rajendra Prasad made significant contributions to the Constituent Assembly, it was Sardar Patel who chaired the Advisory Committee on Fundamental Rights, Minorities, and Tribal and Excluded Areas. Sardar Patel’s leadership in this committee demonstrated his commitment to addressing critical issues related to fundamental rights and the protection of minority and tribal communities. (Hence statement 3 is incorrect). Jawaharlal Nehru’s dual leadership in the Union Powers Committee and the States Committee showcased his comprehensive understanding of the intricacies involved in both central and state governance. This dual responsibility reflects Nehru’s instrumental role in negotiating and defining the distribution of powers between the Union and the States, contributing significantly to the constitutional framework that emerged from the Constituent Assembly’s deliberations. (Hence statement 2 is correct). S.K. Dar was not a member of the Constituent Assembly but headed the Linguistic Provinces Commission. The presence of S.K. Dar’s direct involvement in these roles indicates that the leadership of the Linguistic Provinces Commission was entrusted to someone else within the Constituent Assembly, highlighting the diverse contributions of various members to the formation of linguistic provinces in post-independence India. (Hence statement 3 is incorrect). Important Points/ Value Additions: No. Committee Name Chairman 1 Union Powers Committee Jawaharlal Nehru 2 Union Constitution Committee Jawaharlal Nehru 3 Provincial Constitution Committee Sardar Patel 4 Drafting Committee Dr. B.R. Ambedkar 5 Advisory Committee on Fundamental Rights, Minorities, and Tribal and Excluded Areas Sardar Patel 6 Rules of Procedure Committee Dr. Rajendra Prasad 7 States Committee (Committee for Negotiating with States) Jawaharlal Nehru 8 Steering Committee Dr. Rajendra Prasad Question 5 of 35 5. Question With reference to Constituent Assembly, Consider the following statements: The first meeting of the Constituent Assembly was held on 9th October 1946. Dr. Sachidanand Sinha was elected as the temporary president of the Constituent Assembly. The Present Preamble of the Indian Constitution is a modified version of the Objective Resolution moved by Jawaharlal Nehru. How many of the above statements are correct? a) Only one b) Only two c) All three d) None Correct Solution (b) The first Constituent Assembly meeting was on Dec 09, 1946. It predates India’s independence on Aug 15, 1947. The Muslim League boycotted the initial gathering attended by 211 members. (Hence statement 1 is incorrect). Dr. Sachidanand Sinha played a pivotal role in the Constituent Assembly of India by being elected as its temporary president during its initial sessions. This esteemed position required him to preside over the assembly’s meetings until the permanent president, Dr. Rajendra Prasad, could assume office. (Hence statement 2 is correct). Jawaharlal Nehru’s Objective Resolution, moved on Dec 13, 1946, laid the constitutional groundwork. The present Preamble is a modified version, reflecting its foundational principles. (Hence statement 3 is correct). Incorrect Solution (b) The first Constituent Assembly meeting was on Dec 09, 1946. It predates India’s independence on Aug 15, 1947. The Muslim League boycotted the initial gathering attended by 211 members. (Hence statement 1 is incorrect). Dr. Sachidanand Sinha played a pivotal role in the Constituent Assembly of India by being elected as its temporary president during its initial sessions. This esteemed position required him to preside over the assembly’s meetings until the permanent president, Dr. Rajendra Prasad, could assume office. (Hence statement 2 is correct). Jawaharlal Nehru’s Objective Resolution, moved on Dec 13, 1946, laid the constitutional groundwork. The present Preamble is a modified version, reflecting its foundational principles. (Hence statement 3 is correct). Question 6 of 35 6. Question Consider the following statements: The Indian Constitution borrowed the Federal Scheme and Emergency Provisions from the Government of India Act, 1935. The concept of Judicial review has been taken from the British Constitution. The Weimar Constitution of Germany influenced the provision for the suspension of Fundamental Rights during an Emergency in the Indian Constitution. How many of the above statements are correct?  a) Only one b) Only two c) All three d) None Correct Solution (b) The Indian Constitution drew heavily from the Government of India Act, 1935, adopting its federal structure, Governor’s office, and provisions related to emergencies. The Act laid the groundwork for the constitutional framework, shaping the distribution of powers between the center and states, the role of Governors, and the provisions for declaring emergencies, which were later incorporated into the Indian Constitution. (Hence statement 1 is correct). The concept of judicial review has not been taken directly from the British Constitution. While the British legal system does not have a codified constitution like the United States, the idea of judicial review has historical roots in British legal principles. However, the development and formalization of judicial review, as a specific and potent power of the judiciary to review and potentially invalidate government actions, is often attributed to the United States, particularly through the landmark case of Marbury v. Madison (1803). (Hence statement 2 is incorrect). The Weimar Constitution’s impact on the Indian Constitution is evident in the provision allowing the suspension of Fundamental Rights during emergencies. This provision reflects the lessons learned from the Weimar Republic’s challenges and the need to address exceptional situations. India’s constitutional framers incorporated this feature to provide the government with necessary powers during emergencies while establishing safeguards to prevent misuse, striking a delicate balance between order and liberty. (Hence statement 3 is correct). Incorrect Solution (b) The Indian Constitution drew heavily from the Government of India Act, 1935, adopting its federal structure, Governor’s office, and provisions related to emergencies. The Act laid the groundwork for the constitutional framework, shaping the distribution of powers between the center and states, the role of Governors, and the provisions for declaring emergencies, which were later incorporated into the Indian Constitution. (Hence statement 1 is correct). The concept of judicial review has not been taken directly from the British Constitution. While the British legal system does not have a codified constitution like the United States, the idea of judicial review has historical roots in British legal principles. However, the development and formalization of judicial review, as a specific and potent power of the judiciary to review and potentially invalidate government actions, is often attributed to the United States, particularly through the landmark case of Marbury v. Madison (1803). (Hence statement 2 is incorrect). The Weimar Constitution’s impact on the Indian Constitution is evident in the provision allowing the suspension of Fundamental Rights during emergencies. This provision reflects the lessons learned from the Weimar Republic’s challenges and the need to address exceptional situations. India’s constitutional framers incorporated this feature to provide the government with necessary powers during emergencies while establishing safeguards to prevent misuse, striking a delicate balance between order and liberty. (Hence statement 3 is correct). Question 7 of 35 7. Question With reference to Constitutional Morality, Consider the following statements: It means adherence to or being faithful to bottom line principles of the constitutional values. In the Naz Foundation case, the Supreme Court opined that only Constitutional Morality and not Public Morality should prevail. Which of the statements given above is/are not correct? a) 1 only b) 2 only c) Both 1 and 2 d) Neither 1 nor 2 Correct Solution (d) Constitutional Morality involves a commitment to the core principles and values enshrined in the constitution, ensuring that actions align with the fundamental ideals of justice, equality, and democracy, forming the bedrock of the constitutional framework. (Hence statement 1 is correct). In the Naz Foundation case, the Supreme Court emphasized that legal judgments should be guided by Constitutional Morality, prioritizing fundamental rights and constitutional principles over subjective public morality to protect individual liberties, especially in matters relating to personal freedoms and equality. (Hence statement 2 is correct). Incorrect Solution (d) Constitutional Morality involves a commitment to the core principles and values enshrined in the constitution, ensuring that actions align with the fundamental ideals of justice, equality, and democracy, forming the bedrock of the constitutional framework. (Hence statement 1 is correct). In the Naz Foundation case, the Supreme Court emphasized that legal judgments should be guided by Constitutional Morality, prioritizing fundamental rights and constitutional principles over subjective public morality to protect individual liberties, especially in matters relating to personal freedoms and equality. (Hence statement 2 is correct). Question 8 of 35 8. Question Consider the following statements with reference to Preamble: Kesavananda Bharati Case dealt with whether the preamble can be amended or not. The Preamble has been amended only once by the 52nd Constitutional Amendment Act. The original preamble described the state as a “Sovereign Democratic Republic”. How many of the above statements are correct?  a) Only one b) Only two c) All three d) None Correct Solution (b) The Kesavananda Bharati Case addressed the question of whether the Preamble could be amended. The Supreme Court held that while the Preamble is a part of the Constitution, amendments must not violate the Constitution’s basic structure, ensuring its core principles remain intact.  (Hence statement 1 is correct). The Preamble to the Indian Constitution has not been amended by the 52nd Constitutional Amendment Act. The 52nd Amendment Act, enacted in 1985, primarily dealt with anti-defection laws and did not involve any changes to the Preamble. The Preamble was amended by the 42nd Constitutional Amendment Act in 1976, introducing the words “Socialist”, “Secular”, and “Integrity” to the original Preamble.(Hence statement 2 is incorrect). The original Preamble indeed characterized the state as a “Sovereign Democratic Republic,” emphasizing key principles of democracy and sovereignty in the nation’s governance. (Hence statement 3 is correct)   Incorrect Solution (b) The Kesavananda Bharati Case addressed the question of whether the Preamble could be amended. The Supreme Court held that while the Preamble is a part of the Constitution, amendments must not violate the Constitution’s basic structure, ensuring its core principles remain intact.  (Hence statement 1 is correct). The Preamble to the Indian Constitution has not been amended by the 52nd Constitutional Amendment Act. The 52nd Amendment Act, enacted in 1985, primarily dealt with anti-defection laws and did not involve any changes to the Preamble. The Preamble was amended by the 42nd Constitutional Amendment Act in 1976, introducing the words “Socialist”, “Secular”, and “Integrity” to the original Preamble.(Hence statement 2 is incorrect). The original Preamble indeed characterized the state as a “Sovereign Democratic Republic,” emphasizing key principles of democracy and sovereignty in the nation’s governance. (Hence statement 3 is correct)   Question 9 of 35 9. Question Consider the following statements: The Act ended the trade monopoly of the East India Company in India, except the company’s monopoly in trade with China and trade in tea. According to the act, companies should invest Rs. 1 Lakh every year on the education of Indians. Which of the following act has been described above? a) Charter Act of 1793 b) Charter Act of 1833 c) Charter Act of 1813 d) Charter Act of 1853 Correct Solution (c) The Charter Act of 1813 marked the termination of the East India Company’s monopoly in India. However, the company retained its monopoly in trade with China and the tea trade with India. The act extended the company’s rule in India for an additional 20 years, reaffirming British control over Indian territories. Individuals, particularly Christian Missionaries, were granted permission to travel to India to contribute to moral and religious advancements. The act introduced regulations on the company’s territorial revenues and commercial profits, emphasizing the separation of territorial and commercial accounts. The company’s dividend was fixed at 10.5% per annum, providing stability in financial returns. A provision mandated the Company to invest Rs. 1 Lakh annually in the education of Indians, reflecting an early focus on educational initiatives. The act empowered Local Governments in India with the authority to impose taxes on individuals and enforce penalties on those who failed to pay, enhancing local fiscal governance. Incorrect Solution (c) The Charter Act of 1813 marked the termination of the East India Company’s monopoly in India. However, the company retained its monopoly in trade with China and the tea trade with India. The act extended the company’s rule in India for an additional 20 years, reaffirming British control over Indian territories. Individuals, particularly Christian Missionaries, were granted permission to travel to India to contribute to moral and religious advancements. The act introduced regulations on the company’s territorial revenues and commercial profits, emphasizing the separation of territorial and commercial accounts. The company’s dividend was fixed at 10.5% per annum, providing stability in financial returns. A provision mandated the Company to invest Rs. 1 Lakh annually in the education of Indians, reflecting an early focus on educational initiatives. The act empowered Local Governments in India with the authority to impose taxes on individuals and enforce penalties on those who failed to pay, enhancing local fiscal governance. Question 10 of 35 10. Question With reference to the sources of the Indian Constitution, consider the following statements: The Principle of ‘Procedure established by law’ was borrowed from the British Constitution. ‘The Rule of Law’ was borrowed from the American Constitution. Federal features with a strong Centre are borrowed from the Canadian Constitution. How many of the above statements are correct?  a) Only one b) Only two c) All three d) None Correct Solution (a) The Indian constitution has adopted the principle of ‘Procedure established by law’ from the Japanese constitution. However, the doctrine of ‘Due process of law’ finds its origin in the constitution of the USA. (Hence statement 1 is incorrect) ‘Rule of law’ has been adopted from the British Constitution. The rule of law is the political philosophy that all citizens and institutions within a country, state or community are accountable to the same set of laws, including lawmakers and leaders. (Hence statement 2 is incorrect) India has adopted various provisions from the Canadian constitution which have made the Indian federal structure biased towards center like the Federation with a strong center, residuary powers with the center and appointment of state governors by the Centre. (Hence statement 3 is correct) Incorrect Solution (a) The Indian constitution has adopted the principle of ‘Procedure established by law’ from the Japanese constitution. However, the doctrine of ‘Due process of law’ finds its origin in the constitution of the USA. (Hence statement 1 is incorrect) ‘Rule of law’ has been adopted from the British Constitution. The rule of law is the political philosophy that all citizens and institutions within a country, state or community are accountable to the same set of laws, including lawmakers and leaders. (Hence statement 2 is incorrect) India has adopted various provisions from the Canadian constitution which have made the Indian federal structure biased towards center like the Federation with a strong center, residuary powers with the center and appointment of state governors by the Centre. (Hence statement 3 is correct) Question 11 of 35 11. Question Consider the following statements regarding Union Territories (UTs): UTs are listed in Schedule I of the Constitution of India. All UTs have legislative assemblies. The Ministry of Home Affairs is responsible for matters concerning UTs in India. Himachal Pradesh, Manipur, Tripura, Mizoram, Arunachal Pradesh, and Goa were formerly UTs that have attained statehood. How many of the above statements are not correct?  a) Only one b) Only two c) Only three d) All four Correct Solution (a) Schedule I of the Indian Constitution outlines the names of Union Territories. These territories, while not fully sovereign entities like states, have varying degrees of autonomy and administration under the Central government. Their governance structures differ, some having legislative assemblies while others are directly administered by the President of India or administrators appointed by the Central government. (Hence statement 1 is correct) Not all Union Territories have legislative assemblies. As of the latest update in 2023, only some Union Territories like Delhi, Puducherry, and Jammu & Kashmir have legislative assemblies, allowing them a degree of self-governance and lawmaking power. (Hence statement 2 is incorrect) The Ministry of Home Affairs is the nodal ministry responsible for handling various administrative and governance-related matters concerning Union territories. It oversees the appointment of administrators or Lieutenant Governors and manages matters related to security, law and order, and overall administration in these territories. (Hence statement 3 is correct) These states were formerly Union Territories before they attained full statehood. Each went through a process where, after a period of being administered directly by the Central government, they gained statehood with their own elected governments, legislative assemblies, and more autonomy in decision-making within the framework of the Indian Constitution. (Hence statement 4 is correct) Incorrect Solution (a) Schedule I of the Indian Constitution outlines the names of Union Territories. These territories, while not fully sovereign entities like states, have varying degrees of autonomy and administration under the Central government. Their governance structures differ, some having legislative assemblies while others are directly administered by the President of India or administrators appointed by the Central government. (Hence statement 1 is correct) Not all Union Territories have legislative assemblies. As of the latest update in 2023, only some Union Territories like Delhi, Puducherry, and Jammu & Kashmir have legislative assemblies, allowing them a degree of self-governance and lawmaking power. (Hence statement 2 is incorrect) The Ministry of Home Affairs is the nodal ministry responsible for handling various administrative and governance-related matters concerning Union territories. It oversees the appointment of administrators or Lieutenant Governors and manages matters related to security, law and order, and overall administration in these territories. (Hence statement 3 is correct) These states were formerly Union Territories before they attained full statehood. Each went through a process where, after a period of being administered directly by the Central government, they gained statehood with their own elected governments, legislative assemblies, and more autonomy in decision-making within the framework of the Indian Constitution. (Hence statement 4 is correct) Question 12 of 35 12. Question With reference to the Objectives Resolution, consider the following statements: It was moved by Pt. Jawaharlal Nehru on 15th August 1946. It proclaimed that the power of Sovereign Independent India is derived from the people. The Preamble of the Indian Constitution is a modified version of it. How many of the above statements are correct?  a) Only one b) Only two c) All three d) None Correct Solution (b) On December 13, 1946, Jawaharlal Nehru moved the historic ‘Objectives Resolution’. It laid down the fundamentals and philosophy of the constitutional structure. (Hence statement 1 is incorrect) It proclaimed that all power and authority of the Sovereign Independent India, its constituent parts, and organs of government, are derived from the people. (Hence statement 2 is correct) This Resolution was unanimously adopted by the Constituent Assembly on January 22, 1947. It influenced the eventual shaping of the Constitution through all its subsequent stages. Its modified version forms the Preamble of the present Indian Constitution. (Hence statement 3 is correct) Incorrect Solution (b) On December 13, 1946, Jawaharlal Nehru moved the historic ‘Objectives Resolution’. It laid down the fundamentals and philosophy of the constitutional structure. (Hence statement 1 is incorrect) It proclaimed that all power and authority of the Sovereign Independent India, its constituent parts, and organs of government, are derived from the people. (Hence statement 2 is correct) This Resolution was unanimously adopted by the Constituent Assembly on January 22, 1947. It influenced the eventual shaping of the Constitution through all its subsequent stages. Its modified version forms the Preamble of the present Indian Constitution. (Hence statement 3 is correct) Question 13 of 35 13. Question Consider the following statements about the Constitution of India? It generates a degree of trust and coordination among people. It specifies how the government will be constituted. It lays down limits on the powers of the government. It provides a standard to examine and evaluate any law and action. How many of the above statements are correct?  a) Only one b) Only two c) Only three d) All four Correct Solution (d) The Constitution of a country is a set of written rules that are accepted by all people living together in a country. Constitution is the supreme law that determines the relationship among people living in a territory (called citizens) and also the relationship between the people and government. It draws a structure that defines fundamental political principles, forms the framework, procedures, powers, and duties of government institutions and lays out Fundamental Rights, Directive Principles and Duties of citizens It generates a degree of trust and coordination that is necessary for different kind of people to live together. (Hence statement 1 is correct) It specifies how the government will be constituted, who will have power to take which decisions. (Hence statement 2 is correct) It lays down limits on the powers of the government and tells us what the rights of the citizens are. (Hence statement 3 is correct) It expresses the aspirations of the people creating a good society. It provides a standard to examine and evaluate any law and action of government, to find out whether it is good or bad. (Hence statement 4 is correct) The Constitution of India is the longest written Constitution of any sovereign country in the world. Dr. B.R. Ambedkar is the chief architect of the Indian Constitution. Incorrect Solution (d) The Constitution of a country is a set of written rules that are accepted by all people living together in a country. Constitution is the supreme law that determines the relationship among people living in a territory (called citizens) and also the relationship between the people and government. It draws a structure that defines fundamental political principles, forms the framework, procedures, powers, and duties of government institutions and lays out Fundamental Rights, Directive Principles and Duties of citizens It generates a degree of trust and coordination that is necessary for different kind of people to live together. (Hence statement 1 is correct) It specifies how the government will be constituted, who will have power to take which decisions. (Hence statement 2 is correct) It lays down limits on the powers of the government and tells us what the rights of the citizens are. (Hence statement 3 is correct) It expresses the aspirations of the people creating a good society. It provides a standard to examine and evaluate any law and action of government, to find out whether it is good or bad. (Hence statement 4 is correct) The Constitution of India is the longest written Constitution of any sovereign country in the world. Dr. B.R. Ambedkar is the chief architect of the Indian Constitution. Question 14 of 35 14. Question Parliamentary System of India is inspired from British Parliamentary System, but it is different in the sense that: India has a republican system in place of British monarchical system. Indian system is not based on the doctrine of the sovereignty of Parliament. Which of the above statements is/are correct? a) 1 only b) 2 only c) Both 1 and 2 d) Neither 1 nor 2 Correct Solution (c) The parliamentary system of government in India is largely based on the British parliamentary system. However, it never became a replica of the British system and differs in the following respects: India has a republican system in place of British monarchical system. In other words, the Head of the State in India (that is, President) is elected, while the Head of the State in Britain (that is, King or Queen) enjoys a hereditary position. (Hence statement 1 is correct) The British system is based on the doctrine of the sovereignty of Parliament, while the Parliament is not supreme in India and enjoys limited and restricted powers due to a written Constitution, federal system, judicial review and Fundamental Rights. (Hence statement 2 is correct) In Britain, the Prime Minister should be a member of the Lower House (House of Commons) of the Parliament. In India, the Prime Minister may be a member of any of the two Houses of Parliament. Usually, the members of Parliament alone are appointed as Ministers in Britain. In India, a person who is not a Member of Parliament can also be appointed as minister, but for a maximum period of six months. Britain has the system of legal responsibility of the minister while India has no such system. Unlike in Britain, the ministers in India are not required to countersign the official acts of the Head of the State. ‘Shadow cabinet’ is a unique institution of the British cabinet system. It is formed by the opposition party to balance the ruling cabinet and to prepare its members for future ministerial office. There is no such institution in India. Incorrect Solution (c) The parliamentary system of government in India is largely based on the British parliamentary system. However, it never became a replica of the British system and differs in the following respects: India has a republican system in place of British monarchical system. In other words, the Head of the State in India (that is, President) is elected, while the Head of the State in Britain (that is, King or Queen) enjoys a hereditary position. (Hence statement 1 is correct) The British system is based on the doctrine of the sovereignty of Parliament, while the Parliament is not supreme in India and enjoys limited and restricted powers due to a written Constitution, federal system, judicial review and Fundamental Rights. (Hence statement 2 is correct) In Britain, the Prime Minister should be a member of the Lower House (House of Commons) of the Parliament. In India, the Prime Minister may be a member of any of the two Houses of Parliament. Usually, the members of Parliament alone are appointed as Ministers in Britain. In India, a person who is not a Member of Parliament can also be appointed as minister, but for a maximum period of six months. Britain has the system of legal responsibility of the minister while India has no such system. Unlike in Britain, the ministers in India are not required to countersign the official acts of the Head of the State. ‘Shadow cabinet’ is a unique institution of the British cabinet system. It is formed by the opposition party to balance the ruling cabinet and to prepare its members for future ministerial office. There is no such institution in India. Question 15 of 35 15. Question Considering the Preamble of Indian Constitution, India is a Secular State which means: The State is anti-religion and condemns all religions. The Preamble secures to all citizens of India liberty of belief, faith and worship. The State is neutral in the matter of religion and does not uphold any particular religion as the State religion. How many of the above statements are correct?  a) Only one b) Only two c) All three d) None Correct Solution (b) The Indian Constitution embodies the positive concept of secularism that is, all religions in our country (irrespective of their strength) shall have the same status and support from the state. Atheistic State is anti-religion and hence condemns all religions and it is not a secular State. (Hence statement 1 is incorrect) The Preamble secures to all citizens of India liberty of thought, expression, belief, faith and worship. (Hence statement 2 is correct) The State is neutral in the matter of religion and does not uphold any particular religion as the state religion. (Hence statement 3 is correct) Incorrect Solution (b) The Indian Constitution embodies the positive concept of secularism that is, all religions in our country (irrespective of their strength) shall have the same status and support from the state. Atheistic State is anti-religion and hence condemns all religions and it is not a secular State. (Hence statement 1 is incorrect) The Preamble secures to all citizens of India liberty of thought, expression, belief, faith and worship. (Hence statement 2 is correct) The State is neutral in the matter of religion and does not uphold any particular religion as the state religion. (Hence statement 3 is correct) Question 16 of 35 16. Question With respect to Article 3 of the Constitution, consider the following statements: A bill under Article 3 can be introduced in the Parliament only with the prior recommendation of the President. The President has to refer the bill under Article 3 to state legislature for expressing views. Views expressed by the state legislature are binding. How many of the above statements are correct?  a) Only one b) Only two c) All three d) None Correct Solution (b) Article 3 of the Indian Constitution deals with the formation of new States and alteration of areas, boundaries or names of existing States. In this regard, the Parliament may by law: Form a new State by separation of territory from any State or by uniting two or more States or parts of States, or by uniting any territory to a part of any State. Increase the area of any State. Diminish the area of any State. Alter the boundaries of any State. Alter the name of any State. Article 3 lays down two conditions in this regard: one, a bill contemplating the above changes can be introduced in the Parliament only with the prior recommendation of the President; and two, before recommending the bill, the President has to refer the same to the state legislature concerned for expressing its views within a specified period. (Hence statements 1 and 2 are correct) Further, the power of parliament to form new states includes the power to form a new state or union territory by uniting a part of any state or union territory to any other or union territory. The President (or Parliament) is not bound by the views of the state legislature and may either accept or reject them, even if the views are received in time. (Hence statement 3 is incorrect) Incorrect Solution (b) Article 3 of the Indian Constitution deals with the formation of new States and alteration of areas, boundaries or names of existing States. In this regard, the Parliament may by law: Form a new State by separation of territory from any State or by uniting two or more States or parts of States, or by uniting any territory to a part of any State. Increase the area of any State. Diminish the area of any State. Alter the boundaries of any State. Alter the name of any State. Article 3 lays down two conditions in this regard: one, a bill contemplating the above changes can be introduced in the Parliament only with the prior recommendation of the President; and two, before recommending the bill, the President has to refer the same to the state legislature concerned for expressing its views within a specified period. (Hence statements 1 and 2 are correct) Further, the power of parliament to form new states includes the power to form a new state or union territory by uniting a part of any state or union territory to any other or union territory. The President (or Parliament) is not bound by the views of the state legislature and may either accept or reject them, even if the views are received in time. (Hence statement 3 is incorrect) Question 17 of 35 17. Question With reference to the State Reorganization Commission 1953, consider the following statements: It was headed by Sardar Patel. In its report, rejected the theory of ‘one language– one state’. The commission suggested the abolition of the fourfold classification of states under the original Constitution. How many of the above statements are correct?  a) Only one b) Only two c) All three d) None Correct Solution (b) Explanation: State Reorganization Commission: Fazl Ali Commission also called the States Reorganization Commission was set up to investigate whether the linguistic basis for state separation can be considered or not. The creation of Andhra state intensified the demand from other regions for creation of states on linguistic basis. This forced the Government of India to appoint in December 1953, a three-member States Re-organisation Commission under the chairmanship of Fazl Ali to reexamine the whole question. Its other two members were K M Panikkar and H N Kunzru. (Hence statement 1 is incorrect) It submitted its report in September 1955 and broadly accepted language as the basis of re-organisation of states. But, it rejected the theory of ‘one language– one state’. Its view was that the unity of India should be regarded as the primary consideration in any redrawing of the country’s political units. It identified four major factors that can be taken into account in any scheme of re-organisation of states: (Hence statement 2 is correct) Preservation and strengthening of the unity and security of the country. Linguistic and cultural homogeneity. Financial, economic and administrative considerations. Planning and promotion of the welfare of the people in each state as well as of the nation as a whole. The commission suggested the abolition of the fourfold classification of states under the original Constitution and creation of 16 states and 3 centrally administered territories. (Hence statement 3 is correct) Incorrect Solution (b) Explanation: State Reorganization Commission: Fazl Ali Commission also called the States Reorganization Commission was set up to investigate whether the linguistic basis for state separation can be considered or not. The creation of Andhra state intensified the demand from other regions for creation of states on linguistic basis. This forced the Government of India to appoint in December 1953, a three-member States Re-organisation Commission under the chairmanship of Fazl Ali to reexamine the whole question. Its other two members were K M Panikkar and H N Kunzru. (Hence statement 1 is incorrect) It submitted its report in September 1955 and broadly accepted language as the basis of re-organisation of states. But, it rejected the theory of ‘one language– one state’. Its view was that the unity of India should be regarded as the primary consideration in any redrawing of the country’s political units. It identified four major factors that can be taken into account in any scheme of re-organisation of states: (Hence statement 2 is correct) Preservation and strengthening of the unity and security of the country. Linguistic and cultural homogeneity. Financial, economic and administrative considerations. Planning and promotion of the welfare of the people in each state as well as of the nation as a whole. The commission suggested the abolition of the fourfold classification of states under the original Constitution and creation of 16 states and 3 centrally administered territories. (Hence statement 3 is correct) Question 18 of 35 18. Question Consider the following statements about Citizenship of India: The children of foreign diplomats posted in India can acquire Indian citizenship by birth. If any foreign territory becomes part of India, the government of India specifies who among the people of territory shall be citizen of India. When an Indian citizen voluntarily acquires citizenship of another country, his Indian citizenship is automatically terminated. How many of the above statements are correct?  a) Only one b) Only two c) All three d) None Correct Solution (b) Explanation: A person born in India on or after 26th January 1950 but before 1st July 1987 is a citizen of India by birth irrespective of the nationality of his parents. A person born in India on or after 1st July 1987 is considered as a citizen of India only if either of his parents is a citizen of India at the time of his birth. Further, those born in India on or after 3rd December 2004 are considered citizens of India only if both of their parents are citizens of India or one of those parents is a citizen of India and the other is not an illegal migrant at the time of their birth. The children of foreign diplomats posted in India and enemy aliens cannot acquire Indian citizenship by birth. (Hence statement 1 is incorrect) If any foreign territory becomes a part of India, the government of India specifies the persons who among the people of the territory shall be citizen of India. Such persons become the citizens of India from the notified date. (Hence statement 2 is correct) When an Indian citizen voluntarily (consciously, knowingly and without duress, undue influence or compulsion) acquires the citizenship of another country, his Indian citizenship automatically terminates. This provision, however, does not apply during a war in which India is engaged. (Hence statement 3 is correct) Incorrect Solution (b) Explanation: A person born in India on or after 26th January 1950 but before 1st July 1987 is a citizen of India by birth irrespective of the nationality of his parents. A person born in India on or after 1st July 1987 is considered as a citizen of India only if either of his parents is a citizen of India at the time of his birth. Further, those born in India on or after 3rd December 2004 are considered citizens of India only if both of their parents are citizens of India or one of those parents is a citizen of India and the other is not an illegal migrant at the time of their birth. The children of foreign diplomats posted in India and enemy aliens cannot acquire Indian citizenship by birth. (Hence statement 1 is incorrect) If any foreign territory becomes a part of India, the government of India specifies the persons who among the people of the territory shall be citizen of India. Such persons become the citizens of India from the notified date. (Hence statement 2 is correct) When an Indian citizen voluntarily (consciously, knowingly and without duress, undue influence or compulsion) acquires the citizenship of another country, his Indian citizenship automatically terminates. This provision, however, does not apply during a war in which India is engaged. (Hence statement 3 is correct) Question 19 of 35 19. Question With reference to the “Act for the Good Government of India”, consider the following statements: The act abolished the East India Company, and transferred the powers of government, territories and revenues to the British Crown. It ended the system of double government by abolishing the Board of Control and Court of Directors. It created a new office of the Viceroy of India, who was the member of the British Cabinet. How many of the above statements are correct?  a) Only one b) Only two c) All three d) None Correct Solution (b) Government of India Act, 1858 (Act for the Good Government of India) This act known as the Act for the Good Government of India, abolished the East India Company, and transferred the powers of government, territories and revenues to the British Crown (Hence statement 1 is correct) It provided that India henceforth was to be governed by, and in the name of, Her Majesty. It changed the designation of the Governor-General of India to that of Viceroy of India. Thus, Viceroy of India was not a new official position. Viceroy was the direct representative of the British Crown in India. Lord Canning thus became the first Viceroy of India. It ended the system of double government by abolishing the Board of Control and Court of Directors. (Hence statement 2 is correct) It created a new office, Secretary of State for India, vested with complete authority and control over Indian administration. The secretary of state was a member of the British cabinet and was responsible ultimately to the British Parliament. (Hence statement 3 is incorrect) It established a 15-member Council of India to assist the secretary of state for India. The council was an advisory body. The secretary of state was made the chairman of the council. It constituted the secretary of state-in-council as a body corporate, capable of suing and being sued in India and in England. ‘The act of 1858 was, however, largely confined to the improvement of the administrative machinery by which the Indian Government was to be supervised and controlled in England. It did not alter in any substantial way the system of government that prevailed in India.’ Incorrect Solution (b) Government of India Act, 1858 (Act for the Good Government of India) This act known as the Act for the Good Government of India, abolished the East India Company, and transferred the powers of government, territories and revenues to the British Crown (Hence statement 1 is correct) It provided that India henceforth was to be governed by, and in the name of, Her Majesty. It changed the designation of the Governor-General of India to that of Viceroy of India. Thus, Viceroy of India was not a new official position. Viceroy was the direct representative of the British Crown in India. Lord Canning thus became the first Viceroy of India. It ended the system of double government by abolishing the Board of Control and Court of Directors. (Hence statement 2 is correct) It created a new office, Secretary of State for India, vested with complete authority and control over Indian administration. The secretary of state was a member of the British cabinet and was responsible ultimately to the British Parliament. (Hence statement 3 is incorrect) It established a 15-member Council of India to assist the secretary of state for India. The council was an advisory body. The secretary of state was made the chairman of the council. It constituted the secretary of state-in-council as a body corporate, capable of suing and being sued in India and in England. ‘The act of 1858 was, however, largely confined to the improvement of the administrative machinery by which the Indian Government was to be supervised and controlled in England. It did not alter in any substantial way the system of government that prevailed in India.’ Question 20 of 35 20. Question Which of the following statements is correct regarding the Indian Councils Act, 1909? It created a new office of the High Commissioner for Indian in London. The Governor-General was empowered to nominate one Indian member to his Executive Council. For the first time, separate representation for the Muslim community was introduced. How many of the above statements are correct?  a) Only one b) Only two c) All three d) None Correct Solution (b) Features of the Act of 1909: This Act is also known as Morley-Minto Reforms (Lord Morley was the then Secretary of State for India and Lord Minto was the then Viceroy of India). It considerably increased the size of the legislative councils, both Central and provincial. The number of members in the Central Legislative Council was raised from 16 to 60. The number of members in the provincial legislative councils was not uniform. It retained official majority in the Central Legislative Council but allowed the provincial legislative councils to have non-official majority. It enlarged the deliberative functions of the legislative councils at both the levels. For example, members were allowed to ask supplementary questions, move resolutions on the budget, and so on. It provided (for the first time) for the association of Indians with the executive Councils of the Viceroy and Governors. Satyendra Prasad Sinha became the first Indian to join the Viceroy’s Executive Council. He was appointed as the law member. (Hence statement 2 is correct) It introduced a system of communal representation for Muslims by accepting the concept of ‘separate electorate’. Under this, the Muslim members were to be elected only by Muslim voters. Thus, the Act ‘legalised communalism’ and Lord Minto came to be known as the Father of Communal Electorate. (Hence statement 3 is correct) It also provided for the separate representation of presidency corporations, chambers of commerce, universities and zamindars. Government of India Act of 1919 (Montagu-Chelmsford Reform) created a new office of the High Commissioner for Indian in London and transferred to him some of the functions hitherto performed by the Secretary of State for India. (Hence statement 1 is incorrect) Incorrect Solution (b) Features of the Act of 1909: This Act is also known as Morley-Minto Reforms (Lord Morley was the then Secretary of State for India and Lord Minto was the then Viceroy of India). It considerably increased the size of the legislative councils, both Central and provincial. The number of members in the Central Legislative Council was raised from 16 to 60. The number of members in the provincial legislative councils was not uniform. It retained official majority in the Central Legislative Council but allowed the provincial legislative councils to have non-official majority. It enlarged the deliberative functions of the legislative councils at both the levels. For example, members were allowed to ask supplementary questions, move resolutions on the budget, and so on. It provided (for the first time) for the association of Indians with the executive Councils of the Viceroy and Governors. Satyendra Prasad Sinha became the first Indian to join the Viceroy’s Executive Council. He was appointed as the law member. (Hence statement 2 is correct) It introduced a system of communal representation for Muslims by accepting the concept of ‘separate electorate’. Under this, the Muslim members were to be elected only by Muslim voters. Thus, the Act ‘legalised communalism’ and Lord Minto came to be known as the Father of Communal Electorate. (Hence statement 3 is correct) It also provided for the separate representation of presidency corporations, chambers of commerce, universities and zamindars. Government of India Act of 1919 (Montagu-Chelmsford Reform) created a new office of the High Commissioner for Indian in London and transferred to him some of the functions hitherto performed by the Secretary of State for India. (Hence statement 1 is incorrect) Question 21 of 35 21. Question Consider the following statements: The first Global Wind Day was celebrated in 2023 by the European Wind Energy Association (EWEA). The theme of Global Wind Day 2023 was “Pawan – Urja: Powering the Future of India”. Choose the correct code:  a) 1 only b) 2 only c) Both 1 and 2 d) Neither 1 nor 2 Correct Solution (b) Global Wind Day is an annual event since 2007 to promote wind energy as a clean and renewable source of power. Hence, statement 1 is incorrect. It was started by the European Wind Energy Association (EWEA) and later joined by the Global Wind Energy Council (GWEC). GWEC is a member-based organisation that represents the entire wind energy sector. Global Wind Day 2023 was celebrated on 15th June 2023 by the Ministry of New and Renewable Energy (MNRE) with the theme of “Pawan – Urja: Powering the Future of India”. Hence, statement 2 is correct. MNRE has set the target of 500 GW renewable energy capacity by 2030 and Wind Atlas at 150 meters above ground level was also launched by the National Institute of Wind Energy (NIWE), estimating the onshore wind potential at 1,164 GW. Incorrect Solution (b) Global Wind Day is an annual event since 2007 to promote wind energy as a clean and renewable source of power. Hence, statement 1 is incorrect. It was started by the European Wind Energy Association (EWEA) and later joined by the Global Wind Energy Council (GWEC). GWEC is a member-based organisation that represents the entire wind energy sector. Global Wind Day 2023 was celebrated on 15th June 2023 by the Ministry of New and Renewable Energy (MNRE) with the theme of “Pawan – Urja: Powering the Future of India”. Hence, statement 2 is correct. MNRE has set the target of 500 GW renewable energy capacity by 2030 and Wind Atlas at 150 meters above ground level was also launched by the National Institute of Wind Energy (NIWE), estimating the onshore wind potential at 1,164 GW. Question 22 of 35 22. Question Consider the following statements about the National Cooperative Union of India (NCUI): It is the apex body representing all sectors of the Indian co-operative movement. Its membership is not open to multi-state co-operative societies. It is organising Indian Cooperative Congress (ICC) with the theme of Amrit Kaal. How many of the above statements are correct? a) Only one b) Only two c) All three d) None Correct Solution (b) The National Cooperative Union of India (NCUI) is the apex body representing all sectors of the Indian co-operative movement. Hence statement 1 is correct. Its objectives include the promotion and development of the co-operative movement in India, to educate, guide and assisting the people in their efforts, and to build up and expand the co-operative sector. Its membership is open to national and state-level co-operative organisations as well as multi-state co-operative societies. Hence statement 2 is incorrect. As of 2016, NCUI had 260 members, including, 17 National, 163 State, and 80 Multi-State Cooperatives. It is organising Indian Cooperative Congress (ICC) with the theme of Amrit Kaal: Prosperity through Cooperation for a Vibrant India. Hence statement 3 is correct. It will be chaired by the Union Minister for Home and Cooperation. The objectives of the Indian Cooperative Congress are: To discuss various trends in the cooperative movement. To showcase the best practices being adopted by the successful cooperatives. To deliberate on challenges being faced by the cooperative world.   Incorrect Solution (b) The National Cooperative Union of India (NCUI) is the apex body representing all sectors of the Indian co-operative movement. Hence statement 1 is correct. Its objectives include the promotion and development of the co-operative movement in India, to educate, guide and assisting the people in their efforts, and to build up and expand the co-operative sector. Its membership is open to national and state-level co-operative organisations as well as multi-state co-operative societies. Hence statement 2 is incorrect. As of 2016, NCUI had 260 members, including, 17 National, 163 State, and 80 Multi-State Cooperatives. It is organising Indian Cooperative Congress (ICC) with the theme of Amrit Kaal: Prosperity through Cooperation for a Vibrant India. Hence statement 3 is correct. It will be chaired by the Union Minister for Home and Cooperation. The objectives of the Indian Cooperative Congress are: To discuss various trends in the cooperative movement. To showcase the best practices being adopted by the successful cooperatives. To deliberate on challenges being faced by the cooperative world.   Question 23 of 35 23. Question Consider the following statements about the Central Board for Film Certification (CBFC): It is astatutory body under the Cinematograph Act 1952. It works under the Ministry of Information and Broadcasting. Films can be publicly exhibited in India only after they are certified by the CBFC. How many of the above statements are correct? a) Only one b) Only two c) All three d) None Correct Solution (c) The Central Board for Film Certification (CBFC) is a statutory bodyunder the Cinematograph Act 1952. Hence statement 1 is correct. It is headed by a central government-appointed chairperson and 12-45 non-official members. The members are eminent persons from social science, education, law, arts, or films background appointed. It works under the Ministry of Information and Broadcasting. Hence statement 2 is correct. It is headquartered in Mumbai, Maharashtra. Films can be publicly exhibited in India only after they are certified by the CBFC. Hence statement 3 is correct. It examines films for content that may be harmful or unsuitable for some particular audiences, particularly children and young people. It ensures that films adhere to ethical standards, respecting cultural values and societal norms. After evaluating the content and classifying the film, the CBFC grants a certificate that permits the film’s public exhibition.   Note: Films are certified under four categories:- “U”: unrestricted public exhibition. “A”: restricted to adult audiences. “U/A”: unrestricted public exhibition subject to parental guidance for children below the age of twelve. “S”: restricted to specialized audiences such as doctors or scientists. The board may also refuse to certify. Incorrect Solution (c) The Central Board for Film Certification (CBFC) is a statutory bodyunder the Cinematograph Act 1952. Hence statement 1 is correct. It is headed by a central government-appointed chairperson and 12-45 non-official members. The members are eminent persons from social science, education, law, arts, or films background appointed. It works under the Ministry of Information and Broadcasting. Hence statement 2 is correct. It is headquartered in Mumbai, Maharashtra. Films can be publicly exhibited in India only after they are certified by the CBFC. Hence statement 3 is correct. It examines films for content that may be harmful or unsuitable for some particular audiences, particularly children and young people. It ensures that films adhere to ethical standards, respecting cultural values and societal norms. After evaluating the content and classifying the film, the CBFC grants a certificate that permits the film’s public exhibition.   Note: Films are certified under four categories:- “U”: unrestricted public exhibition. “A”: restricted to adult audiences. “U/A”: unrestricted public exhibition subject to parental guidance for children below the age of twelve. “S”: restricted to specialized audiences such as doctors or scientists. The board may also refuse to certify. Question 24 of 35 24. Question Consider the following statements about the United Nations Office for Outer Space Affairs (UNOOSA): It helps countries use space data and technologiesto prevent and manage disasters. It is headquartered in New York. The Office of the Director (OD) oversees and coordinates the strategic direction and operational priorities of UNOOSA. How many of the above statements are correct? a) Only one b) Only two c) All three d) None Correct Solution (b) The United Nations Office for Outer Space Affairs (UNOOSA) helps countries use space data and technologiesto prevent and manage disasters. Hence statement 1 is correct. Its objective is to help all countries, especially developing countries, access and leverage the benefits of space to accelerate sustainable development. It helps countries understand the fundamentals of international space law and increase their capacity to draft or revise national space law and policy. It supports transparency in space activities. It helps to promote sustainable development through space. It is headquartered in Vienna, Austria. Hence statement 2 is incorrect. It is headed by an Office of the Director and has five sections The Office of the Director (OD) oversees and coordinates the strategic direction and operational priorities of UNOOSA. Hence statement 3 is correct. Committee, Policy and Legal Affairs Section (CPLA) provide substantive, secretariat, organisational and administrative support. Space Applications Section (SAS) plans and implements the United Nations Programme on Space Applications. United Nations Platform for Space-Based Information for Disaster Management and Emergency Response (UN-SPIDER) is a programme of UNOOSA to leverage space data and applications for disaster risk reduction. The Executive Secretariat of the International Committee on GNSS (ICG) brings together all global navigation satellite system (GNSS) providers. The Office of the Director (OD) oversees and coordinates the strategic direction and operational priorities of UNOOSA.   Incorrect Solution (b) The United Nations Office for Outer Space Affairs (UNOOSA) helps countries use space data and technologiesto prevent and manage disasters. Hence statement 1 is correct. Its objective is to help all countries, especially developing countries, access and leverage the benefits of space to accelerate sustainable development. It helps countries understand the fundamentals of international space law and increase their capacity to draft or revise national space law and policy. It supports transparency in space activities. It helps to promote sustainable development through space. It is headquartered in Vienna, Austria. Hence statement 2 is incorrect. It is headed by an Office of the Director and has five sections The Office of the Director (OD) oversees and coordinates the strategic direction and operational priorities of UNOOSA. Hence statement 3 is correct. Committee, Policy and Legal Affairs Section (CPLA) provide substantive, secretariat, organisational and administrative support. Space Applications Section (SAS) plans and implements the United Nations Programme on Space Applications. United Nations Platform for Space-Based Information for Disaster Management and Emergency Response (UN-SPIDER) is a programme of UNOOSA to leverage space data and applications for disaster risk reduction. The Executive Secretariat of the International Committee on GNSS (ICG) brings together all global navigation satellite system (GNSS) providers. The Office of the Director (OD) oversees and coordinates the strategic direction and operational priorities of UNOOSA.   Question 25 of 35 25. Question It is the oldest and largest national park in Maharashtra. It is Maharastra’s second tiger reserve with tropical dry deciduous forest: The above statements refer to which of the following Tiger Reserve? a) Melghat Tiger Reserve b) Tadoba-Andhari Tiger Reserve c) Sahyadri Tiger Reserve d) Pench Tiger Reserve Correct Solution (b) Tadoba-Andhari Tiger Reserve is located in Chandrapur district in Maharashtra. It is Maharashtra’s oldest and largest national park. The reserve is the second Tiger Reserve in Maharashtra. It has a tropical dry deciduous forest with dense woodlands. It comprises about 67% of the protected area. Teak is the predominant tree species. The other trees are Bamboo, Ain, Bija, Dhaudab, Haldu, Salai, Semal, Shisham, Sisoo, Surya, and Sirus. Hence option b is correct. Note: The first Tiger Reserve established in the Maharashtra is Melghat Tiger Reserve Incorrect Solution (b) Tadoba-Andhari Tiger Reserve is located in Chandrapur district in Maharashtra. It is Maharashtra’s oldest and largest national park. The reserve is the second Tiger Reserve in Maharashtra. It has a tropical dry deciduous forest with dense woodlands. It comprises about 67% of the protected area. Teak is the predominant tree species. The other trees are Bamboo, Ain, Bija, Dhaudab, Haldu, Salai, Semal, Shisham, Sisoo, Surya, and Sirus. Hence option b is correct. Note: The first Tiger Reserve established in the Maharashtra is Melghat Tiger Reserve Question 26 of 35 26. Question Consider the following statements about the Order of the Nile: It is South Africa’s highest state honour instituted in 1915. It is a pure gold collarconsisting of three-square gold units comprising pharaonic symbols. It is conferred upon Heads of state, Crown Princes, and Vice-Presidents. How many of the above statements are correct? a) Only one b) Only two c) All three d) None Correct Solution (b) Recently, Egypt’s ‘Order of the Nile’ was conferred to India’s Prime Minister, Shri Narendra Modi. The Order of the Nile is Egypt’s highest state honour instituted in 1915. Hence statement 1 is incorrect. It is a pure gold collarconsisting of three-square gold units comprising pharaonic symbols. Hence statement 2 is correct. Pharaonic symbols were numerous symbols in the life of ancient Egyptians and varied in their symbols, rituals, and use. It is conferred upon Heads of state, Crown Princes, and Vice-Presidents who offer Egypt or humanity invaluable services. Hence statement 3 is correct. Incorrect Solution (b) Recently, Egypt’s ‘Order of the Nile’ was conferred to India’s Prime Minister, Shri Narendra Modi. The Order of the Nile is Egypt’s highest state honour instituted in 1915. Hence statement 1 is incorrect. It is a pure gold collarconsisting of three-square gold units comprising pharaonic symbols. Hence statement 2 is correct. Pharaonic symbols were numerous symbols in the life of ancient Egyptians and varied in their symbols, rituals, and use. It is conferred upon Heads of state, Crown Princes, and Vice-Presidents who offer Egypt or humanity invaluable services. Hence statement 3 is correct. Question 27 of 35 27. Question Consider the following statements about Rani Durgavati: She was born in 1524in the Chola dynasty. Madan Mahal Fortin Jabalpur is associated with Rani Durgavati. The University of Jabalpurwas renamed as Rani Durgavati Vishwavidyalaya. How many of the above statements are correct? a) Only one b) Only two c) All three d) None Correct Solution (b) Rani Durgavati was born in 1524in the Chandela dynasty. Hence statement 1 is incorrect. She is hailed as a patriotic rulerand defender of culture, becoming a symbol of pride and honour. Madan Mahal is a suburban area of Jabalpur famous for the historical Durgavati fort. The area also has a railway station named Madan Mahal. The fort dates back to 11th century AD. The fort is well associated with Rani Durgavati the Gond Queen and her son Veer Narayan. Rani Durgavati eventually died fighting the Mughals, and is hailed as a martyr in Indian history. Hence statement 2 is correct. The University of Jabalpurwas renamed Rani Durgavati Vishwavidyalaya by the Madhya Pradesh government in 1983. Hence statement 3 is correct. On June 24, 1988, the Indian government released a postage stamp in memory of her. The Durgavati Express (11449/11450) runs between Jabalpur Junction and Jammutawi and is named for the Queen. The third Inshore Patrol Vessel (IPV) of its kind, ICGS Rani Durgavati, was commissioned by the Indian Coast Guard on July 14, 2018.   Incorrect Solution (b) Rani Durgavati was born in 1524in the Chandela dynasty. Hence statement 1 is incorrect. She is hailed as a patriotic rulerand defender of culture, becoming a symbol of pride and honour. Madan Mahal is a suburban area of Jabalpur famous for the historical Durgavati fort. The area also has a railway station named Madan Mahal. The fort dates back to 11th century AD. The fort is well associated with Rani Durgavati the Gond Queen and her son Veer Narayan. Rani Durgavati eventually died fighting the Mughals, and is hailed as a martyr in Indian history. Hence statement 2 is correct. The University of Jabalpurwas renamed Rani Durgavati Vishwavidyalaya by the Madhya Pradesh government in 1983. Hence statement 3 is correct. On June 24, 1988, the Indian government released a postage stamp in memory of her. The Durgavati Express (11449/11450) runs between Jabalpur Junction and Jammutawi and is named for the Queen. The third Inshore Patrol Vessel (IPV) of its kind, ICGS Rani Durgavati, was commissioned by the Indian Coast Guard on July 14, 2018.   Question 28 of 35 28. Question Consider the following statements about the Mahadayi River: It originates in the Western Ghatsfrom the Bhimgad Wildlife Sanctuary in  Its left-bank tributaries are the Tambaraparani River, Bainganga River, and Wardha River. The Salim Ali Bird Sanctuaryis situated on it. How many of the above statements are correct? a) Only one b) Only two c) All three d) None Correct Solution (b) The Mahadayi River is also known as the Mandovi River. It originates in the Western Ghats from the Bhimgad Wildlife Sanctuary in the Belgaum district of Karnataka. Hence statement 1 is correct. It flows for about 81 km before emptying into the Arabian Sea. The river is formed by the confluence of two rivers: the Daddi and the Markandeya. Its left bank tributaries are the Daddi River, Malaprabha River, and Markandeya River. Its right bank tributaries are the Tambaraparani River, Bainganga River, Wardha River. Hence statement 2 is incorrect. Dams on the Mahadayi River are: The Hidkal Dam: Located in the Belagavi district of Karnataka. The Hidkal Dam: Located in the Belagavi district of Karnataka. The Selaulim Dam: Located in South Goa. The Virdi Dam: Located in the Belagavi district of Karnataka. The Salim Ali Bird Sanctuary is situated on the island of Chorao, which is located in the Mandovi River. Hence statement 3 is correct. The Mahadayi River is of great importance to the states of Goa and Karnataka, serving as a source of water for drinking, irrigation, and tourism.   Incorrect Solution (b) The Mahadayi River is also known as the Mandovi River. It originates in the Western Ghats from the Bhimgad Wildlife Sanctuary in the Belgaum district of Karnataka. Hence statement 1 is correct. It flows for about 81 km before emptying into the Arabian Sea. The river is formed by the confluence of two rivers: the Daddi and the Markandeya. Its left bank tributaries are the Daddi River, Malaprabha River, and Markandeya River. Its right bank tributaries are the Tambaraparani River, Bainganga River, Wardha River. Hence statement 2 is incorrect. Dams on the Mahadayi River are: The Hidkal Dam: Located in the Belagavi district of Karnataka. The Hidkal Dam: Located in the Belagavi district of Karnataka. The Selaulim Dam: Located in South Goa. The Virdi Dam: Located in the Belagavi district of Karnataka. The Salim Ali Bird Sanctuary is situated on the island of Chorao, which is located in the Mandovi River. Hence statement 3 is correct. The Mahadayi River is of great importance to the states of Goa and Karnataka, serving as a source of water for drinking, irrigation, and tourism.   Question 29 of 35 29. Question Consider the following statements about Florence Nightingale Awards: It is the highest national distinction a doctor can achieve for exceptional professionalism. It is given by the Ministry of Health and Family Welfare. It consists of a cash award of 50000/-, a certificate, and a medal. How many of the above statements are correct? a) Only one b) Only two c) All three d) None Correct Solution (b) The Florence Nightingale Award is the highest national distinction a nurse can achieve for selfless devotion and exceptional professionalism. Hence statement 1 is incorrect. Florence Nightingale was an English social reformer, statistician and the founder of modern nursing. She came to prominence while serving as a manager and trainer of nurses during the Crimean War. She organized to care for wounded soldiers at Constantinople. It is given by the Ministry of Health and Family Welfare. Hence statement 2 is correct. It was instituted as a mark of recognition for the meritorious services rendered by nurses and nursing professionals to society. The award consists of a cash award of 50000/-, a certificate and a medal. Hence statement 3 is correct. The award is given to outstanding Nursing personnel employed in Central, State/UTs, Private, Missionary and Voluntary Organizations. Incorrect Solution (b) The Florence Nightingale Award is the highest national distinction a nurse can achieve for selfless devotion and exceptional professionalism. Hence statement 1 is incorrect. Florence Nightingale was an English social reformer, statistician and the founder of modern nursing. She came to prominence while serving as a manager and trainer of nurses during the Crimean War. She organized to care for wounded soldiers at Constantinople. It is given by the Ministry of Health and Family Welfare. Hence statement 2 is correct. It was instituted as a mark of recognition for the meritorious services rendered by nurses and nursing professionals to society. The award consists of a cash award of 50000/-, a certificate and a medal. Hence statement 3 is correct. The award is given to outstanding Nursing personnel employed in Central, State/UTs, Private, Missionary and Voluntary Organizations. Question 30 of 35 30. Question Consider the following statements about the Artemis Accord: It is a non-bindingset of principles designed to guide military space exploration. Its founding members include Australia, Canada, Italy, and Luxembourg. Choose the correct code: a) 1 only b) 2 only c) Both 1 and 2 d) Neither 1 nor 2 Correct Solution (b) Artemis Accord is a non-binding set of principles designed to guide civil space explorationand use in the 21st century. Hence statement 1 is incorrect. It is a non-binding agreement with no financial commitments. It was established in 2020 by NASA, in coordination with the U.S. Department of State, and established the Artemis Accords with eight founder nations. Its founding members are Australia, Canada, Italy, Japan, Luxembourg, United Arab Emirates, the United Kingdom, and the US. Hence statement 2 is correct. Artemis Accords signatories as of May 2023 are Australia, Bahrain, Brazil, Canada, Colombia, Czech Republic, France, Israel, Italy, Japan, Luxembourg, Mexico, New Zealand, Nigeria, Poland, the Republic of Korea, Romania, Rwanda, Saudi Arabia, Singapore, Spain, Ukraine, the United Arab Emirates, the United Kingdom, and the United States. Incorrect Solution (b) Artemis Accord is a non-binding set of principles designed to guide civil space explorationand use in the 21st century. Hence statement 1 is incorrect. It is a non-binding agreement with no financial commitments. It was established in 2020 by NASA, in coordination with the U.S. Department of State, and established the Artemis Accords with eight founder nations. Its founding members are Australia, Canada, Italy, Japan, Luxembourg, United Arab Emirates, the United Kingdom, and the US. Hence statement 2 is correct. Artemis Accords signatories as of May 2023 are Australia, Bahrain, Brazil, Canada, Colombia, Czech Republic, France, Israel, Italy, Japan, Luxembourg, Mexico, New Zealand, Nigeria, Poland, the Republic of Korea, Romania, Rwanda, Saudi Arabia, Singapore, Spain, Ukraine, the United Arab Emirates, the United Kingdom, and the United States. Question 31 of 35 31. Question The smallest 6 digit number exactly divisible by 111 is a) 111111 b) 110011 c) 100011 d) 110101 Correct Solution (c) The smallest 6-digit number is 100000.   111) 100000 (900 0999 —– 100 — Required number = 100000 + (111 – 100) = 100011.   Incorrect Solution (c) The smallest 6-digit number is 100000.   111) 100000 (900 0999 —– 100 — Required number = 100000 + (111 – 100) = 100011.   Question 32 of 35 32. Question How many times does the 29th day of the month occur in 400 consecutive years? a) 4497 times b) 5500 times c) 3024 times d) 4400 times Correct Solution (a) Number of leap years in 400 years = 24 * 3 + 25 = 97 years. Hence, in 400 consecutive years February has the 29th day 97 times And the remaining eleven months have the 29th day = 400 x 11 = 4400 times. Thus the 29th day of the month occurs = 97 + 4400 = 4497 times   Incorrect Solution (a) Number of leap years in 400 years = 24 * 3 + 25 = 97 years. Hence, in 400 consecutive years February has the 29th day 97 times And the remaining eleven months have the 29th day = 400 x 11 = 4400 times. Thus the 29th day of the month occurs = 97 + 4400 = 4497 times   Question 33 of 35 33. Question If a two-digit positive integer has its digits reversed, the resulting integer differs from the original by 27. By how much do the two digits differ? a) 3 b) 4 c) 5 d) 6 Correct Solution (a) Let the two digit number be 10a + b, where ‘b’ is unit and ‘a’ is tens digit respectively. Thus reverse of the number will be 10b + a According to given data, (10a + b) – (10b + a) = 27 ->9a – 9b = 27 -> a – b = 3. ∴ the difference between two digits of the number is 3 Incorrect Solution (a) Let the two digit number be 10a + b, where ‘b’ is unit and ‘a’ is tens digit respectively. Thus reverse of the number will be 10b + a According to given data, (10a + b) – (10b + a) = 27 ->9a – 9b = 27 -> a – b = 3. ∴ the difference between two digits of the number is 3 Question 34 of 35 34. Question 40% of the women are above 30 years of age and 80 percent of the women are less than or equal to 50 years of age. 20% of all women play basketball. If 30 percent of the women above the age of 50 plays basketball, what percent of players are less than or equal to 50 years? a) 50% b) 60% c) 70% d) 80% Correct Solution (c) Let us assume the total number of women =100 Thus out of 100, women less than or equal to 50 years = 80 and, women above 50 years = 20 Number of women plays basketball = 20% of 100 = 20 30% of the women above 50 plays basketball = 30% of 20 = 6 So, remaining 14 women who plays basketball are less than or equal to 50 years. So the percentage of players who are less than or equal to 50 years = 14/20 x 100 = 70% Incorrect Solution (c) Let us assume the total number of women =100 Thus out of 100, women less than or equal to 50 years = 80 and, women above 50 years = 20 Number of women plays basketball = 20% of 100 = 20 30% of the women above 50 plays basketball = 30% of 20 = 6 So, remaining 14 women who plays basketball are less than or equal to 50 years. So the percentage of players who are less than or equal to 50 years = 14/20 x 100 = 70% Question 35 of 35 35. Question Find the length of the longest pole that can be placed in a room which is 24m long, 16m broad and 18m high.   a) 27 m b) 19 m c) 34 m d) 23 m Correct Solution (c) Given that, dimensions of room are 24m long, 16m broad and 18m high. The longest rod that can be placed in a room has length equal to the diagonal of the room. We know that diagonal of the cuboid = √(length^2) +(breadth^2) +(height^2) Length of the longest pole = √(24^2 + 16^2 + 18^2) = √576 + 256 + 324 = √1156 = 34 m   Incorrect Solution (c) Given that, dimensions of room are 24m long, 16m broad and 18m high. The longest rod that can be placed in a room has length equal to the diagonal of the room. We know that diagonal of the cuboid = √(length^2) +(breadth^2) +(height^2) Length of the longest pole = √(24^2 + 16^2 + 18^2) = √576 + 256 + 324 = √1156 = 34 m   window.wpProQuizInitList = window.wpProQuizInitList || []; window.wpProQuizInitList.push({ id: '#wpProQuiz_3495', init: { quizId: 3495, mode: 1, globalPoints: 70, timelimit: 0, resultsGrade: [0], bo: 704, qpp: 0, catPoints: [70], formPos: 0, lbn: "Test-summary", json: {"29399":{"type":"single","id":29399,"catId":0,"points":2,"correct":[0,0,1,0]},"29400":{"type":"single","id":29400,"catId":0,"points":2,"correct":[0,1,0,0]},"29401":{"type":"single","id":29401,"catId":0,"points":2,"correct":[0,0,1,0]},"29404":{"type":"single","id":29404,"catId":0,"points":2,"correct":[1,0,0,0]},"29406":{"type":"single","id":29406,"catId":0,"points":2,"correct":[0,1,0,0]},"29408":{"type":"single","id":29408,"catId":0,"points":2,"correct":[0,1,0,0]},"29409":{"type":"single","id":29409,"catId":0,"points":2,"correct":[0,0,0,1]},"29410":{"type":"single","id":29410,"catId":0,"points":2,"correct":[0,1,0,0]},"29412":{"type":"single","id":29412,"catId":0,"points":2,"correct":[0,0,1,0]},"29413":{"type":"single","id":29413,"catId":0,"points":2,"correct":[1,0,0,0]},"29416":{"type":"single","id":29416,"catId":0,"points":2,"correct":[1,0,0,0]},"29419":{"type":"single","id":29419,"catId":0,"points":2,"correct":[0,1,0,0]},"29420":{"type":"single","id":29420,"catId":0,"points":2,"correct":[0,0,0,1]},"29423":{"type":"single","id":29423,"catId":0,"points":2,"correct":[0,0,1,0]},"29425":{"type":"single","id":29425,"catId":0,"points":2,"correct":[0,1,0,0]},"29428":{"type":"single","id":29428,"catId":0,"points":2,"correct":[0,1,0,0]},"29429":{"type":"single","id":29429,"catId":0,"points":2,"correct":[0,1,0,0]},"29430":{"type":"single","id":29430,"catId":0,"points":2,"correct":[0,1,0,0]},"29431":{"type":"single","id":29431,"catId":0,"points":2,"correct":[0,1,0,0]},"29433":{"type":"single","id":29433,"catId":0,"points":2,"correct":[0,1,0,0]},"29435":{"type":"single","id":29435,"catId":0,"points":2,"correct":[0,1,0,0]},"29436":{"type":"single","id":29436,"catId":0,"points":2,"correct":[0,1,0,0]},"29439":{"type":"single","id":29439,"catId":0,"points":2,"correct":[0,0,1,0]},"29441":{"type":"single","id":29441,"catId":0,"points":2,"correct":[0,1,0,0]},"29443":{"type":"single","id":29443,"catId":0,"points":2,"correct":[0,1,0,0]},"29445":{"type":"single","id":29445,"catId":0,"points":2,"correct":[0,1,0,0]},"29448":{"type":"single","id":29448,"catId":0,"points":2,"correct":[0,1,0,0]},"29450":{"type":"single","id":29450,"catId":0,"points":2,"correct":[0,1,0,0]},"29453":{"type":"single","id":29453,"catId":0,"points":2,"correct":[0,1,0,0]},"29454":{"type":"single","id":29454,"catId":0,"points":2,"correct":[0,1,0,0]},"29455":{"type":"single","id":29455,"catId":0,"points":2,"correct":[0,0,1,0]},"29458":{"type":"single","id":29458,"catId":0,"points":2,"correct":[1,0,0,0]},"29460":{"type":"single","id":29460,"catId":0,"points":2,"correct":[1,0,0,0]},"29461":{"type":"single","id":29461,"catId":0,"points":2,"correct":[0,0,1,0]},"29462":{"type":"single","id":29462,"catId":0,"points":2,"correct":[0,0,1,0]}} } }); All the Best IASbaba